Вы находитесь на странице: 1из 199
Other Macmillan titles of related interest Microcomputer Applications in Sthuctural Engineering 'W. H. Mosley and W. J. Spencer Reinforced Concrete Design by Computer R. Hulse and W. H. Mosley Prestressed Concrete Design by Computer R. Hulse and W. H. Mosley Computer Spreadsiveet Applications in Building and Surveying B. Cooke and S. V. Balakrishnan Practical BASIC Programming P.E. Gosling Program your Microcomputer in BASIC PLE. Gosling. Giit Engineering Materials, Second Kaition N. Jackson (ed.) Plastic Methods for Steel and Concrete Structures Stuart §, J. Moy. ‘Strength of Materials, Third Edition G.H. Ryder A Guide 10 the Preparation of CWil Engineering Drawings M.Y. Thomas Structural Theory and Analysis, Second Edition J.D. Todd Energy Methods of Structural Analysis - Theory, worked examples and problems BLA. Young REINFORCED CONCRETE DESIGN W. H. Mosley and J. H. Bungey Department of Civit Engineering University of Liverpool ‘THIRD EDITION MACMILLAN EDUCATION ‘©W. H. Mosley and J. H. Bungey 1976, 1982, 1987 All rights reserved, No reproduction, copy or transmission (of this publication may be made without written permission No paragraph of this publication may be reproduced, copied or transmitted save with written permission or in accordance ‘with the provisions of the Copyright Act 1956 (as amended), ‘or under the terms of any licence permitting limited copying issued by the Copyright Licensing Agency, 33-4 Alfred Place, London WCIE 7DP. ‘Any person who does any unauthorised act in relation to this publication may be liable to criminal prosecution and civil claims for damages First published 1976 Reprinted 197, 1978, 1980, 1981 (twice) Second edition 1982 Reprinted 1983, 1984, 1985, 1986 Third edition 1987 Published by MACMILLAN EDUCATION LTD Houndmills, Basingstoke, Hampshire RG21 2XS and London Companies and representatives throughout the world Printed and bound in Great Britain at The Camelot Press ple, Southampton Bh Library Catatoguing in Publication Data Mosley, W. H, een Reinforced concrete design —3rd ed 1 Reinforced ones eanstaction Tie tl Bungey. 9H Soavesey PAR ’ ISBN 0-333-45182-1 ISBN 0-333-45183-X Pox Contents Preface to Third Edition Notation 1.1 Composite Action 1.2. Suess-Strain Relations 13 Shrinkage and Thermal Movement 14 Creep 1S Durability 1.6 Specification of Materials Limit States Characteristic Material Strengths and Characteristic Loads 23 Partial Factors of Safety 24 Global Factog of Safety 2 Limit State Design 3. Analysis of the Structure 3.1 Loads 3.2 Load Combinations 33. Analysis of Beams and Frames 3.4 Redistribution of Moments. 4 Analysis of the Section 4.1. Stress-Strain Relations 4.2 The Distribution of Strains and Stresses across a Section 4.3 Bending and the Equivalent Rectangular Stress Block 44° Singly Reinforced Rectangular Section in Bending 4.5 Rectangular Seetion with Compression Reinforcement at the Ultimate Limit State 4.6 Flanged Section in Bending at the Ultimate Limit State 4.7 Moment Redistribution and the Design Equations ‘CONTENTS: 48 Bending Plus Axial Load at the Ultimate Limit State 49 The Rectengular-Parabolic Stress Block 4.10 The Triangular Stress Block ‘Shear, Bond and Torsion Si Shear 5.2 Anchorage Bond $3 Laps in Reinforcement $4 Analysis of Section Subject to Torsionsl Moments Serviceability, Durability and Stability Requirements 6.1 Detailing Requiremonts 6.2 Span-Effective Depth Ratios 63 Calculation of Deflections 64 Flexural Cracking 65. Thermal and Shrinkage Cracking 6.6 Other Serviceability Requirements 67. Stability Design of Reinforced Concrete Beams 7A Preliminary Analysis and Member Sizing 7.2. Design for Bending 73. Design for Shear 74 Bar Spacing 7S. Continuous Beams 7 Cantilever Beams 7.7 Design for Torsion Devon of Rsinfoced Contes Site ‘Simplified Analysis £2 shesriSiae 8.3 Span-Effective Depth Ratios 8.4 Reinforcement Details 8.5. Solid Siabs Spanning in One Direction 8.6 Solid Slabs Spanning in Two Directions 8.7 Flat Slab Floors 8.8 Ribbed and Hollow Block Floors 8.9 Steir Slabs 8.10 Yield Line and Strip Methods otumn Desin ‘Loading and Moments 82 Short snd lees Columns 9.3 Reinforcement Details 9.4 Design of Short Columns 9.5 Nonrectangular Sections 9.6 Design of Slender Columns 9 88 1 98 103 106 107 412 113 47 119 134 141 144 147 1s4 156 159 174 180 180 187 188 192 192 193 198 200 201 209 24 222 26 230 239 229 2a 216 261 264 i CONTENTS 10 Foundations 10.1 Pad Footings 10.2 Combined Footings 103 Strap Footings 10.4 Strip Footings 10.5. Raft Foundations 10.6 Piled Foundations 11 Watersetaining Structures and Retaining Walls 11.1 Watersretaining Structures 11.2 Joints in Water-retaining Structures 11.3 Design Methods 114. Reinforcement Details 11.5. Retaining Walls 12 Prestressed Concrete 12.1 Principles of Prestressing 12.2. Methods of Prestressing 123 Analysis of Concrete Section Under Working Loads 12.4 Design for the Serviceability Limit State 12.5. Analysis and Design at the Ultimate Limit State Appendix Further Reading Index 270 23 280 284 286 289, 291 296 296 298 302 312, 316 327 327 330 333, 339 365 380 384 386 Preface to Third Edition ‘The purpose of this book is to provide a straightforward introduction to the principles and methods of design for concrete structures It is divected primarily at students and young designers who requie an understanding ofthe basic theory and a concise guide to design procedures. Although the detailed design methods are generally according to British Standards, much of the theory and practice is of a fundamental natute and should, thezefoze, be useful to engineer: in other countries. Limit state concepts, as cecently introduced in the new Codes of Practice, are used and the calculations are in SI units throughout, ‘The subject matter has been arranged so that chapters 1 0 5 desl mostly with theory and analysis while the subsequent chapters cover the design and detailing of ratious types of member and structure. In order to include topics that are usually in an undergraduate course, there is chapter on earth-etaining end ‘woterztaining structures, and also a final chapter on prestessed concrete, Important equations that have been dorived within the text are highlighted by an asterisk adjacent to the equation number. In preparing the third edition of this book the opportunity has bean taken to rearrange the sequence of presentation of some material and make amendments to reflect developments in design practice. The principal changes ar associted ‘ith the introduction of the revised British Stendatd BS 8110 to replace CP 110. ‘This has led to extensive detlled changes, especially to numerial examples, many ‘of which are associated with the aim of achieving improved durability performance It should be mentioned that standard Codes of Practice such as B31 10 are always liable tobe revised, and readers should ensure that they ere using the latest edition of any relevant standard “Extracts from the British Standards are reproduced by permission ofthe British Standards Institution. 2 Park Street, London WIA 2BS, from whom complete copies can be obtained Finally, the authors wish to thank Mss B. Cotgreave wo prepared the diagrams and Mrs F. Zimmermann who typed most of the draft and final copies ofthe rmanuscript Notation ‘Notation is gencrally in accordance with BS 8110, and the principal symbols are listed below, Other symbols are defined in the tex! whore necessary. The symbols ‘for strain and f for stress have been adopted throughout, with the general system ‘of subscripts such that the frst subscript refers to the material, e— concrete, 's— steel, and the second subscript rafers to the type of stress. ¢ — compression, t— tension, Ay Cross-sectional ates of tension reinforcement ‘AL Cross-sectional aree of compression reinforcement Agy Cross-sectional area of shear reinforcement in the form of bentup bars ‘Age Cross-seetional area of shear reinforcement in the form of links a Deflection > Width of section by Breedth of web or rib of a member By, Breadth of web or rib of a member d” Effective depth of tension reinforeement d’ Depth to compression reinforcement E, Static secant modulus of elasticity of concrete E, Modulus of elasticity of steel e Eccentricity F Ultimate load fox Characteristic concrete cube strength. Characteristic strength of prestressing tendons Service stress or steel stress Characteristic strengti of reinforcement Characteristic strength of link reinforcement POND, Ge Characteristic dead load ge Characteristic dead load per unit length or area Ay Overell depth of section in plane of bending hig Thickness of flange T Second moment of area ; ky Average compressive stress in the concrete for a rectangular-parabelic stress Block T SESE" SPP MEARE Px NOTATION ‘A factor that relates the depth to the centroid of the rectangular- parabolic stess block and the depth of the neutral axis Lever-arm factor = #/d [fective height of @ column or wall Bending moment Ultimate moment of resistance ‘Axial load Ultimate Load per unit area ‘Axial load on @ column corresponding to the balanced condition Final presress force (chapter 12) Characteristic imposed load Characteristic live load per unit length or area Curvature of a beam at point x Depth of equivalent rectangular stress block Spacing of links along the member Torsional moment Perimeter Shear force Shear stress Ultimate shear stress in concrete Characteristic wind load Ultimate load per unit length Neutral axis depth Lever arm Modular ratio Partial safety factor for load Paria afety factor For strength Shrinkage strain Coefficient of friction Bar size Creep coefficient 1 Properties of Reinforced Concrete Reinforced concrete is a strong durable building material that ean be formed into ‘many varied shapes and sizes ranging from a simple rectangular column, to a slender curved dome or shell Its utility and verstatility is achieved by combining the best features of concrete and steel, Consider some of the widely difTering properties of these two materials that ae listed below. Concrete Steel stuength in tension poor good strength in compression good good, but slender bars wall buckle strength in shear fair durability good corrodes if unprotected fire resistance good poor ~ suffers rapid loss of strength st high temperatures It can be soon from this list that the materials ere snore or less complementary. ‘Thus, when they aze combined, the steel is able to provide the tensile sirength and probably some of the shear strength while the concrete, strong in compression, protects the steel to give durability and fie resistance. This chapter can present ‘only a brief introduction to the basic properties of concrete and its steel reinforce- ‘ment. For a more comprehensive study, itis recommended that reference should ‘be made to the specialised texts listed in Further Reading at the end of the book. 1.1 Composite Action ‘The tensile strength of concrete is only about 10 per cent of the compressive strength. Because of this, nearly all reinforced conerete structures are designed on the assumption that the concrete does not resist any tensile forces, Reinforcement is designed to cerry these tensile forces, which are transferred by bond between ‘the interface of the two materials If this bond is nat adequate, the reinforcing bars 1 2 REINFORCED CONCRETE DESIGN ‘will just slip within the concrete and there will not be a composite action, Thus ‘members should be detailed so that the concrete can be well compacted around the reinforcement during construction. In addition, some bars are ribbed or twisted so that there is an extra mechanical grig. In the analysis and design of the composite reinforced concrete section, itis assumed that there is perfect bond. so that the strain in the reinforcement is identical to the strain in the adjacent conczete, This ensuses that thece is what is known as compatibility of strains’ across the cross-section of the member, The coefficients of thermal expansion for steel and for concrete are of the order of 10 x 10~® per °C and 7-12 x 10° per °C respectively. These values ate sufficiently close that problems with bond seldom arise from differential expan sion between the to materials over normal temperature ranges, loo A rt werner 4 WA som seen a sstrostion aa Reintorcament hacking Figure 1.1 Compasite action Figure |.1lutrates the behaviour ofa simply supported bearn subjected to bending and shows the position of steel reinforcement to resist the tense forces, ‘while the compression forces inthe top of the beam are caried by the conesete. ‘Wherever tension oveurs its lkely that cracking of the concrete will take place ‘his cracking, however, does not detract from the safety of the structure provided there is ood reinforcement bone to ensure that the cracks aro restrained from ‘pening so thatthe embedded steal cantinues to be protected from corrosion When the compressive or shearing forces exceed the strength of the concrete, then tee einforcement must again be provided, but in thes case itis only required to supplement the load-carrying capacity ofthe concrete. For example, compression reinforcement is generally required in a coluran, where it takes the form of vertical bars spaced near the perimeter. To prevent these bors buckling, steel binders are used to assist the restraint provided by the surrounding concrete. 1.2 Stress-Strain Relations The loads on a structure cause distortion of its members with resulting stresses and strains in the concrete and the steel reinforcement. To carry out the analysis and ‘cosign of a member it is necessary to have a knovledge of the relationship between these stresses and strains, This knowledge is particularly important when dealing ‘with reinforced concrete which is « composite material for in this case the analysis PROPERTIES OF REINFORCED CONCRETE 3 of the stresses on a crosssection of a member must consider the equilibrium of the forces in the concrete and stecl, and also the compatiblity of the strains across the erosssection. 1.2.1 Conerere Concrete isa very variable material. having a wide range of strengths and stress- sirain curves. A typieal curve for concrete in compression is shown in figure 1.2. ‘As the load is applied, the vatio between the stresses and strains is approximately liner at fizst and the concrete behaves almost as an elastic material with virtually 2 full recovery of displacement ifthe load is removed. Eventuslly, the curve is 10 longer linear and the concrete behaves more and more as. plastic materiel. IF the load were removed during the plastic range the recovery would no longer be com plete and a permanent deformation would remain. The ultimate strain for most structural concretes tends to be a constant value of approximately 0.0035, inres- peetive of the strength of the conerete. The precise skape of the curve is very dependent on the length of time the loud is applied, a {actor which willbe further discussed in section 1.4 on ereep. Figure 1.2 is typical for a short-term loading, stress Siren Figure 1.2 Seres-train curr for concrete compression Concrete generally increases its strength with age. This characteristic is lustra- ted by the graph in figure 1.3 which shows how tho increase is rapid at first, ‘becoming more gradual later. Some codes of practice allow the concrete stronath ‘used in design to be varied according to the age of the concrete when it supports the design load. A typical variation in strength of en adequately cured Ordinary Portland cement concrete, as suggested by BS 8110, is 7 days Lmonth 2months 3 months 6 months 1 year 20 30 33 35 36 37 Nim? BS 8110 does not permit the use of strengths greater than the 28-day value iB calculations, but the Modulus of Elasticity may be modified to account for age as shown overleaf. 4 REINFORCED CONCRETE DESIGN m _ i a 8 OF 7 26S 1 Ss Age of concrete (leg scale! Figure 13 Increse of concrete strength with age. Typleel curve far an Ordinary Postand cement concrete ‘Modulus of Elasticity of Conerete It is seen from the stress-strain curve for concrete that although elastic behaviour may be assumed for stresses below about one-third of the ultimate compressive strength, this relationship is not truly linear. Consequently itis necessary to define precisely what value is to be taken as the modulus of elasticity stress stain A number of alternative definitions exist, but the most commonly adopted is E=E, where B, isknown as the secant or static modulus. This is measured for a particular concrete by means of a stati test in which 2 cylinder is loaded to just above one-thitd of the corresponding control cube stress and then cycled back to zero stress. This removes the effect of initial “bedding in’ and minor stress redistri- ‘butions in the concrete under load. Lond is then reapplied and the behaviour will ‘then be almost linear the average slope ofthe line up to the specified stress is, token asthe value fer Ee. The testis described in detail in BS 1881 and the result fs generally known as the nstansoneous statie madulus of elasticity. ‘The dynamic modulus of elasticity. Fg, is sometimes referred to since this is ‘much easier to measure in the laboratory and there is fairly well-defined relation- ship between Ey und Fcq. The standard testis based on determining the resonant frequency of a laboratory prism specimen and is also described in BS 1861. tis also posible to obtain a good estimate of fq from ulteasonie measuring techni ues, which may sometimes be used on site To asses the concrete in an actuel structure, The standard test for Feq is on an unstressed specimen. It can be seen from figure 1-4 that the value obtained represents the slope ofthe tangent at 2er0 stress anid Bog is therefore higher than E¢. The relationship between the two modu is given by Static modulus £, = (1.25 Beg ~ 19) kN/onm? PROPERTIES OF REINFORCED CONCRETE 5 ‘strass Eqldynomic) Eetetatic? Figure 14 Moduli of easttey of concrete ‘This equation is sufficiently accurate for normal design purposes. ‘The actual value of £ for a concrete depends on many factors related to the imix, but a general relationship is considered to exist between the modulus of clasticity and the compressive cube strength, Ranges of E, for various concrete grades whieh are suitable for design are shown in table 1.1. The magnitude of the ‘modulus of elasticity is required When investigating the deflection and cracking of aastructure, When considering short-term effects, member stiffnesses will be based fon the static modulus Bg, as defined above. If long-term effects are being consider- ced, it can be showa that the effects of creep can be represented by modifying the value of E, and this is discussed in section 6.3.2 Table 1.1 Short 1m modulus of elasticity of concrete 28 day characteristic Statie modulus Ee.25 ccube strength (Nim?) fes.28, (Xfm?) ‘Typical range Mean 5 19-31 5 30 20-32 26 40 22-34 28 50 24-36 30 60 26-38 2 ‘The clastic modulus at an age other than 28 days may be estimated from Eoyg* Be 004+ 06 feud feu28) 6 REINFORCED CONCRETE DESIGN (0) High yreta staat BcaF strom Figure LS Stressstirin curves for stest 1.22 Steet Figure 15 shows typical stress-strain curves for (a) mild steel, and (3) high yield stec], Mild steel behaves as an elastic material, with the strain proportional to the ‘ress up tothe yield, at which point there is «sudden ineresse in strain with no change in stress. After the yield point, mild steel becomes a plastic material and the sirain increases rapidly up to the ultimate value High yield ste! on the other hhand, does not have «definite yielé point but shows more greduel change from, an elastic toa plastic behaviour ‘The specified strength used in design is based on the yield stress for mild steel, whereas for high yield steel the strength i based on a specified proof ste, A 0.2 per cent proof sires is defined in figure 1.5 by the broken line drawn peraliel to the lear part of the stress-strain curve Removal ofthe 1oad within the plastic range would result in the stress-strain diagram following line approximately parallel to the loading postion ~ se line BC in figure 1.6, The steel will be left witha permanent strain AC, which i known as “sip: IF the steel is again loaded, the stress-strain diagram will follow the un- loading curve until it almost reaches the original stress at B and then it will eurve jn the dizecton of the fist loading. Thus, the proportional limit forthe second loading i higher than for the initial loading. This action is efeured to as ‘strain hardening’ or ‘work hardening strase ff Stroin Figure 1.6 Stain hardening PROPERTIES OF REINFORCED CONCRETE ‘The deformation of the steel is also dependent on the length of tne the loed is applied, Under a constant stress the strains wil gradually increase — this pheno- renon is known as ‘cteep” or ‘relaxation’. The amount of creep that takes place cover period of time depends on the grade of ste! and the magnitude ofthe stress. Creep of the steel is of ttle significance in normal reinforced concrete work, but its an important factor in prestressed concrete where the presteessing steel very highly stressed 1.3 Shrinkage and Thermal Movement ‘As concrete hardens there is 4 reduction in volume. This shrinkage is liable to cause ‘racking of the concrete, but it also has the beneficial effect of strengthening the bond between the concrete and the steel reinforcement. Shrinkage begins to take place as soon as the conerete is mixed, and is caused initially by the absorption of the water by the concrete and the aggregate. Further shrinkage is caused by evaporation of the water which rises to the conerete surface. During the setting process the hydration of the cement causes a great deal of heat to be generated, and as the conerete cools, further shrinkage takes place as a result of thermal contraction. Even after the concrete hus hardened, shrinkage continues as drying ut persists over many months, and any subsequent wetting and drying can also ‘cause swelling and shrinkage. Thermal shrinkage may be reduced by restricting the temperature rise daring hydration. which may be achioved by the following provedures, (1) Use a mix design with a low cement content, (2) Avoid rapid hardening and finely ground oement if possible, G) Keep aggregates and mixing water cool (@) Use steel shuttering and cool with a water spray. (8) Strike the shuttering early to allow the heat of hyération to dissipate, ‘A low water-cement ratio will help to reduce drying shrinkage by keeping to a minimum the volume of moistuce that can be lost. If the change in volume of the concrete is allowed to take place freely without ‘restraint, there will be no stress change within the concrete. Restraint of the shrinkage, on the other hand, will cause tensile strains and stresses. The restraint ‘may be caused externally by fixity with adjoining members or friction against an earth surface. and internally by the action of the steel reinforcement. Fore long wall or floor slab. the restraint from adjoining concrete may be reduced by using system of consinicting successive bays instead of alternate bays. This allows the free ond of every bay 10 contract before the next bay is cast. Day-to-day thermal expansion of the conereto can be greater than the move: ‘ments caused by shrinkage, Thermal stresses and strains may be controlled by the seetion or Lsection, 4 REINFORCED CONCRETE DESIGN Frames Supporting Vertical and Laterat Loads feral loads on a structure may be caused by wind pressures, by rtsined earth, x tyolnie fren An brand rsmeatjesd toting force mst be ana ed forall the three loading combinations described in section 3.2.1, The vertical loading analysis can be caried out by the methods deseribed previsly for braced frames (ee page 35). The analysis forthe lateral loads should be kept separate and ‘the focees may be calculated by an elastic analysis or by s simplified approximate smathod. For preliminary design calculations, and alo for mediumsize regular Structures, 2 simplitied analysis may well be adequate. BS 8110 recommends that ary simplified form of analysis should assume points of contrflexure at the mid-lengths of al the columns and beams. A sult- ole approximate analysis isthe cantilever method. I assumes tt (1) Points of contraftexure are located at the mid.points of all columns and beams: and (2) The direct axial loads in the columns are in proportion to thelr distances from the centre of gravity of the frame. It ig also usual to assume that all the colurans in a storey are of equal cross-sectional area, Application of this method is probably best illustrated by an example, as follows, Example 3.5 Simplified Analysis for Lateral Loads — Cantilever Method Figure 3.20 shows a building frame subjected to characteristic wind loud of 3.OKN per mette height of the frame, This load i assumed to be transferred to the frame asa concentrated load at each floor level as indicated inthe figure. By inspection, there is tension in the two colurans tothe left and compression in the columns to the right; and by assumption 2 the axial foroes in columns are Proportional to their distances from the centre lie ofthe frame, Thos axial force in exterior column: axial force in interior column = 4,0P 1.0P The analysis ofthe frame continues by considering a section through the top- storey columns: the removal of the frame below this section gives the remainder show in figure 3.214. The forces in this subframe are calculated as fellows. (a) Axial Forces in the Columns Taking moments about point s, EM, = 0, therefore 5.25% 1.75 +Px6.0~Px 10.0 4Px 16.020 and therefore P= 0.135 kN thus ANALYSIS OF THE STRUCTURE 45 ease g EA] sou 4th 5 SE rose we] 3 a on am | & i a #4 wo «| 4 “| a ar wr q 2.0m 4AOm_60m Figure 3.20. Frome with lzort food (b) Vertical Shearing Forces Fin the Beams For each part of the subframe, 2 Fy = Ny =0.540N Py Ny +My = 0,675 kN therefore (©) Horizontal Shearing Forces # in the Columns ‘Toking moments about the points of contraflexure of each beam, EAf=0, therefore Hy 3.95 Ny 03. Hy =0.93 kN and (Hy +H) 1.75 — Ny ¥8.0— Ns x 2.0 Hy =1.70kN ‘The caleulations of the equivalent forces forthe Fourth floor (figure 3.21) follow a sunilar procedure as follows, (4) Axial Forces in the Columas For the frame above section tt’, 24M, = 0, therefore $.25 (3 x 1.75) + 10.8 x 1.75 +Px 6.0 —P x 100 — 4P x 16.0 PHOGTSEN therefore Ny =4.0P = 2.70 kN fy = 1.0P = 0.68 kN 4% REINFORCED CONCRETE DESIGN ANALYSIS OF THE STRUCTURE 97 1054 20879 :0544N fist loor Ny =2437KN My = 6.09KN se, t t Fy =10.23KN Fy =12.79 KN iyi l. fossa cto slgcoss lye 2099 ‘Te bending moments in the beams an columns a thet connections ean be 4 caloulated from these results by the following formulae sy beams Mp =F x $ beam span ‘e) Root columns Mfc = A$ storey height ose ome ons oes sat theron central ommecton ont vot sod oad My = 054x460 236 27s | 26 as o16ENm 10 t Me =0.93x 4x35 v 1 | hrs 2 16ENm them Les fos vere i rot r 20 ok ote 20 co) 4th Foor Figure 3.21 Subjrames a! rhe roof and fourth floor (€) Beam Shears A A 2.70 —0.54=2.16kN 2.70 +0,68 — 0.54 — 0.135 = 2.705 KN @) Column Shears Hy «1.75 40.93% 1.75 (2.70 0.54)3,.0=0 Hy =2.78kN Hy = $05 +8.28) ~ 2.78 =5.1 N Values calculated for sections taken below the remaining floors are third floor Ny = 7.03 kN Fy = 433 kN Hy =4.644N second floor Ny = 14.14 kN. Fy =T11KN Hy = 6.61 kN Ny = 1.76KN Fy =5.A1KN Hy ~8.49 KN My =3.53 kN Py =8.88 kN Hy = 12.145N 11g 1 16 Fea He 30 16 |__ssl_sa__és} a0 49 Ss a4 65 = 49 129] os BS es 81 Tio oe 150 a asl a: [a3 ie las saa 202] se]? 202 2, hs 220 Kor fee bse v5. ame aha 1 es 4 External ® ° 244 stare! ‘Colne Beoms olumn Figure 3.22, Moments (RN m) and reactions (kN Asa check at exch joint, 2Mfy = DM ‘The bending moments due to characteristic wind loads in all the columns and beams of this structure are shown in figure 3.22, 48 REINFORCED CONCRETE DESIGN 3.4 Redistribution of Moments, Some method of elastic analysis is generally used to ealeulate the forces in a con rote structure, despite the fact that the structure does not behave elastically near its ultimate load, The assumption of elastic behaviour is reasonably tme for low stress levels; but a8 2 section approaches its ultimate moment of resistance, plastic ‘deformation will occur. This is recognised in BS 8110, by allowing redistribution ff the elastic moments subject to certain limitations. Reinforced concrete behaves in a manner midway between that of steet and concrete, The stress-strain curves for the two materials (Bgures 1.5 and 1.2) show the elastoplasic behaviour of steel and the plastic behaviour of concrete. The latter will fil ata relatively small compressive strain. The exact behaviour ofa reinforced conerete section depends on the relative quantities and the individuel properties of the two materials, However, sucha section may be considered virtually elastic until the stel yielés; and then plastic uatil the concrete fails in compression, Thus the plastic behaviour is limited by the concrete failure; or more specifcaly, the conoret failure limits the rotation that may take place ata seetion in bending. A typical moment-curvature diagram for a reinforeed eonerete member is shown in figure 3.23, EYES L concrate ast crock SPUN curvature: Figure 3.23. Typice! momencicurature diagram Thus, in an indeterminate structure, once a beam section develops its ultimate moment of resistance My. i then behaves as a plastic hinge resisting a constant moment of that value. Further loading must be taken by other parts of the struc- ture, with the changes in moment elsewhere being just the same as if areal hinge existed. Provided rotation of a hinge does not cause crushing of the conerete, further hinges will be formed until a mechanism is produced. This requirement is considered in more detail in chapter 4 ANALYSIS OF THE STRUCTURE 49 Example 3.6 Moment Redistribution ~ Single Span Fixed-end Beam ‘The beam shown in figure 3.24 is subjected to an increasing uniformly distributed load. we 1 wi? Elastic span moment = “4 lastic span mer oA Elastic support moment In the case where the ultimate bending strengths are equal at the span and at the supports; and where adequate rotation is possible, then the additional load w, ‘which the member can sustain by plastic behaviour, can be found. stunt langth pooeiettienatetitaneny Loos tt iat Elastic BMD 8 Moles, a = © oe Additvona! erarnantsalogram <=” inges ct Aone) OS™ callopse eenanisin » seo Elestic © MO (Cottons loecs) zl Final Collapse BMD Figue3.24 Moment redistribution - one span beam At collapse wh? = WET + additional mid-span momento, * 1 rvid-span mi B where my © (wy£7)/8 as for a simply supported beam with hinges at A and C. Thus ee Boot ‘where is the load to cause the first plastic hinge; thus the beam may carry a load of 1.33 with redistribution. 50 REINFORCED CONCRETE DESIGN From the design potat of view, the elastic bending-moment diagram can be obtained for the required ultimate loading inthe ordinary way. Some of these ‘moments may then be reduced; but this will necessiate increasing others to min: tain the static equilibrium of the structure. Usually it is the maximum support moments which are reduced, so econemising in reinforcing stee| and also reducing congestion at the columns. The requirements for applying moment redistribution (@) Equilibrium between internal and extemal forces must be maintained, hence it is necessary to recalculate the span bending moments end the shear forces for the load case invalved. (b) At sections of largest moment the depth of neutral axis, x, is imited by x> Gy —04)d ‘where d = the effective depth, and ‘moment at section after redistribution moment at seetion before redistribution Ae This rule effectively prevents any reduction of the moments in columns Wich are priarily compression members with large values of x, and this is dealt with more fully in chapter 4 ———Eostie moments —-———Resietributed maments = 70% of elastic moments Reaisiebured decign moments, Figure 3.25. Redisertbuton of hogstng moments ANALYSIS OF THE STRUCTURE sl (©) The moment of resistance of any section should be at least 70 per cent of the moment from the elastic analysis, hence allowing up to 30 per cent redistribution, This requirement ensures that there can be no ‘movement in the position of the points af vontraflewuze obtained fron ‘the elastic analysis as shown by figure 3.25. I thus also ensures that a sufflefent length of tension reinforcement is provided to resist cracking atthe serviceability limit state For unbraced structures over four storeys the redistribution is 10 10 per eent, to prevent lateral instability. ited Example 3.7 Moment Redistribution In example 3.3, figure 3.14 it is required to reduce the maximum support moment of Mga = 15OKN mas much as possible, but without increasing the span moment above the preseat maximum value of 126 KN m. 1302 1124150 2 10 7 ’ @ c D Tai a la} Grigine! Moments, txtim) . 12 opt? ” Woe 10 an Se - SS 128 125 (m) Reaisteisutes Moments tet 102 yes 102 168 wr (fe) Shears, (KN) Figure 3.26 Moments and sheors wth redistribution Figure 3.26a duplicates the original bending-moment diagram (part 3 of figure 3.14) of example 3.3 while figure 3.26b shows the redistributed moments, with the span moment set at 126 KN m. The moment at support B cen be calculated, using 8 rearrangement of equations 3.4 and 3.1, Thus Va = VMecax — Map) 20] and sta (Van 2) tan 2 REINFORCED CONCRETE DESIGN 1 kN/m, therefore Van = V{(026-+ 70) x2 x 51] = 141 kN (1-288) oon 142 KN my and 06 — 141 = 165 kN Reduction in Mya = 150 — 142 kNm _ 8x 100 150 Yan = 5.3 per cent In order to ensure that the moments in the columns at joint B are not changed by the distribution, moment Myc must also be reduced by 8 KN m. Therefore Myc =112-8=104KNm hogging Figure 3,26e shows the revised shearing-force diagram to accord with the redistei- buted moments. This example illustrates how, with redistribution (1) the moments ata section of beam oan be reduced without exceeding the maximum design moments at other sections (2) the values of the column moments are not affected; and (3) the equilibrium between external loeds and internal forces is meintained, 4 Analysis of the Section [A satisfactory and economic design of a concrete structuse rarely depends on a complex theoretical analysis. It is achieved more by deciding on a practical over- all layout of the structure, careful attention to detai] and sound constructional practice, Nevertheless the total design of a structure does depend on the analysis ‘and design of the individual member sections. Wherever possible the analysis should be kept simple, yet it should be based on. the observed and tested behaviour of reinforced concrete members. The thanipula- tion and juggling with equations should never be allowed to obscure the fund meatal prineiples that unite che anelysis. The three most important principles are (J) ‘The strestes and strains are rlated by the material properties, neduding the stress-strain curves for concrete and steel. (2) The distribution of strains must be compatible with the distorted shape of the eross section (3) The resultant forces developed by the section must balance the applied loads for static equilibrium, ‘These principles are true irrespective of how the stresses and strains are distributed, ‘or how the member is loaded, or whatever the shape ofthe crosssection. ‘This chapter describes and antlyses the action of a member section under load. It derives the basle equations used in design and also those equations required for ‘the preparation of design charts. Emphasis has been placed mostly on the analysis associated with the ultimate limit state but the behaviour ofthe section within the clastic eange and the serviceability Hit state has also been considered. Seetion 4.7 deals with the redistribution of the moments from en elastic a of the structure, and the effect it hes on the equstions derived and the design procedure, ys 4,1 Stress-Strain Relations Short-term stress-strain curves are presented in BS 8110. These curves are in an idealised form which can be used in the analysis of member sections. 3 34 REINFORCED CONCRETE DESIGN 4.1.1 Concrete ‘The beheviour of structural concrete (figure 4.1) is represented by a parsbolic strese-strain relationship, up toa strain ¢9, from which point the strain increases while the stress remains constant. Strain ¢ is specified as a function of the characteristic strength of the concrete (fey), a8 is also the tangent modulus at the origin, The ultimate design stress is given by DSF fon = 16S = 0.447 fg = 0.45 fo om 5 whore the factor of 0.67 allows for the difference between the bending strength ‘nd the cube crushing strength of the concrete, and 7q,~ 1.5 isthe usual partial safety factor for the strength of concrete when designing members cast sit ‘The ultimate strain of 0.0035 is typical for all grades of concrete, O67 fey Farapone i fy -2-annoa [te 00035, strain Figure 4.1. Short-term design stets-strain curve for concrete in comprewtion 4.1.2 Retnforcing Steel ‘The representative short-term design stress-strain curve for reinforcement i given in figure 4.2. The behaviour ofthe stel i identical in tension and compression, being linear in the elastic range up to the desi yield stress of fy yg, Where fy 18 ‘the charecerste yield stress and 7 i the partial factor of safety Within the elastic range the relationship between the stres and stain is stress elastic modulus x strain Axe ep so that the design yield strain is ANALYSIS OF THE SECTION 3s Tension ond Compression 200KN/mm? Stroin Figure $.2. Short-term design sessstratn curve for reinforcement At the ultimate limit for fy = 460 Nfram? = 460)(1.15 x 200x 10°) =0002 andfor fy #250 N/mm? cy = 250/(1.15 x 200 x 10°) 00109 nfsey 5 =0-9x eae ) | eautest | [ewe rr) ws tmongulor rectengular+ euivetent porte: ceetanguar section strains stress sioens Figure 4.3. Section with strain digrem and stress Blocks 4.2 The Distribution of Strains and Stresses across a Section The theory of bending for reinforced concrete assumes that the concrete will crack in the rogions of tensile strains and that, after eracking, all the tension is 56 REINFORCED CONCRETE DI carried by the reinforcement, Its also assumed that plane sections ofa structural ‘member remain plan after staining so that across the section there must be linear distribution of strains. Figure 4.3 shows the cross-section of « member subjected to bending, and the resultant strain diagram, together with three different types of stress distribution in the concrete. IGN (1) The triangular stress distribution applies when the streses are very realy proportional tothe strains, which generally occurs atthe losding levels encountered under working conditions an I, therefore. used at the serviceability limit tae (2) The rectangolar-parabotiestres block represents the distebution at failure when the compressive strains are within the plastic range and i ssasociated with the design forthe ultimate limit state (@) The equivalent rectanglarstess block i simplified alternative tothe sectangular-perabolic distribution As there is compatibility of strains between the reinforcement end the adjacent ‘concrete, the steel strains éy in tension and és. in compression can be determined from the strain diagram. The relationship between the depth of neutral axis (2) and the maximum concrete strain (éc.) and the steel strains is given by 42) wld ” where dis the effective depth of the beem and d’ is the depth of the compression reinforcement. Having determined the strsins, we can evaluate the stresses in the reinforcement from the stressstrain curve of figure 4.2, together with the equations developed in section 4.1.2, For analysis of a section with known steo strains, the depth of the neutral axis cam be determined by rearranging equstion 4.2 as a and x (a4) 1+ At the ultimate limit state the maximum compressive strain in the concrete s taken a5 0.0035, For steel with ino equation 44 160 N/mm? the yleld strain is 0.002. Inserting these values ANALYSIS OF THE SECTION 7 0.0035 Hence, to ensure yilding of the tension steel at the ultimate limit state: x >0636d ‘At the ultimate limit state it Is important that member sections in flexure should be ductile aad that failure should occur with the gradual yielding of the tension steel and not by a sudden catastrophic compression failure of the concrete, ‘Also, yielding of the reinforcement enables the formation of plastic hinges so that redistribution of maximum moments can occur, resulting ina safer and more economical structure. To be very certain of the tension steel yielding, the code of practice limits the depth of neutral exis so thst xP Gy — 08d where | -Moment at the section after redistribution ‘moment at the section before redistribution Thus with moment redistribution not greater than 10 per cent,and py, *0.9, xPOSd ‘This limit will normally be adopted for ultimate limit state design, but larger degrees of moment redistribution will require a smaller limit tox to ensure that plastic hinges can form, providing adequate rotation at the critical sections (see section 4.7 and table 41), 4.3 Bending and the Equivalent Rectangular Stress Block For the design of most reinforced concrete structures i is usual to commence the design for the conditions at the ultimate limit state, which is then followed by checks to ensure that the structure is adequate for the serviceability limit state without excessive cellection or cracking of the conerete. For this reason the analysis in this chapter will first consider the simplified rectangular stress block which can be used for the design at the ultimate limit stst. ‘The rectangular stress block as shown in figure 4.4 may be used in preference ‘to the more rigoroxs rectanguler-paraboli stress block. This simplified stress ‘distribution will faellitate che snalysis and provide more manageable design equations. in particular when dealing with non-rectangular cross-sections. It can be seen from the figure that the stress block does not extend to the ‘neutral axis of the section but has « depth s = 0.9 x. This will result in the centroid of the stress lock being s/2 = 0.45 x from the top edge of the section, which is very nearly the same location as for the more precise rectangular-parabolic stress block: also the arezs of the two types of stress block are approximately equal (see section 4.9), Thus the moment of resistance of the section will be similar using, calculations based an either of the two stress blocks 58 REINFORCED CONCRETE DESIGN 0.0035 067 fey ‘ . 203! Pine 9-5 fy aa T J [7 0s] [e } Je ne Sa 4 rha l ote pa) e Fa Section Strains Stress Block Figure 48 Singly reinforced section with rectangular stress block ‘The design equations derived in sections 4.4 to 4.6 ace for redisteibution of ‘moments being not greater than 10 per cent, When a greater moment redistribue tion is applied. reference should be made to seetion 4.7 which describes how to modify the design equstions. 44 Singly Reinforced Rectangular Section in Bending Bending of the section will induce a resultant tensile force Py Sn the reinforcing steel. and g resultant compressive force in the concrete Fg. which acts through the centroid of the effective arca of concrete in compression, as shown in figure 4a, For equilibrium. the ultimate design moment, Af must be balanced by the nioment of resistance of the section so that M= Fog X= Fy 45) ‘where zis the lever arm between the resultant forces Fee and Fy. Fg = stress x area of action 0.45 fou x bs = 32 68) ‘So that substituting in equation 4.5 M= OAS fay bs x2 and replacing s from equation 4.6 M=0.9 foyb (d~2)2 an Rearcanging and substituting = Afb? fy (fa? - Gay +Kjo9=0 Solving this quadratic equation: [05+ yi0.25 — xj0.9) 48)" and ANALYSIS OF THE SECTION 39 ‘which is the equation in the code of practice BS 8110 for the lever arm, 2, of a singly reinforeed section, In equation 4.5 F, fylTm Ag With Yyy = LIS O87 fyAy Honce i oy O87 hx Equations 4.8 and 4.9 can be used to design the area of tension reinforcement in 2 concrete section to resist an ultimate moment, M. 0-95 Compression reinforcement 0-85 ai requred ‘ ‘ » -t0| — t AS nom 2m \ ar 0-08 O10 015 0-156 Kem rod toy The % values on the K oxi mark the limits for singly reinfercea sections mith mament redistribution applied (see Section 4.7) Figue ss Lerevarm curve Equation 4.8 for the lever arm z can be used to set up 2 table and draw a lever- ‘arm curve as shown in figure 4.5, and the curve may be used to determine the lever «um, z, instead of solving equation 4.8. 0 REINFORCED CONCRETE DESIGN ‘The upper limit of the leverazm curve, z = 0.9, is specified by BS 8110, The lower limit of 2 = 0,775 d is when the depth of neutral axis x =d/2, which is the ‘maxiraum value allowed by the code for a singly reinforced seetian in order 10 provide » ductile section which will have a gradual tension type failure as already described. With 2 = 0.775 d from equation 4.7) M= 09 foyb (0.775 d) x 0.775 d M0156 fou ba? lor 18 marked on the lever-arm diagram. The coefficient 0.156 has actually been calculated using the concrete stress as more precisely equal t0 0.67 fay! = 0.447 fey. instead of O45 fog ‘When’ Mu BP hes compression reinforcement is lso required to supplement the moment of resist: ance of the concrete =K>0156 Example 4.1 Design of a Singly Reinforced Rectangular Section The ultimate design moment 10 be sisted by the section in figure 46 is 185 KN m, Determine the area of tension reinforcement (4,) required given the characteristic nnateralstengths are fy = 460 Nfmm® and fey = 30 Nimm? 2260, any 440 ke bd hoy 185 x 108 therefore compression steel is not required. [ANALYSIS OF THE SECTION 6 fos+ /(ors-- S)t wo fosr (025-22) = 369 mm (Oralternatively, the value of z = fd could be obtained from the lever-arm siagram, figure 4.5.) Lever arm: uM O87 hz 185 x 108 G87 5 460 x39 = 1253 mm? A Analysis Equations for a Singly Reinforced Section ‘The following equations may be used to calculate the moment of resistance of a ghven suction with a known area of steel reinforcement . For equilibrium of the compressive force in the concrete and the tensile force in the steel in figuie 4. Foo Fa 045 foyb %3= O87 Sy Ay Therefore depth of stress block is DBT FyAs (ny 045 fu and x=909 Therefore moment of rexstanos ofthe section is Mi Fg xt =08T fA, (922) =08T HAL («- 2824) a1) 09 fab ‘These equations assume the tension reinforcement has yielded, which will be ‘the case if x 0,636 d. If this isnot the cese, the problem would require solving by trying successive values of x until ro REINFORCED CONCRETE DESIGN Foo = Fox With the steel strains and hence sitesses being determined from equations 4.2 and 4.1, to be used in equation 4.12 instead of 0.87 f,. Example 4.2 Analysis of Singly Reinforced Rectangular Section in Bending Determine the ultimate moment of resistance of the crost-seetion shows in figure 4.7 given that the characteristic strengths are f, = 460 N/mm? for the reinforce- ment and fo, 30 N/mm? for the cancrete. sst09 ost 2208, nse T Fc a lL [AF | f L lees a 4 z Figure 4.7 Analysts example ~ sgl reinforced section For equilibrium of the compressive and tensile forces on the section Feo = Fox therefore 0.45 fxyb8 = O87 fyAy 0.45 x 30 x 300 x5 = 0.87 x 460 x 1470 therefore 22145 mm and x= 910.9 = 145/0.9 = 161 mm ‘This value of x is less than the value of 0.636 a derived from section 4.2, and therefore the stecl has yielded and fax = 0.87 fy as assumed. Moment of resistance of the soction is Ma Fg xz = 0.87 fA, (4-9/2) 0.87 x 460 x 1470 (520 — 14572) x 10-* =263 KN m ANALYSIS OF THE SECTION 03 the Ultimate Limit 4.$ Rectangular Section with Compression Reinforcement. State a) Derivation of Basic Equations It should be noted that the equations in this saetion have been derived for the case ‘where the reduction in moment ata section due to moment redistribution is not greater than 10 per cent, When this is not the cast, reference should be made to section 4.7 which deals with the effect of moment redistribution, a8 hey Ss => a bk section Steams Stress Block Figure4.8. Section wich compression rfnforcenient From the section dealing with the analysis of a singly reinforced section when M>0.156 fey bd® the design ultimate moment exceeds the mement of resistance of the concrete and therefore compression reinforcement is required. For this condition the depth of neutral axis, x3 0.5 d, the maximum value allowed by the code in order to ensure 1 tension failure with a ductile section, Therefore zad—s/2 0.9 .x/2 09x05 al2 ms¢ For equiltrium of the section in Figure 4.8, Fa Feo Fac so that withthe reinforcement at yield 087 fyAy= 0.45 foybs # 087 HyAi s=09xd/2=0.45 d 087 fy = 0.201 foybd #087 fy (4.13) and taking moments about the centroid of the tension steel, Ay 6 REINFORCED CONCRETE DESIGN M= Fee X24 Pu (d-@) 0.201 faybd x 0.175 d+ 0.87 fA, (da) 0156 foubd® + 0.87 fyAi (dd) 4.14) From equation 4.14 M0156 fegbd? 0.87 f, @ a) Multiplying both sides of equation 4.13 by 2 = 0.775 d end rearranging gives = 2.156 foybd? , . 0874, x2 4 erg (41s) with 2 0.775 @ Hence te areas of compression steal, Aj, nd tension tel 4, con be ealeuated from equations 415 and 3.16, ‘Substituting A’ = 0.156 and K = Mfbd?f., into these equations thers into the same forms asin the code of practice, BS 8110, whi Ape RAK Vesta? uld convert Mee (ayy K foubl™ 4s . oar: 7“ @usy In this analysis it has been assumed that the compression steel has yielded so that the steel stress f, = 0.87 J. From the proportions of the strain distribution diagram: = 20088 (4.19) so that x At yield with f, = 460 Nimm? the steel strain yg = ¢, * 0.002, Therefore for yielding of the compression steel fy — 20 s045 (4.20)¢ x ”)~ poo3s 1s specified in the code, or with x =d/2 ANALYSIS OF THE SECTION 6s a a ‘The ratio of dd for the yielding of other grades of steel can be determined by using their yield strain in equation 4.19, but for values of fy less than 460 Nimm* the application of equation 4.2 will provide an adequate safe check. dd > 0.215, thon itis necessary fo calculate the sirain ég from equation 4.19 and then determine fg from Fac * Ex Xb, * 200000 bye ‘This valu of stress forthe compressive stel must then be used inthe denominator ‘of equation 415 in place of 0.87 fy in order to calculate the area 4, of compres- sion Steel, The ares of tension steel is calculated from a modified equation 4,16 such that 0215 (421) OU56faybd® , 4s y See A iS ae 4 oaR (6) Deen Charts FO | tees 0, ys 480, adr ero +08 a6 os * e a8 oO : os 0 18 20 25 30 35 100 Ag vod Figure4.9. Typical design char for dowdy rebaforeed beams ‘The equations forthe design charts are obtoined by taking moments about the neutral xis. Thos M= OAS fey 0.9.x ( — 0.9812) + eS Ot d') + fas 4-8) ‘This equation and 4.13 may be written in the form 66 REINFORCED CONCRETE DESIGN Ay A, * fy At = 0.201 fin © the fs feu the 01 fa 5 (1 0.45)+ fe a ¢) tat ( 5) oe aod For specified ratios of Aj/bd, x/d and d'/d, the two non-dimensional equations cen be solved to give values for Ag/éd and Af/d? so that a set of design charts such as che one shown in figure 4.9 may be plotted. Before the equations ean be solved, the steel stresses fx: and fig must be ealeulated for each value of 2/d. This is achieved by first determining the relevant strains from the strain dagram (or by applying equations 42 and 4.3) and then by evaluating the stresses ftom the stss- strsin curve of figure 4.2. Values of xfd below 0.3 apply when moments are redistributed Example 4.3 Design of a Reerangular Section with Compression Reinforcement (Moment Redistribution Factor i, 2 0.9) “The section showin in figure 4.10 sto resist an ultimate design moment of 285 KN m, The characteristic material stengths are f, » 480 Nimm? and fou =30 Noun? Determine the areas of reinforcement required M bah 2 285 x 108 260 x 440 130 >0.156 therefore compression steal is required afd = 50/440 =0.11 <0.2 asin equation 4.21 and the compression steel will have yielded. ‘Compression steel: = 0.189 Aya EEK fogba® O87f, da") = (0.189 - 0.156) 30 x 087 x 460 (440 =319 10m Tension stot: 2 0.156 30) 0.87 460 (0.775 x 440) = 1726 +319 = 2045 mm? 4319 ANALYSIS OF THE SECTION 67 Jouso paris) A t fete Figute 4.10. Design example wth compression retorcement, 5p > 0.9 Example 4.4 Analysis of a Doubly Reinforced Rectangular Section Determine the ultimate moment of resistance of the cross-section shown in figure 4.11 given that the characteristie strengths aze fy = 460 Nimm? for the einforce rent and fay = 30 N/mm? for the concrete -45te4 289, far + Ty terse re Sand s-092| a-510 oom L bye2ne fe Section Stress Block Figure 4.11 Analysts exampie, doubly reinforced section For equilibrium of the tensile and compressive forees on the section’ Fas Foot Fee ‘Assuring intially thatthe steel stresses fg and fi are the design yield valves, then 087 fy Ag = 0.45 fey bs + 0.87 fy AS Therefore = 0874, Uy = A2) 045 feud = 287 x 460241 0.45 «30 x 280 = 189 mm 628) 68 REINFORCED CONCRETE DESIGN x= 5/0.9= 210m, xf = 210/510 = 0.41 <0.636 so the tension steel will have yielded, Also d'fx=S0/210= 0.24 <0.43 s0 the compression steel will also have yielded, as assumed, ‘Taking moments about the tension steel M= Food ~ 512) + Fg @—a') = OAS fou bs (d — 5/2) + 0.87 fy Agta — a") = 0.45 x 30 x 280 x 189 (510 — 189/2) + 0.87 x 460 x 620 (510 — 50) = 412x108 N mm Itthe depth of neutral axis was such that the compressive or tensile steel had not yielded. it would have been necessary to try successive values of x until Fut Foot Fi balances, with the steel strains and stresses being caloulated from equations 4.2, 4.3 and 4). The steel stresees at balance would then be used to calculate the ‘moment of resistance, 4.6 Flanged Section in Bending at the Ultimate Limit State —"—. 24 r A ss00x nf [oeeror one | of OT | a - Fat 1 Figuro 4.12 scion, eres Mock within the flonge, $< ig ‘T-sections and L-scetions which have their flanges in compression ean both be designed or analysed in z similar manner, and the equations which are derived can ‘be applied to either type of cross-section. As the flanges generally provide a large ‘compressive ares, itis usually unnecessary to consider the case where compression. steel is required; if t should be required, the design would be based on the principles derived in section 4.6.3 ANALYSIS OF THE SECTION 9 For the singly reinforced section itis necessary to consider two conditions: (2) the stress block lies within the compression flange, and (2) the stress block extends below the flange. 4.6.1 Flanged Section — the Depth of the Stress Block Lies Within the Flange, s iy then the procedure would then be similar to that in example 47. 4.6.2 Flanged Section ~ the Depth of the Stress Block Extends Below the Flange, s> he FFor the design of a flanged section, the procedure described in section 4.6.1 will check if the depth of the stress block extends below the flange. An alternative procedure is to calculste the moment of resistance, Mr, of the section with = hs, the depth of the flange (see equation 4.22 of example 4.6 following). Hence if the sign moment, Af, is such that M>My then the stress block must extend below the flange, and shy In this case the design can be cartied out by either: (@) using an exact method to determine the depth of the neutral axis, es in ‘example 4.6 or ANALYSIS OF THE SECTION a (b) designing for the conservative condition of x = d/2 as described at the end of this section, Example 4.6 Design of a Flanged Section with the Depth of the Stress Block ‘Below the Flange ‘The T-section beam shown in figure 4.14 is requited to cesist an ultimate di moment of 180 KN. The characteristic msterial strengths are f, = 460 Nimm?* an fey = 30N/mm®, Calculate the area of reinforcement required. x= 400 ast Tb 2380] a Fa ee Section sie a ee ‘ite fnwe Fer is the force developed in the Mange Frey is the force developed in the area of web in compression Moment of resistance. My. of the flange is My Fag 2, 145 fase he (d — de?) 420)" 45 x 30 x 400 x 100 (350 — 10/2) x 10°F 162 KN m <180 KN m, the design moment Therefor, the stress block must extend below the Mange. This now necesury to determine the depth, ty, ofthe web in compression, where ye =~ Ie For equlibrium Applied moment 18O= Fag 21 + Fon X2 = 162 +085 faded n REINFORCED CONCRETE DESIGN = 162 + O45 x 30% 200 Sy (250 ~sw/2) x 10- 162 +2700 se (250 — 52) x 10-* ‘This equation can be rearranged into 8 = $005 + 13.33 4 10° Solving this quadratic equation fq = 28. mm So that the depth of neutral axis 100 + 280.9 = 142 mm Asx Fee so that > he ANALYSIS OF THE SECTION B by = 450 O45 fey “ nee + fa T2190 oe am ee dad | ural geaao) PP de oe [as2ai0 J 942300 stress section ue Figure 4.18 Analyse example ofa Tscrfon. s> hy and the force in the web is Fray 20.45 Fos bue (= hi) = 0.45 x 30 x 300 (6 — 150) x 10% = 4.05 (5 ~ 150) For equilibrium Fou = Fa ~Fet or 4.05 (5 ~ 150) = 964,5 — 911.2 Hence $= 163 mm 9/09 = 181 mm With this depth of newteal axis the reinforcement has yielded, as assumed, and Fegy = 4.05 (163 — 150) = 53 kN (If Fog > Fp, then the stress block would not extend beyond the flange and the Section would be analysed as in example 4.2 for a rectangular section of dimensions be xd) Taking moments about the centroid of the reinforcement Me Fag (A — g!2) + Foy (d~ 8) —hng2) = [911.2 (440 — 150/2) + $3 (440 — 163/2 ~ 1502)] x 10-* = 348 KNm Example 4.8 Design of a Flanged Section with Depth of Neutral Axis x=d/2 A safe but conservative design for a flanged section with s > iy can be achieved by setting the depth of neutral axis to x =d/2, the maximum depth allowed in the code, Design equations can be derived for this condition as follows. 4 REINFORCED CONCRETE DESIGN heal Section Stress Blocie Figure 4.16. Alnged secon with depth of neutrat axis x ~ 4/2 Depth of stress block, s= 0.9.x #045 ¢ Divide the flanged section within the depth of the stress block into areas 1 and 2 asshown in figure 4.16, so that bye XS= O45 bud Bg — bee) xe and the compression forces developed by these areas are Fay = O45 fog 04S ed = 0.2 fab Fea = OAS foyhs (br — bw) Taking moments about Fez at the centroid of the flange M = Fag Cd — higf2)~ Fey (912 — Ag!) = 087 fyAa (d — hel2) — 0.2 fasbyd (O45 d ~ hi? Area Ara Therefore MAO foybud (O45 d hg) 087 fy (d— 05h) ‘This i the equation given in clause 3.4.4.5 of BS 8100, It should not be used when ig 2 0.48 a. “Applying this equation to example 4.6 Ag~ 80X10 + 0.1 x 30x 200 x 350 0.45 4350 — 100) 10,87 x 460 (350 — 100/2) = 1600 mm? (compare with 1540 mm? of example 4.6) Before using equation 4.23 for calculating ds. itis necessary to confirm that compression reinforsement is not requited, This is achieved by using equation 4:24Ho check tha the moment of resunce Of the eonerste A, Wrest than the design moment, iene 423y ANALYSIS OF THE SECTION 7 4.6.3 Flanged Section with Compression Reinforcement With x = d/2in figure 4.16 and taking moments about dy, the maximum resistance rmoment of the concrete is Me = Fey X21 # Fea X22 0.156 feyby id? +085 fey (Oy bu) (d= hesf2) 4.24) (ote that the value of 0,156 was derived previously forthe rectangular section.) Dividing through by feuded” A zo.1s6 & +0. wh Fob? be a ‘which f similar fo the equation given in BS 8110. Tf the applied design moment, M > Af, compression reinforcement is required. Tnwhich ease the areas of steel can be calculated from by a9 *) (42sy 1 MM, js fe (4.26 0877, @-a) ae and considering the equilibrium of forces onthe section Fu=FatFat Pe so that y= OR ead +043 fase (r= Be) 5 gt an 08th Again, d’x > 0.43, othornise the design compressive steel stress sles than 087 f,. ‘When, because of moment redistribution, fy <0.9 the limiting depth of neutral axis is less than 2 and these equations wil require modification using the factors given in the table 4,1 of section 4.7 whicl. deals with moment redistribution. 4.7 Moment Redistribution and the Design Equstions ‘The plastic behaviour of reinforeed concte' at theultimate limit state affects the distebution of moments ina structure. To allow for this, the moments derived from an elastic analysis may be redistributed based on the assumption that plastic hinges have formed a the sections with the largest moments. The formation of plstic hinges requires olatively larg rotations with yielding ofthe teasion rein. forcement, To ensure large strains in the tension set, the code of practice restricts the depth ofthe neutral axis of a section azcording to the reduction of the elastic moment so that xP Gy —04)d (4.28)" ‘whore d is the effective depth 76 REINFORCED CONCRETE DESIGN ane fg = momenta setion after redistribution a i. moment at section before redistribution $0, forthe design of «section with compression senforcement after moment disiibutio the depth of neutral aie Wl take the muximom sae Eee equation 428 “herafoe the depth ofthe stress bck s 9 By — O44 ‘and the level arm is ~ 0.985 0.4) a2 (429) ‘The moment of resistance of the concrete in compression is Me™ Fog x2 #045 fobs x2 $045 fyb x09 Oy — 04) Ax [d— 0.9 (8 —0.4) a2] Therefore bakfjg “O95 ¥09 By — 04) [1 ~ 0.45 ~0.4)] = 0.402 (By — 0.4) — 0.18 (By — 0.4)? So that earrangiag Me = K'bd? fay where = 0.402 (f — 0.4) - 0.18 (@ — 0.4)? (430)" ‘This is the equation for A’ given in BS 8110. (should be noted that in calculating the coeffiients 0.402 and 0.18, the more Brake value of conrete ses fx = 0.67 ol as bee wed and not the vale ‘When theultimate design moment is such that M> K'bd? fey or K>K then compression sea is required such that aye KK feud? 087K dd’) (aan and = Kfesta? ys 4 ostg: *4 (432) ANALYSIS OF THE SECTION 7” u - 439" K BM he (4.33) ‘These equations are enti! in form :o those derived previously for he desin of 4 section with compcesin reinforcement fr By 0.9 “Table 1 shows the various design factors associated with he moment sedis tribution, If the value of d’fd for the section exceeds that shown in the table, the erapresion tel wil ot ave yslded and the compressive stress wil be less than OTF Unsuch cases. the compressive stress fc Wl be Eye where the stain ces otal from the proportions ofthe sain diagram. This value of fj should feplace O87 f, inequation 4.31, and equation 432 becomes Mabel? Kifoubd? 5 4: Se OSsTsye * O87 fy where ‘Table 4.1 Moment redistribution design factors I Redistribution By xd aid aie (per cent) ewe <10 09 os O77s 0.160.215 15 oss o4s 0797 Oa 0.193 20 08 4 082 0132 0.172 25 7s 03508420199 0.150 30 07 03 0865 ois 0129 ee It should be noted that for e singly reinforced section (K K" therefore compression stee! is required. Compression steel: Ae KEK Vey bd? O87 fda’) (0.151 ~ 0,132) 30 x 260 x 440? 0.87 x 460 (440 — 50) = 184mm? (CThe vasiation with the previous result is due to rounding-off errors in the arith metic and the subtraction of two numbers of similar magnitude in the numerator.) Tension steel: 95 + v0.25 -«'09)) (05+ ¥(0.25 - 0.13309) = 0.824 K fosbil? yy o8ry: 0.32% 30 x 260 x 4402 0.87 460% 082 x 0 = 138) + 184= 1565 mm* +184 4.8 Bending Plus Axial Load at the Ultimate Limit State ‘The applied axial force may be tonsite or compressive. In the analysis that follows, ‘compressive force is considered. For a tesile load the same basic principles of cquilibriam, compatibility of strains. and stress-strain relationships, would apply. but it would be necessary to change the sign of the applied load (NW) when we consider the equilibrium of forees on the cross-section, (The area of conerete in compression has not been reduced to allow for the concrete displaced by the compression steel, This could be taken into account by reducing the stress fye the compression steel by an amount equal © 0.45 fay.) Figure 4.18 represents the cross-section, of a member with typical strain and stress distributions for varying positions of the neutral axis, The cross-section is subject 10 a moment M and an axial compressive force WV. and in the figure the direction of the moment is such as to cause compression on ihe upper purl of the section and tension on the lower part. REINFORCED CONCRETE DESIGN 0-0035 O45 fy pt eile re bee fe | oe a onl |e il ie oe | Yi neuteat_* | . a tS 7 Section ake fo) tO 0.0035 Og me a | } 7 > Fe se | If LL. a Strains Stresses Moh Zen Figure 4.18 Bending plus exit fad with verstig postions ofthe neutna ast so ze ee oS Mon mm Figure $19. 7yplcel coum design chart ANALYSIS OF THE SECTION 81 Lat Fee be the vompressve fore developed inthe conctete and acting through the centroid of the stress block Fee be the compressive force inthe rinforeement acea A and acting through its centroid F,_be the tensile or comprossive fore inthe reinforcement area Ay and acting through its centroid {i) Basie Equations nat Design Charts “The applied foree (WV) must be balanced by the forces developed within the cross section, therefore N= Poet Pet Fy In this equation, F, will be negative whenever the postion of the neutral axis is such thatthe reinforcement isin tension, asin figure 418a, Substituting into this equation the terms for the stresses and areas = 045 fiu8 + fers + Sey G35 where fc fs the compressive stress in reinexeement 4s and fi the tenile or compressive stress in reinforcement As ‘The design moment M must be balanced by the moment of resistance of the forces developed within the cross-section. Hence, taking moments sbout the ‘mid-depth of the section a 7 sevoasgme(" 2) oraat(* a) fils ( *) (436) When the depth of neutral axis is such that 0. x 3 fe asin part (b) of figure 4.18, then the whole constete section is subject :0 a uniform compressive stress of (0.45 feu. in this ease, the concrete provides no contribution to the moment of resistance and the first term on the right sie of equation 4.36 disappears | Fora symmetrical arangement of veinforcement (4; = 4, = Aye/? and =f: ~ d), equations 4.35 and 4.36 can be rewritten in the following form N 045 fous hk M 045 feas (9.5 : fads (a _ bh? nh ar) th Mh 1 (8 03) ap + As fei “kG asp 82 REINFORCED CONCRETE DESIGN In these equations the steel strains, and hence the stresses fig and f,, vary with the depth of the neutral axis (x). Thus N/bh and M/bi can be calculated for specified ratios of 4,/oh and x/h 30 that column design charts for a symmetrical arrange- ‘ment of zeinforcement such as the one shown in figure 4.19 can be plotted, ‘The direct solution of equations 4.37 and 4.38 for the design of column rein: forcement would be very tedious and, therefore, a set of design charts for the ‘usual case of symmetrical sections have been prepared by the British Standards Institution. Examples showing the design of columa steel aze given in chapter 9. (ii) Modes of Failure ‘The relative magnitude of the moment (M) end the axial lood (W) governs whether the section will fil in tension or in compression. With lage effective eecentrcity (e= MIN) a tensile failure is bkely, but With a small eccentricity 2 compressive failure is more likely. The magnitude ofthe eccenricty affects the position of the neutral axis end hence the strains and stresses in the reinforcement. Let ae be the compressive strain in reinforcement 4 be the tensile or compressive strain in reinforcement Ay be the tensile yield strain of steel as shown in the stress-strain curve of figure 4.2. From the lear sun dtution of gure 4.18 eq: = 0.0035 &) and (439) wows (69 ‘The stee! stresses and strains are then related aveording to the stressstrain curve of Figure 4.2. Consider the following modes of failure of the section as shown on the inter action diagram of Figure 4.20. (2) Tension Failure, ¢, > ‘This type of failure is associated with large eccentricities (e) and small depths of neutral axis (x), Failute begins with yielding of the tensile reinforcement, followed by crushing of the concrete as the tensile strains rapidly increase (b) Balanced Failure, e, = ey. point b on figure 4.20 When failure occurs with yielding of the tension steel and crushing of the concrete at the same instant i is described asa “balanced” failure, With ¢, = ¢, and from equation 4.39 a apg = 1+ fe 0035 ANALYSIS OF THE SECTION 8 For example, substituting the values of e, = 0.002 for grade 460 steel put 70.6364 Equations 4.35 and 4,36 become Neat = Foc + Fee ~ Fs and =O45 fyb 0.9 Xpat + fue; — O87 frAy 4.40) an Fa(4— 222m) oe (8—2) vA, (0-2) where fie = $087 fy ‘At point b on the interaction diagram of figuee 4.20, 4/= Nygy M= Moar and J,= — 0.87 fy. Whon the design load N> Nix the sbetion val fal in compression, whist if Ny there will bean initial tone fllure, with yielding of reinoree- ment Ay. comprassion 2 fonture at Tension failure o ™ THEE Figure 4.20 Bending, plus anit load chert with: modes of joture (©) Compression Faure {mn this caso x > xq) and > Npay ‘The change in slope at point rin figure 4.20 occurs when ete ang fom equation 439 92.0035 20.0035 — e,) 2.33 d’ for grade 460 steel. Point r will occur ir the tension failure zone of the interaction diagram if Xe Spa 84 REINFORCED CONCRETE DESIGN Whenxd F,S 087 fy and compressive When x = 2.33 d. then from equation 4.39 = 0.00: AL this stage, both layers of stee! will have yielded and there will be zero moment of resistance with a symmetrical section, so that Ns #045 {bh +0874, (4,44) Such MCN interaction diagrams can be constructed fr any shape of eros ¢etion ich has an ais of eymmoty bj appying tho bass equim and stain co patibility equations with the stress-strain elations, as Gemonstrated in the fellow: ing examples. These diagrams can be very useful for design purposes. y for grade 460 steel Example 4.10 MAN Interactive Diagram for an Unsymimetrical Section Construct the interaction diagram forthe section shown in figure 4.21 with fog = 30 N/mm? and f, = 460 N/mm’. The bending causes maximum compression ‘om the Face adjacent to the steel ates A 22350 at = re i -00 arse | f | e450] 380] se nautrol a,.g62. | -Teirel 798? | Tons q ae stean Section Diagram Figure 4.21 Son-symiotetical section Man interaction example For a symmetrical cross-section. taking moments about the centredine of the concrete section will give M = 0 with W= No and both areas of steel at the yield stress. This is no longer true for unsymmetrical steel areas as Fu, # FF at yield ‘therefore, theoretically, moments should be calculated about an axis referred to ANALYSIS OF THE SECTION 85 as the ‘plastic centroid’. The ultimate axial load No acting throug the plastic ‘centroid causes a uniform strain across the section with compression yielding of all the reinfereement, and thus there is zero moment of resistance, With uniform Ssuain the noutral-axis depth, x, is at infinity ‘The location of the plastic centroid is determined by taking moments of all the suressresultants about an arbitrary axis such as AA in figure 4.21 so that <= 2 EFalil2+ Fad’ + Fd) 00 Eee Fat 05 fey Age +087 fy, +087 fA, (O45 x 30 x 350 x 450? 2+ 0.87 x 460 (1610 x 60 + 982 x 390) 0.45 x 30 350 x 450 + 0,87 x 460 (1610 + 982) = 212 mm from AA ‘The fundamental equation for calculating points on the interaction diagram ‘with varying depths of neutral axis are (© Compatibility of stains (used in table 4.2, columns 2 and 3): Gan Gi) Stress-strain relations for the steel (table 4.2, columns 4 and 5): ee, 0.002, F2O87 fy (442) ecg PsExe (4 Equilibrium (tbe 4.2, columns 6 and 7): N* Feet Fee +B OF O9XKH N= DAS feyb09E4 felt dy O9x> A N=04S fyb fel + Hedy Taking moments about the plastic centroid O9x0002 0 -087f, - 138 17 2334'=140 0.002 >0.002 0.87f, -087f, B47 257 sw206364 0002 c002 087% -087f, 1306 774 a= 390 >0002 0 a87f, 0 2303158 2334=909 >0002 >0002 0874 O87f 31640 Example 4.11 M-N Interaction Diagram for a Non-rectangular Section Construct te intersstion diagram forthe equilateral jangle columa seton in figure 4.23 with fa, = 30 Nimm? and fy = 460 N/mm®. The bending is about an ts parallel tothe side AA and causes craximum compression onthe corner Adjacent to the see aren A ANALYSIS OF THE SECTION 87 Figuce4.23. sorrectongular section MeN bnrersetion exemple For this triangular section, the plastic centroid is at the same location as the ‘geometric centeoig, since the moment of Fy, equals the moment of F, about this axis when all the bars have yielded in compression. ‘The fundamental equations for strain compatibility and the steels stress-strain relations are as presented in example 4.9 and are used again in this example. The equilibrium equations for the triangular section become N= Feet Feet F, o9xch N= O45 foysil2 + fe Ai fey o9x>h N=OA5 foyh 400/24 fies + Ay O9x0002 «0 0874, — 330 368 weet 0.016 9.002328 -087f, — 9 730 233a'=233 0.002 0.00095 O87, 189 354689 d=29 = >0002 0 087 f, 0 780 S81 233d=690 >0.002 >0002 087f, 087, 1523 0 4.9 The Rectangular-Parabolie Stress Block ‘A rectangular-parabolic stress block may be used to provide a more rigorous analysis of the reinforced concrete section. The stress block is similar in shape to the stress-strain curve for concrete in figure 4.1, having a maximum stress of OS fog at the ultimate strain of 0.0035, ANALYSIS OF THE SECTION 89 In figure 4.25 > = the coacrete strain at the end of the parabolic section w= the distance from the neutral axis to strain ¢5 x = depthof the neutral axis ky = the mean conerete stress K,x= depthto the centroid of the stress block, x 90035 045 Jeg Bentrors of tiene 7 fe section strome Stross Block Frigate 4.25 Section bn bending witha rcrongular-perabotte sress block (a) To Determine the Mean Concrete Stress, &y From the strain diagram 005 * therfore ee 003s Sbsttting for eo = 24x 10* Vifea fq) (88 gue 4.1) wn Mi With Y= 15 (4.43) For the sess lock fy = aa of set block Thus, using the ar [properties of a parabola as shown in figure 4.26, we have iy = DEB Fa — 045 fo. 0.15 Vfe (015-5) was areas: a= Fut, aye BE Position ef centrows: 9:20, opr 2 Figure 4.26. Properties ofa parabola (b) To Determine the Depth of the Centroid kx sky is determined for a rectangular section by taking area moments ofthe stress block about the neucral avis ~ seo figures 4.25 und 4.26. Thus _atea pars * x/2 ~ area ist x w/4 183 of stress block hax (O45 fay) x12 = (05 fog wi3) wl hax 0.45 foyle?/2-— w?12) hax Substituting for w from equation 4.43 Ge k= OAS Sear? [os fa] ‘ 383 si 048 hy [55 few . Slo [os és] easy Values of &, and &; for varying characteristic concrete strengths have been tabulaied in table 4.4 ‘Once we know the properties of the stress block, the magnitude and position of the resultant compressive force in the conerete ean be determined, and hence the moment of resisiance of the section caleulated using procedures similar to those for the rectangular stress block. ANALYSIS OF THE SECTION 1 Table 4.4 Values of ky and k for different concrete grades Fou Tr (vimm?) (N/mm?) ki Mew ky aa fkafow 20 209) ‘OaI2 0.460 DG 25 10.200 0.408 0.456 089s 30 12.120 0.404 0.452 0.804 40 15.875 0.397 0.4es 0.893 30 19.331 0391 0.439 0.890 2) 23.097 0385 0.434 0887 ‘Typical values oa 0.45 089 Using typical vues from table 4.4, a comparison of the rectangular-parabolic and the rectangular stress blocks provides, {(W)Stress resultant, Fe rectangular-parabolic: ky 6x 0.4 faybx rectangular: 0.45 fey x 0.9 Bx = 0.4 faybx (8) Lever arm,z rectangular putabolie: d— kyx =~ 0.45 rectangular: d ~ } x 0.9x =d ~ 0.4Sx So both stress blocks have almost the same moment of resistance, Fee x2, showing is adequate to use the simpler rectangular stress block for design calculations, 4.10 The Triangular Stress Block ‘The triangular stress block applies to elastic conditions during the serviceability limit state. In practice it is not generally used in design calculations except for liquid retaining structures, or for the calculations of crack widths and deflections as described in chapter 6, With the triangular stress block, the eross-section can be considered as (i) ctacked in the tension zene, or (i) unotacked with the concrete resisting a small smount of tension, 4.10.1 Cracked Section ‘A cracked section s shown in figure 4.27 witha stress resultant Fy acting through the centroid ofthe stel and Fe acting through the centroid of the wiangular sures block, For equilibrium of the section Fat Fa or OS bfee * Ash 4.46)" 2 REINFORCED CONCRETE DESIGN 4 € fee * ce fe Neutrol 5 jo fu Section Strain Stress Figuie$.27- Tlngulr stress Mock ~ cracked seetion and the moment of resistance Me Fog 2 Fig x2 or M05 dxfeg (@—x/3)= Aska (d 3/3) aan (1) Analysis ofa Specified Section ‘The dopti of the neutral axis, x, can be determined by converting the section into an ‘equivalent’ area of conerete as shown in figure 4.28. where ae = E/E, the ‘modular ratio. Taking area moments about the upper edge: xe 2) EA /Trensformed Steel Area = Figure 6.28 Egulvolon transformed tection with the concrete cracked Therefore = BERND + ayAad bx tasAy ANALYSIS OF THE SECTION 3 Lag “ 1 bx? +a Aye agi =0 Solving this quadratic equation gives sex Seda tea? 92 oat wasp Fqustion 4.48 may be solved using chart such asthe one showa i figure 4.29. Equations 446 to 4.48 con be used to analyse a specified reinforced concrete section gait 0302010 o3 Soe . on © 32 oa oe xia Figure 4.29. Neutrat-oxis depth for eroked rectangular secfons ~ clastic beiviour (1H) Design of Steet Area, A,, with Stresses fx and foe Specified ‘The depth of the neutral axis can also be expressed in terms of the strains snd stresses of the vonerete and steel From the linesr strain distribution of figure 4.27. xy _fe SeclEe 4 eet ee SeclBe + fulBe 4 REINFORCED CONCRETE DESIGN ‘Therefore (4asye Equations 4.47 and 4.49 may be used to design the area of tension steel required, at a specified stress, in order to resists given moment Example 4.12 Analysis of @ Cracked Section using a Trianguiar Stress Block For the section shown in figure 4.30, determine the concrete and steel stresses caused by a moment of 120 kN m, assuming a cracked section. Take y/Ee = b+ 300 n-$20 2128. t |e we 167 0mm? aa Figure 4.30. Analysis exmp with iingule stress biook a, As = 15x1470 * ed” 300% 460 Using the chart of figure 4.29 or equation 4.48 gives x = 197 mm. From equation 4.47 1 2 m=} oxhe (¢-2 pote (5 therefore 120. 108 = 1 20005 197 fs (100 1) therefore foo ® 10.3 Nimmn? From equation 4.45 SaAe™ 5 08 fee ANALYSIS OF THE SECTION 95 therefore 1 2 =300«197 x 123 x 1 2207 Nfenm’ £23004 197 x 82 x Te t 4.10.2 Triangular Stress Block ~ Uncracked Section ‘The concrete may be considered to resist a small amount of tension. In this ease @ tousile stress resultant F, acts through the centroid of the triangular stress block jn the tension zone as shown in figure 4.31 Moutea! gar ptley a Section ‘strom Stress Figure 431 Trianguersiress Hock — uncracked section For equilibrium of the section Feo Fert Fre 4.50) where Fog 0S bY fos Fy 2 0Sbth- xf and Fa 2 Aha Taking moments about Fee. the moment of resistance of the sectfon is given, by = Faq xld x13) + Fax (2x4 2 Gx M> Fa x(d— x13) + Fey (3 #y )) sry ‘The depth of the neutral axis, x. can be determined by taking area moments about the upper edge AA of the equivalent concrete section shown in figure 4,32, such that pa ECA) BA fy a = 7 termed the module rato 9% REINFORCED CONCRETE DESIGN Figure 4.32. Fqulvalont nonsformed seerion with the concrete neracked ‘Therefore DRxN2 + agAs xd Bit eA, = A oor (asa 29 dar where 7 = Aslbi From the linear propoitions of the strain diagram in figure 4.31: (453) Therefore as stress = E x strain: asaye Hence ifthe maximum tensile strain or stress is specified, i s possible to calculate the corresponding conerete compressive and steel tensile stresses from equations 434, ‘The equations derived can be used to analyse a given cross-section In order to determine the moment of resistance of the uncracked section, as for liquid- retaining structures. This is lustrsted further by examples in chapter 11 Example 4.12 Analysis of an Uncracked Section For the section shown in figure 4.30, calculate the serviceability, moment of resistance with no cracking ofthe concrete, given fey = 3 N/mi®, = 30 KN/mn? and = 200 KN)? ANALYSIS OF THE SECTION 7 1470 | i 300x520 fs 70 2 669 E30 At age rar = 5204.2 6.67 x 000094 x 4 D2 x 6.57 x 0.0094 2) sah [460 — 272) 6.67 x3 (520-273) 0.0094 15.2. Nfmm* fa (- 3) + Soha fax G 2x: 2 sas) 2170x152 (4s0 2) 10+ ! 3006620 a ee i x(2x2724 2620-272) 10 (3 220 x) = 83+38,7=47kNm 5 Shear, Bond and Torsion This chapter deals with the theory and derivation of the design equations for shear, bond and torsion. Some of the more practial factors goveraing the choice and arrangement of the reinforcement are dealt with in the chapters on member design, particularly chapter 7, which contains examples of the design and detailing of shear and torsion reinforcement in beams. Punching sheat caused by concentrated loads ‘on slabs is covered in section 8.2 of the chapter on slab design. Sil Shear Figuce 5.1 represents the distribution of principal stresses across the span of a homogeneous concrete beam. The direction of the principal compressive stresses ‘takes the form of an arch, while the tensile stresses have the curve of a catenary or suspended chain. Towards midspan, where the shear is low and the bending, stresses are dominant, the direction of the stresses tends to be parallel to the beam axis, Near the supports, where the shearing forces are greater, the prineipal stresses are inclined at a steeper angle, so that the tensile stresses are liable to eause ;gonal cracking. Ifthe diggonal tensfon exceeds the limited tensile strength of tie conerete then shear reinforcement must be provided, This reinforcement is either inthe form of (1) sticrups, or (2) inclined bars (used in conjunction with stizrups), oomeression ‘Diagonal tension igure 8.1. Principe sess ino beam: 98 SHEAR, BOND AND TORSION ~ ‘The shear in a seinforeed concrete bean without shear reinforcement is carried by a combination of three main components. These are (concrete in the compression zone Gi), doweling scion of tensile reinforcement ii) _searegate inteslock across flexural cracks. “The actual behavious is complex, and difTeult te analyse theoretically, but by applying the sesults from many experimental investigations, reasonable simplified procedures for analysis and design have been developed, S11 Stirups In order to derive simplified equations the action ofa reinforced concrete beatn in shear i epresented by an analogous truss in which the longitudinal reinforcement ous the bottom chord, the stirrups ae the vectical merabers and the concrete cts as the diagonal and top chord compression members as indicated in figure 5.2 In the truss show, the stimups are spaced ata distance equal to the effective depth (2) af the beam so that the diagonal concrete compression members are at an angle of 45°, which more or less ages with experimental observations ofthe cracking of reinforced concrete beams close to their supparts section Figure 8.2. Surraps end che analogous russ too REINFORCED CONCRETE DESIGN In the analogous truss, let Ag be the cross-sectional area of the two legs of the stirrup ‘yy be the characteristic strength of the stirrup reinforcement ‘Pe the shear force due to the ultimate loads, Using the method of seetions it ean be seen at section XX in the figure that at the ukimate limit state the foree in the vertical stirrup member must equal the shear force V, that is 087 fy ¥ 087 fp Aye = vd on where v= P/bd is the average shear stress on the section. ‘When the stirup spacing is less than the effective depth, a series of super- ‘imposed equivalent trusses may be considered, so that the force to be resisted by the stirrup Is reduced proportionally. Thus ify = the stizrup spacing, equation 5.1 becomes 087 fyyAgy = 38d (5) Aw Sy OBT Fy Since the concrete is also capable of resisting a imited amount of shear this ‘equation is rewritten as, Age _ SORT Sy where ve isthe ultimate shear stress that can be resisted by the concrete, Values of ve are given in table 5.1. It can be seen from the table that v- increases for shallow ‘members and those with larger percentages of tensile reinforcement. The longitu inal tension bars contribute to the shear resistence by their dowelling action ané they also help to prevent shear cracks from commencing at small tension cracks. To be effective, these tension bars should continue an effective depth, d, beyond the section, or ata support they should be adequately curtailed and anchored. Close to supports, sections have an enhanced shear resistance owing in part to the induced compressive stresses from the concentrated reaction and the steeper angle of the failure plane, which would normally occur at angle of 30° to the horizontal for an unreinforced section. Within a distance of 2d from a support or a concentrated load the design concrete shear stress y- may be increased to v,2dfa,. The distance a, is measured from the support or concentrated load to the section being designed. This enhancement is useful when designing beams with concentrated loads near to a support, or with corbels and pile caps ‘Asa simplified approach for beams carrying mainly uniformly distributed loads, the critical section for design may be taken ata distance d from the face of the ear SHEAR, BOND AND TORSION ol ‘Table 5.1 Value of ultimate shear stress v (N/mm?) for a concrete strength of fa, = 30 Nim? Effective depth (mm) 150175200 2s 250 300 400 O44 043 O41 0.40038 0,36 052 050 049 048 += 0460.82 066 «064 «06259057053 O75 072 0.70 089 064061 083° 080 0.78 «075072067 093 091 088 086 «0830.76, 104 1.01 «097085 Ost, M9 LIS LM 1.08104 0.97 For charagtersti stenaths other chan 30 Nfmm® the values in the table may be multiplied by (F25)"?1.06. Theva of fey should not be sreter than 40 Ninn support using the value of vg from table $.1 in equation §.2. The shear links required should then continue to the face of the suppor Large shearing forces are also liable to cause crushing of the conerete along the directions of the principal compressive stresses, and therefore atthe face of a Support the average shear stress should never exceed the lesser of 0.8 Wifey or SN)? ‘The areas end spacings of the stirups can be celeulated from equation $.2. Reacrangement of the equation gives the shearing resistance for a given stirrup size and spacing thus srorieersie(Arvostinsn)# 6 Farther information on the practealdeails end design examples are given in section 7.3 (Design for Shes). S12 Bentoup Bars To resist the shearing forces, bars may be bent up near the supports es shown in figure 5.3. The bent-up bars and the concrete in compression are considered t0 act 45 an analogous atic girder and the shear resistance of the basis determined by ‘aking a section XX through the giréer. From the geometry of part (a) of the figue, the spacing of the bentup basis Sp = (d — d') (cot a + cot 6) fang at the section XX the shear resistance of the single br is ¥=087 fy sin 6a) ‘here dgy it the crosssectionel aren ofthe bent bar woz REINFORCED CONCRETE DESIGN Anchorage, fe es { Aa SS, th NK PH ! Z x1 sg: ie ¢Vlcotavcotp} (Single System Multiple Systern Figure $3 Rentup bars For a multiple system of bentup bas, asin par (b) of the figure. the shear resistance is increased proportionately to the spacing, sy. Hence (4- a’) feot a +eot) (5) 5 ‘The angles and f should both be greater than or equal to 45° and the code requires that the spacing sy hos © maximum value of 1.Sd, With and sy" (d— a"), equation 5.5 becomes 1.23 fyyden 66), ‘and this arrangement is commonly referred to asa double system. 0.87 SyyAap sin & Example 5.1 Shear Resistance at a Section Determine the shear resistance of the beam shown in figure 5.4, which carries a uniformly distributed load. The characteristic strengths are: fyy = 250 Nimm? for the stirrups. fyy = 460 N/mm? for the bentup bars and fey = 30 N/mm? for the concrete, 1004, _ 100x982 bd 350x650 ‘Thus, from table $.1, ¥. = 0.5 N/mm? by interpolation. Crosssectional area of @ size 12 bar = 113 mm? ‘SHEAR, BOND AND TORSION 103 Pie sirruns ot 100 speq, ae ele t / \ Rie oh Yes — Dole system section 25 pant up bors 2125 deesSisaunn each Ay = 982sqmm Figure $4 Boars ith sirups and bent bars Thus, forthe stirups, Ap/5, = 2 x 113/100 = 2.26 The shear resistanes ofthe sticrups plus the concrete i given by equation 5.3 us ye Maver fd ened 6 O87 x 280 x 650+ 35005 x 650 (319 + 114) x 10? N=433 ¥ 10° N ‘The bent-up bars ate arcanged in a double system. Hence the shear resistance of the bentup bars is % 23 Fyway 23 x 400x491 78 x10 N ‘Total shear resistance of the stirrups, concrete and bent-up bars is therefore Ve Wy + My =(433 +278) 10 = T1110 N It should be noted that the shear resistance of 319 KN provided by the stirrups 's greater than the shear resistance of the bent-up bars, 278 KN. us required by BSs110. It should also be checked that at the face of the support V/bd does not exceed the lesser of 0.8 fey or § Nini? 5.2 Anchorage Bond ‘The reinforcing bar subject taditeet tension shown in figure S.$ must be firmly anchored if itis not to be pulled cut of the concrete, Bars subjected to forces induced by flexure must similarly be anchored to develop their design stresses. The 104 REINFORCED CONCRETE DESIGN ‘anchorage depends on the bond becween the bar and the conerete, and the atea of contact. Let L = minimum anchorage length to prevent pull out ® = bar size or nomine! diameter ou = ultimate anchorage bond stess Je = the direct tensile or compressive stress in the bar Figures. Anchorage bond ‘Considering the Forces on the bar ‘ensile pullout force = bar's cross seetional area x direct stzess ag? a anchorage force = contact area x anchorage bond stress =(Ln0) kg therefore Cr) fog Eg, + hence Late 4 fou and when f, = 0.87, the ultimate (esl or compressive stress, the anchorage gh is 087 f, on DBT 5.7)" 4 fw 6 ‘The design ultimate anchorage bond stress, fy. is obtained from the equation Sou =BVfew (58) ‘The coefficient f depends on the bar type ond whether the bar isin tension or compression. Values of 8 are given in cable 5.2. Equation 5.7 may be rewritten as anchorage length L = K,® SHEAR, BOND AND TORSION Vos Table 5.2 Value of bond coefficient 6 6 Bar type Bars in Bars in tension compression Plain bars 0.28 03s ‘Type 1: deformed bars 0.40 0.50 ‘Type 2: deformed bars 050 0.63 Fabric 06s 081 ‘Values of K', corresponding to the anchorage of tension and compression bars for various grades of concrete and reinforcing bars have been tabulated in the appendix. Anchorage may also be provided by hooks or bends inthe reinforcement; thelr anchorage values are indicated in figure 5,6, When a bent bat or hook is used, the beating stress on the inside of the bend should be checked as described in section 73.2 and example 7.8, Ja} Anchorage value = be but nat greater than 126 la (0) Anchorege value but not greater than 268 =r, 28, For milé steel bare minimum = 26 For high yiela bers minimum © = 38 or 48 for sizes 2imm and odove Figure 56 Anchorage vues for bands ond hook Example 5.2 Calculation of Axchorage Length Determine the length of tension anchorage required for the 25 mm diameter pla nl stetreinforeing bars in the cantilever of figuee 8.7. The characteristic material strengths are fey = 30 N/mm? and f, = 250 N/mm? The ulnar ‘anchorage bond stress, fy = 8-Vfey = 0.28/30 = 1.5 N/mm? (see table §.2) 106 REINFORCED CONCRETE DESIGN = open ]_~Ancnorage length & roy R26 bars yh Figure 8.7. Anchorage fore cantilever baam 0874, snchorage length L = 25741 Anchorage length = fo 2 087 x 250 28 = 36,2 28 4x15 therefore L=910mm 5.3 Laps in Reinforcement Lapping of reinforceaent is often accessory to transfer the forces from one bar 10 another. The rules for this are: (1) The logs should preferably be staggered and be away from sections with high stresses, (2) The evinimumn lap length should be not less than the greater of, 15@ o¢ 300 mm for bars 250 mm for fabric (3) Tension laps should be equal to at least the design tension anchorage length, but ix certain conditions this should be increased as shown in fAgure 5.8, according to the following rules. (a) At the top of a section and with minimum cover <2 snultiply by Lt (b) At comers whete minimum cover to either face <24 or clear spacing between adjacent laps < 75 mm or 64 imoltiply by 1.4 (©) Where both (a) and (b) apply multiply by 2.0 SHEAR, BOND AND TORSION Ww? ‘The concrete at the top of a member is generally less compacted and also tends to have a grester water content, resulting in a lower concrete strength. Also. at the corners of members there fs less confinement of the reinforcement. For these reasons longer lap lengths are required at these locations. (4) Compression laps should be at least 25 per cent greater than the com= pression anchorage length, (5) Lap Tengiiss for unequal size bars may be based on the smaller bar. ‘A table of mininmura lap lengths i included in the appendix. tata ——— 9) Reinforcement lap) lop langth « 2 ancrorage Hangin x10 Sx he 5) Increased lap lengths igure 8.8. Lapping ofrinforing bars 54 Analysis of Section Subject to Torsional Moments Torsional moments produce shear stresses which result in principal tensile stresses inclined at approximately 45° to the longitudinal axis of the member. Diagonal cracking occurs when these tensile stresses exceed the tensile strength of the con- crete, The cracks wil form a spiral around the member as in figure 5.9 ‘Reinforcement in the form of closed links and longitudinal bars will earry the forees from incressing torsional moment after cracking, by a truss action with reinforcement as tension members and concrete as compressive struts between Jinks, Failure will eventually occur by reinforcement yielding, coupled with crush- ing of the concrete along line AA as the cracks on the other faces open up. ‘tis assumed that once the torsional shear stress on a section exceeds the value ‘to cause cracking, tension reinforcement in the form of closed links must be provided to resist the full torsional moment, los REINFORCED CONCRETE DESIGN Figure 5.10. Porsonalrlnforcement Tenson fen tnk = 48° 087 jy moment of force F about centre Line for vertical leg and ea for horizontal leg, where Ag = cross-sectional ares of the two legs ofa link. The total torsional ‘moment provided by one closed link i, therefore, given by the sum of the moments due to each leg of the link about the centre line of the section, that is TaFAt x24 7% = x2 SHEAR, BOND AND TORSION too Where Links are provided at a distance s, apart, the torsional resistance of the system of irks is obtained by multiplying the moments due to each leg in the above expressions by the number of legs crossing each crack. This number is given by Sy for vertical legs and x, /5, for horizontal logs i itis assumed that ll eracksate approximately at 45°, The total torsional resistance then becomes Ae (0.87 fy.) 2 Hence iy (O87 fey) «0.8 ‘The efficiency factor of 0.8 is included to allow for errors in assumptions made about the truss behsviour. Hence closed links must be provided such that ‘Toensure the proper action of these links, longitudinal bars evenly distributed round the inside perimeter of the links must be provided. This reinforcement Which resists the longitudinal component of the diagonal tension fozces should be such that the total quantity is equal to the same volume as the steel in the links, suitably adjusted to allow for differing strengths, This is given by Ay fy Aga DY (x, ty, oy wines the dbase ytd sient of longitudinal rinforement The Caleulsederounts of torsional enforcement mas be powsed i ition ithe fl bonding and chat ettoranont cereals re elle oad Combination crtesyondng tothe torsional moment consdred. Whee longtdinal bending enforcement equed, the adda torsional tea a may exert pov by incense thy sae of ac pronto by aldnal bar & nembe Sich bcd fa Tocon pl andog or sbar cell ge Tobe ene Tanned "Thy eee GAG wisn Ionia Ode atta nk aaknxa SODA snd minanum of fou bars nut be used in each in All oso stel must aso sate datas ec eatequl pale east abe desos pat Te Sih pl eked te tbs nea enaca tat wo patie ces Hagatey preted The oe hance aa en an We tee OT methods BS 120 ncommonds a pasts analy sch tht fo a vectangult a isin’ (max ~ Pemin/3) he REINFORCED CONCRETE DESIGN ‘where Min i the smaller dimension of the section. mas is the larger dimension of the seetion, or v= <2 fors thin hollow seetion where Jy is the wall thickness and A isthe area enclosed by the centre-ine of the walls. If the sum of wall thicknesses of a hollow section exceeds one-quarter of the overall dimension, this should be treated as solid 'A section having a T-, L-or [shape should be divided into component rectangles to maximise the funetion E¢Fppint Hons). The torsion shear stress on each rectangle should then be calculated by considering the rectangle as carrying a torsional moment of po { Peis? Homa Elimin® Fier ‘Torsion reinforcement will be required if the torsional shear stress, exceeds the capacity of the concrete section. It has been found experimentally that this value related approximately to the square root of the characteristic concrete cube strength, and the limiting value recommended by BS 8110 is 0.057 Vey bat not more than 0.4 Nfmin# Torsion Combined with Bending and Shear Serese Torsion is seldom present slone, and in most practical situations will be combined with shear and bending stresses. (2) Shear Stresses Diagonal cracking starts on the side of a member where torsional and shear stresses are additive, The shear force has a negligible effect on ultimate torsional strength when V'< vd, the shear strength of the concrete section, bul once diagonal ‘racks form, the torsional stiffness is reduced considerably. To ensure that crushing of the concrete does not occur (figure 5.9) the sum of the shear and torsion stresses on a section should not be excessive so that (40) whore Yqy=O8Vfey oF SN am? Aditionslly in the ease of small sections where y is less than $S0:mm bey Ze ere 350 ‘must be satisfied to prevent spalling of the comers. ‘The recommendations for reinforcement 10 resist a combination of shear and torsion are given in table 7.3, SHEAR, BOND AND TORSION mi (8) Bending Stresses When a bending moment is present, diogonal cracks will usually develop from the {op of the flexural cracks. The flexural cracks theraselves only slightly reduce the torsional stffuess, provided that the diagonal cracks do not develop. The final mode of failuce will depend on the distribution and quantity of reinforcement present, Figure 5.11 shows typical ultimate moment and ultimate torsion interaction curve for a section. As can be seen, for moments up to approximately O8M, the section can also resist the full ultimate torsion 7... Hence no calculations for torsion are generally necessary for the ultimate limit state of reinforced concrete ‘unless torsion has been included in the original analysis or is required for equilibrium. oem Figure 5.11 Combined bending and torsion 6 Serviceability, Durability and Stability Requirements ‘The concept of serviceability Limit states has been introduced in chapter 2, and for reinforced conerete structures these states are often satisfied by observing empirical rules which affect the detailing only, In some circumstances, however, t may be desired {0 estimate the behaviour of a member under working conditions, and ‘mathematical methods of estimating deformations and cracking must be used. The design of water-retaining structures, and prestressed concrete, are both ased primarily on the avoidance or limitation of cracking and these are considered Separately in other chapters, ‘Where the foundations ofa structure are in contact with the ground, the pres: sures developed will influence the amoun’ of settlement that is likely t6 aeeur. To ensure that these movements are limited to acceptable values and are similar throughout a structure, the sizes of foundations necessary are based on the service loads for the structure Durability is necessery to ensure that a stcucture remains serviceable through: coat its lifetime, This requirement will involve aspects af design, such as concrete ‘mix selection and determination of cover to reinforcing bars, as well as selection of suitable msterials for the exposuce conditions which ate expected. Good con: struction procedures including adequate curing are also essential if reinforced concrete is to be durable. ‘Simplified rules governing the seleetion of cover, member dimensions, and rein- forcement detailing are given in section 6.1 and 6,2, while more rigorous procedures for calculation of actus) deflections and crack widths are described in sections 6.3, to 6.5, Durability and fire resistance are discussed in section 6.6. ‘The stability of a structure under sccidental loadings, although an ultimate Himlt state analysis, will usually tte the form of a check to ensure that empirical rules designed to give a reasonab¥e minimum resistance against misuse ot accident are satisfied. Like serviceability checks, this will often merely involve detailing of reinforcement and not affect the total quantity provided. Stability requirements are described in section 6.7, ‘SERVICEABILITY, DURABILITY AND STABILITY REQUIREMENTS 113 6.1 Detailing Requirements ‘These are to ensure that a structure has satisfactory durability and serviceability, performance under normnal circurnstances. BS 8110 recommends simple rules con: ering the concrete mix and cover to reinforcement, minimum member dimen sions, and limits to reinforcement quantities and spacings which must be taken into aocount at the member sizing and reinforcement detailing stages. Reinforce: ‘meat detailing may also be affected by stability considerations as desecibed in section 6.7, as well as ules concerning anchorage and lapping of bars which have been discussed in sections 5.2 and 5.3 6.11 Minimum Concrete Mix and Cover {Exposure Conditions) ‘These requirements are interrelated, and BS 8110 specifies minimum combinations of thickness of cover and mix characteristics for various exposure conditions. The mixesare expressed in terms of minimum cement content, maximum water! cement ratio and corresponding minimum strength grade, These basic requirements are given in table 6.1 ‘The nominal cover is that to al steel, and allows for a maximum fixing toler- ance of + 5 mm, Adjustments must be made to cement contents if different aggregate sizes are used, and details of these and other posable modific givea In BS 8110, 6.1.2 Minimum Member Dimensions and Cover (Fire Resistance) BS 8 110 also provides tabulated values of minimum dimensions and nominal ‘covers for various types of concrete member which are necessary to permit the member to withstand fire for @ pacified period of time. These are summatised in tables 6.2 and 6.3. 6.1.3 Maximum Spacing of Reinforcement ‘The maximum clear spacings given in table 6.4 apply to bars in tension in heams when a maximum likely crack width of 0.3 mmis aeeptable and the cover te reinforcement does not exeeed 50 min It can be seen that the spacing is restricted according to the amount of moment ‘edistibution applied. Any bar of diameter less than 0.45 times that ofthe largest boar ina section must be ignored when applying these spacings. Bars adjacent 10 comers of beams must not be more than ore-alf ofthe clear distance given in table 6.4 from the comer. Rules for slabs permit greater spacings under specified conditions as follows {a) If < 200 mm with high yield steel (fy = 460 N/mm?) or (b) If -<250 mm with Mild steel (jy = 250.Nimm*) oF) E100 A fod © 03 per cent then the maximum clear spacing between bars should not exceed 750 mm o: 3d, whichever is smaller 1s REINFORCED CONCRETE DESIGN Table 6.1 Nominal cover and mix requirements for normal weight 20 mm ‘maximum size aggregate concrete Nominal cover to all reinforcement (mn) Environment classification Mild: for example, protected against ‘weather or aggressive conditions 23 2 2 2 Moderate: for example, sheltered from sovere rain and freezing while ‘wert; subject to condensation or ‘continuously under water; in contact ‘with non-aggressive soil, - 3 30 2820 Severe: for example, exposed 10 severe rain; alternate wetting and ‘drying; occasional freezing or severe condensation ~ = 493025 Very Severe: for example, exposed 10 sea water spray. de-icing sats, corrosive fumes or severe wet freezing © — = — = SO 40® 30 Extreme: for example, exposed to abrasive action (sea water and solids, flowing acid water, machinery or vehicles) - ~ oor 50 Maximum free water/cement ratio 065 060 055 050 04s Minimum cement content (kglm®) 275 300325350400 Lowest concrete grade GO G5 CHO CHS C50 "Epsrained ait requiced for wet feezing Table 6.2 Nominal cover for fire resistance Fire ‘Nominal cover to all reinforcement (mm) resistance (hours) Beams Floors Ribs —_ _- —— Columns SS com. SS. cont, $8, cont. 05 2 20 20 10 2 20 0 20 20 1s 2 2 % » 3 20 403035 Os a5" 35 30 Ce er) Sst ast 40 yos 50% 55% ase SS ‘Adulijonal measures necessry 19 reduce rik of paling SERVICEABILITY, DURABILITY AND STABILITY REQUIREMENTS 115 Table 6.3 Minimum dimensions of RC members for fire resistance (nominal cover requitements satistied) Minimum dimensions (mm) (ours) Exposed Wall thicknesses Beam Rib Floor column —-——_——_— with width thickness width 4 <0.4% > 10% 0s 200125 150 150 75 10 200135 200 150 B 15 200125 250 175 100 20 foo 128 500 100 3.0 240 150 400. 15 40 280175 500 180 ‘Table 6.4 Maxinum clear spacings (mm) for tension bars in beams % Moment redistribution ty 30 --200~«Wi ODD HO 250-0240 270300 300300300 460 11S, 1304S 160180195210 If none of these apply, the maximum spacing should be taken as that given in table 6.4, except that if the ratio 1004q/hd is less than 1.0, the values from table 644 should be divided by that ratio. f the emouat of moment redistribution is ‘unknown when usiag table 6.4 for slabs, zero should be assumed for span moments and —15 per cent for suppost moments. 6.14 Minimum Spocing of Reinforcement To permit conerete flow sround reinforcement during construction the minimum clear gap between bars, o groups of bars, should exceed (Iya * 5 mi) horizen- tally and 2gg/3) vertically, where ogy i the maximum sie of the coarse aggre gate. The gap Must also exceed the bar eiameter, or inthe case of “bundled bats the diameter of a bar of equivalent total cross-sectional area 6.1.5 Minimum Areas of Reinforcement For most purposes, thermal and shrinkage cracking may be controlled within acceptable limits by the use of minimum reinforsement quantities specified by 3S 8110, although requirements of water-retaining structures will Be more stn Bent (see chapter 11), The principal requirements are summatised in table 6.5, although other requirements include 0.15 per oznt tanstetse reinforcement in the tp surfaces of flanges in flanged beams and 0.25 per cent (High yield) or 0,30 per cent (Mild steel) ant-crac steel in plain walls (bar diameter <6 mm or 116 REINFORCED CONCRETE DESIGN ‘one-quarter diameter of vertical compressive bars). Requirements for shear links and column binders are given in stetions 7.3 and 9.3 respectively, ‘Table 6.5 Minimum reinforcement areas High yield Mia stet steel (Gy =250 (f= 460 Nim?) Nimm#) Tension reinforcement (0) Pure tension WOOAvAe = 0.87 ose ©) Flexure (0) ectangale tion loo Je (both ways in solid slabs) ~ 0.24 7 (b) flanged — web in tension Sat “OLR bu /b 204 100A by bef <0.4 1OUyeu 032% = O.18% ~ flange in tension T-beam 1004 bgt 48% 026% beam 1004/4 136% 0.20% Compression reinforcement (1) General euee® 2) Reet. column oF wall eh oe (3) Flanged beam Oe OMe flange in compression 100A e/g ‘web in compression 1004.5/5,.4 (4) Rectangular beam 1O0As/AS i =02% 0.2% 6.1.6 Maximum Areas of Reinforcement ‘These are determined largely from the practical need to achieve adequate com paction of the conerete around teinforcement. The kits specified by BS 8110 eas follows. (@) For a Slab or Beam, Longitudinal Steel 1004, 1004 204s og 100d not greater than 4 per cent each, a OS ee Le ‘Where bars are lapped, the sum of the bar sizes n a layer must not be sreater than 40 per cent of the section breadth. (®) For Column 104 go gene thn pe ces cs vey rot greater than 8 per cent if cast horizontally not greater than 10 per cent at Japs in either case SERVICEABILITY, DURABILITY AND STABILITY REQUIREMENTS 117 6.1.7 Side Face Reinforcement in Beams Where beams exceed 750 mm in depth, longitudinal bars should be provided near side Faces a a spacing > 250 mm over a distance 2/3 from the tension face. “These bars, which may be used in calculating the morvent of resistance, must have a diameter > (5, b/f,) where sy isthe bar spacing and b the brosdth of the Section (07 $00 mim if es), as indicated in igure 6.1 > oase § = oy Required wee750, Figure 6.1. Side face reinforcement in bezms 6.2 Span-Effective Depth Ratios BS 8110 specifies a set of basic span-effective depth ratios to control deflections ‘which are given in table 6.6 for rectangular sections and for flanged beans with spans less than 10-m, Where the wob width of flanged beam éy > 0.36, linear interpolation should be used between the values for a flanged beam and a rectan- Bula: section. Ratios for spans > 10 m are factared as in example 6.1 Table 6.6 Besic span-effective depth ratios. Rectangular Flanged section (ew $038) Centilever 7 56 Simply supported 20 16.0 Continuous 26 208 The basie (@) The service sties in the tension steel and the value of M/bd, as shown, in table 6.7, which is also presented in the form of a chart in figure 8.4 (0) The area of compression steel as in table 6.8 ‘The area of tension reinforcement provided is related to the value of M/bd®, thus lower values of service stress and Mod? will result in smaller depths of neutral ‘xis x. This effect will reduce deflections due to creep, as there will be less of the given in table 6.6 are modified in partculr cases according to 1s REINFORCED CONCRETE DESIGN’ ceaBILITY. DURABILITY AND STABILITY REQUIREMENTS 119 section subject to compressive stresses. Compression reinforcement rest van-eptctive cepit ratio (table 6.6) = 26. Aeflections in a similar menmer and also reduces the effects of shrinkage, i 10 co finishes, mocked rato = 26 x 7° = 21.7. Table 6.7 Tension reinforcement modification factors wkd damage i Reinforcement service stress (N/mm?) ne reinforcement modification factor: Mg MXIT a5 950075 10 15 20 30 40 56 a : 4 ja? 300% 600" 20.36 Ie atti 67 fr fy = 459N)mn?maietion itor = 08. }95 1.36 151 reson ceiaforcement modification factor: 168 150 138 121 y ona; _ 100 x 402 ‘Bd 300% 600 ‘The service stress in the reinforcement fj is ususlly 2 function of the yi fom table 6.8, modification factor = 1.07. stress fy, 28 indicated inthe table. The reinforcement areas Ay and Aj. modified span-effectve depth ratio is equal to athe centre of span, or at the support for & eantilever, and the value of aie GR =< S67 ‘with table 6 8 should include all bars located inthe compression zone, 21.7089 « L.07= 22, Table 6.8 Compression reinforcement ‘modification factors 1004, Ai Aaptor Factor bt 0.09 3.00 ois 15 te Lee ion of Defetions ta We al requirement is that neither the efisiency nor appearance ofa structure us 14 by ike deflections which wil occur durig isi. eflections must thas re 1 Sondre at wrioes ages The linatonsnesesary ro satity the req: 1s rn Mens wl vary considerabiy according fo the natne ofthe stretare and ie & as but er enfored concrete the following may be egsded a reasonable 25 tas 30 150 (1) The inal deecvon of horizontal members below the lvl of casting should not exceed span/250. (2) The detection aking place ater fang of partion o aplication of ‘cedures for making adjustments to the basic ratios to allow for other loadin * amp Sif Dt Rate Chck ae aya che ae ise ec ‘The use of these factors i illustrated by example 6.1. It should be no ‘rectangular continuous beam spens 12.m with @ mid-span ultimate mo 400 KN m. If the breadth is 300 mm, check the seceptability of an effec w 31600 na ten big ye elnforecment f= 60 Nimes wed. TD Bec oce cnn rb ates othe maleated emus ‘bars are located within the compressive zone. q ‘ions, which will have varying degrees of accuracy. 120 REINFORCED CONCRETE DESIGN (2) The precise loading and duration cennot be predicted and errors in dead loading may have significant effect. (3) A cracked member will behave differently to one that is uneracked — this may be a problem in lightly reinforoed members where the working load may be close to the cracking limits (4) The effects of floor screeds. finishes and partitions ate very difficult 10 assess, Frequently these are neglected despite their ‘stiffening” effect. tansy sometines be posible to allow fer these factors by averaging maxima and ainimm estimated effects, and provided that this is done thete area number of calculation methods avaiable which wil ge ressonable esos. The method adopted by BS 8110 i very comprehensive, andi based onthe caculetion of Curtatues of sections subjected to the appropiate moments, with llowance for creep and sivinage effec where nvesny. Deletions are then alle fom these covetres “The procedure for estimating deflections i ater lengthy and complex, vol ing the following tages which se stated in example 6.2. (1) Calculate the short-term curvature under total lead: Co. (2) Calculate the short-te deflection from (1), and ifthe long-term deflection is require: (8) Calculate the shoct-erm curvature due to permanent 1024s, C, pm (4) Calculate the long-term curvature due to permanent loads, (5) Calculate the curvature due to shvinkage. Cay {6) Estimate the otal long-term curvature Cas G= Corot ~ G.perm + G.perm * Ge (7) Calculate the long-term deflection using the value from (6). Ler ‘The curvatures in (1), (3) and (4) are taken a5 the larger value from considering the section as (a) cracked (b) uneracked. As the concrete may have cracked under the total load, the additional short-term curvature Comp due to the temporary loading is obtained from Caremp = Cerot ~ Caperm in past (6) of the procedure and is not caleulated direetly If deflections are assumed to be small, elastic bending theory is based on the expression 0) where My, isthe bending moment ata section distance x from the origin as shown § in figure 6.2 For small deflections the term d®y/dx® approximately equals the curvature, Which is the reciprocal of the radius of curvatures thus, SERVICEABILITY, DURABILITY AND STABILITY REQUIREMENTS 121 4 y Beam Sy Figure 6.2. Gurrcture ofa beam 62 where Ir, is the curvature atx. Integrating expression 6.1 twice will yield values of displacements y of the ‘member. thus if curvatures of a member are known, displacements can be deduced. ‘The analysis of deflections will use the partial factors of safety from tables 2.1 and 2.2. which effectively mean that materials properties are taken as the charac~ teristic values, and that loadings are true working loads. 6.3.1 Calculation of Curvanures — Short Term ‘The curvature of any section should be taken as the larger value obtained from considering the section to be either uncracked or cracked. & fe 7 £04 fq BONE stron sieass Figure 6.3. Cnermeked section ~ strain and sires distribution Uneracked Section ‘The assumed elastic strain and stress distributions are shown ia figure 6.3, and the "upper limit to concrete stress at the level of tension reinforcement should be noted. iba REINFORCED CONCRETE DESIGN From equation 6.2 1 rrvature ou 7 From the theory of bending hence where M_ = applied moment at section considered Eq instantaneous static modulus of elasticity of concrete (for short term deflections) J = second moment of area of section fe * maximum compressive stiess in the concrete X= depth of neutral axis, ‘The above expression gives the instantaneous curvature of the uncracked section, If this is found to be greater than for a cracked section, the tensile stress fig of the cconctete at the level of tension reinforcement must be checked o ‘ eas YONimm? short tare ce OSSNienm ong term Figure 6 Cracked section stain and sreseaisttbuston Ciacked Section ‘The recommended stress and strain distribution are given in figure 6.4 where the stiffemng effect of the cracked concrete is taken into account by the tensile stress block shown, Curvature 1 = fe ro XE. SERVICEABILITY, DURABILITY AND STABILITY REQUIREMENTS 123 Henoe itis necessary to analyse the section subjected to its applied moment BF to jbtain values of x and either fe ot fy. This caleulation is ideally suited to computer pplication, but if required to be solved manually must be performed on a tril and ertor basis ‘Considering the section equilibrium by taking morients about the centre of compression Me Ay (e- 3) + $b hfe 2) (62) and fiom the strain distribution ©. fe Bh 64) Fae i" fen sand equating tension and compression Forces A bxfe= fide +E5G fer (6.5) where fg *mmcmam tel sess alowed in the ene (ts (3) Ey. = 200 N/mm? 2, = instantaneous static modulus of elasticity of eonerete (For short-term defection) ‘The most convenient method of solving these expressions is to assume a neutral axis position; for this value of x evaluate f, from equation 6.3 and using this value obtain two values of fe from equations 6.4 and 6 5. This should be repeated for ‘wo farther trial volues of x.and a plot of f; from each expression js made against ‘x. The intersection of the two curves will yield values of x and f_ with sufficient sveuracy to permit the curvature to be calculated, This method is demonstrated in example 6.2. 6.3.2 Calculation of Curvatures — Long Term In calculating long-term curvatures itis necessary to take into account the effects of cteep and shrinkage in addition to the reduced tensile resistance of the cracked concrete as indicated in figure 6.4 Creep ‘This allowed for by reducing the effective modulus of elasticity of the concrete to Eye = Bei(I +6) where 6 i 4 creep coe! cient, equal Lo the catia of ereep Sirain to initial elastic strain ‘The valve of 6. while being affected by aggregate properties, mix design and curing conditions. i governed also by the age at first loading, the duzation of the load and the section dimensions. Figure 6.5 pives long-term values of 9, as suggested by BS 8110, and it may be assumed that about 40 per cent, 60 per cont and 80 pet cent of this will eccur within 1 month, 6 months and 30 months under load 124 REINFORCED CONCRETE DESIGN respectively for constant relative humidity. The effective section thickness is taken as Iwice the cross-sectional area divided by the exposed perimeter, but is taken as £600 mun if drying is prevented by sealing or immersion of the concrete in water 150 [ 300 cod yy as }20 S10 os foe Shrinkage Curvature due to shrinkage must be estimated and added to that due to applied ‘moments. such that 1 fee Se rs 7 where és is the fiee shrinkage strain Is the mnodular ratio &/Reye, and Ss the first moment of area of the reinforeement (5%) was ax 2% ite and a reduced Een ‘The effective section thickness equals twice cross sectional area _ 2x 700 x 300 perimeter 2(700 + 300) thus the value of @ feom figure 6.5 for loading at twenty-eight days with indoor exposure is approximately 2.75, Hence =210mm 6.93 kN/min? ‘Thus, using the same approacla as previously for the cracked analysis, it is found that when x= 300 mma then fey =4.$ N/mm®, fey =8.1 N/mm? 27 350mm fey = 8.6 Nmm®, fog = 7.1 Nim? X=370 mm fey = 7.7 Nimm?, fon =6.8 Nim? ‘Thus as can be seen from figure 6.9, the sotution lies at x = 360 mm when f.=1.0.Nimm*. Therefore Le 70 3.84 10-4fmm ry 3606.93 x 108 In this instance it isnot necessary to evaluate the uncracked ease since in part (¢) thas been established that the permanent loads yield the higher instantaneous curvature when the section is cracked. (@) Calculate Shrinkage Curvature where % 693 ‘And for a transformed eracked section (see figure 4.28) bet +02) taedeta ot io 12 REINFORCED CONCRETE DESIGN therefore with ! }60 mm From part (F) (1743.50 + 4.08) « 10? £8.75 «10? mit and Sy2 A, @-x) 2480 x 240 588 x 10° mm? From figure 6.6 for indoor exposure, the long-term value Gey 390 x 10% Thus 390 x 10- x 28.9 x 588 x 107 = B75 «10° $08 x 10-/mm (ha) Caleulate Total Lon Short-term curvature, non-permanent loads = Short-term curvature, total loads = Short-term curvature, permanent loads 5 x 10 1.8 x 10 setm Deflection = 1.0% 10-/mm Longeterm curvature, permanent loads = 2.8 x 10°4/mm ‘Shrinkage curvature = 08 x 10-4/a Therefore Total long-term curvature = 46x 10~/mm a ie hence estimated total, KL? fong-erm deflection” “7, 1.104 x 12? x 108 x 4.6.x 10* 8 mm 6.3.4 Basis of Span-Effective Depth Ratios The calculation of deflections has heen shown to be a tedious operation, however, for general use rules based on limiting the span-effective depth ratio of a member are adequate to ensure that the deflections are not excessive, The application of this method is described in section 6.2. "The relationship between the deflection ané the span-effective depth ratio of a ‘member ean be derived from equation 6.9; thus SERVICEABILITY, DURABILITY AND STABILITY REQUIREMENTS 133 ty t te H \ Fee Figure6.10. Curtanureand siren irribution deflectiona=K 1p % ‘and for small deflections it can be seen from figure 6,10 that for unit length, s 1 +m cs a where em = maximum compressive strain in the concrete - frm * tensile strain in the reinforcement K- =a factor which depends on the pattern of loading ‘Therefore LK Gat em) ‘The strains in the concrete and tensile reinforcement depend on the areas of rein- forcement provided and their stresses. Thus fora particular member ection and a pattern of loading, itis possible to determine a span-effective depth ratio to satisty 4 particular a/Z or éeflection/span limitation ‘The modified span-effective depth ratios obtained in section 6.2 are based on limiting the total deflection to span/250 for a uniformly distributed loading. For spans of less than 10 m this should also ensure that the limits of span/S0O oF 20 mm. after application of finishes are met but, for spans over 10 m where avoidance of damage to finishes may be important, the basic ratios of table 6.6 should be fectored by 10/span. For loading patterns other than uniformly distributed a revised ratio is given by changing the basic ratio in proportion to the relative values of K. as shown in ‘example 6.3, Similarly. for limiting the deflection to span/B reiedcnlon bat 22 134 REINFORCED CONCRETE DESIGN Im cases where the basic ratio has been modified for spans greater than 10 m, maximum deflections are unlikely to exceed 20 mm after construction of partitions and finishes, When another deflection limit is required, the ratios given should be multiplied by af20 where ais the proposed maximum deflection. Example 6.3 Adjustment of Basie Span to Effective Depth Ratio Determine the appropriate basic ratio fora cantilever beam supporting 2 uniform Jad and a concentrated point load at its tip as shown in figure 6.11 20KN . TORN at p= FFigae 6.11. Point Joad on centilver example HHective span tc _,| Basic ratio from table 6.6= 7 fora ud. From table 6.9. K for cantilever with w.d 1, over full length K for cantilever with point load at tip = 0.33, ‘Thus, for the point load only, adjusted basic ratio equals 028 “033 ‘An adjusted basic ratio to account for both louds can be obtsined by factoring the ‘moment due to the point load by the ratio of the K values as fellows Mya =10XL/2=5L 7 253 Mootat = 201, Adjusted bas ato = Bs aio (Mt * Moin Ha Muar + Mpoine = (Saas) ¥20 #56 6.4 Flexural Cracking ‘Members subjected to bending generally exhibit a series of distributed flexural — cracks, even at working load. These cracks are unobtrusive and harmless unless the widths become excessive, in which case appearance and durability suffer as the reinforcement is exposed to corrosion. SERVICEABILITY, DURABILITY AND STABILITY REQUIREMENTS 135 ‘The actual width of eracks in @ reinforced concrete structure will vary between ‘wide limits, and cannot be precisely estimated, thus the limiting requirement to be Satisfied is that the probability of the maximum width exceeding a satisfactory ‘alue is small. The maximum acceptable value suggested by BS 8110 is 0.3 mm at iny position on the surface of the concrete in normal environments, althoug!s ome other codes of practice recommend lower values for important members. Requirements ‘or specialised cases such a8 water-etaining structures may be more stciagent and these are given in chapter 11. calculations to estimate maximum crack widths are performed. they are based on ‘Working’ loaus with 7 = 1.0 and material partial factors of safety of ‘Yq, = 10 for steel and concrete, BS 8110 secommends that the effective modulus Of elasticity of the concrete should be taken as half the instantaneous value as siven in table 1.1 to allow for creep effects Prestressed concrete members are designed primarily on the basis of satisfying limitations which are different from those for reinforced concrete, Eee —— cy Figure 6.12 Bending of length of beam 6.4.1 Mechanism of Flexural Cracking Tais can bo illustrated by considering the behaviour of s member subjected to 3 uniform moment. ‘Allength of beam as shown in figure 6.12 will initially behave elastically through- ‘out, as the applied uniform moment Af is inereased. When the Linuting tensile strain for the concrete is reached a crack will form. and the adjacent tensile gone will no Jonger be acted upon by direct tension forces. The curvature of the beam, however causes further direct lension stresses to develop at some distance from the original crack to maintain internal equilibrium, This in turn causes further cracks to form, ‘and the process continues until the distance becween eracks does not permit sufficient censle stresses to develop to cause further cracking. These initial cracks ate called “primary cracks’. and the average spacing in a region of constant ‘moment has been shown experimentally 10 be approximately 1.67 (fr —x)and will be largely independent of reinforcement detailing. ‘As the applied moment is increased beyond this point, the development of cracks is governed to a large extent by the reinforcement. Tensile stresses in the Concrete surrounding reinforcing bars are caused by bond as the strain in the rein forcement increases. These stresses inerease with distance feom the primary cracks and may eventually cause further cracks to form approximately nuidway between. the primary cracks, This action may continue with increasing moment until the 136 REINFORCED CONCRETE DESIGN bond between conerete and steel is incapable of developing sufficient tension in the conerete to cause further cracking in the length between existing cracks. Since the development of the tensile stresses is caused directly by the presence of the reinforcing bars. the spacing of cracks will be influenced by the spacings of the re- inforcement. If bars are sufficiently close for their ‘zones of influence’ to overlap, then secondary cracks will oin up across the member. while otherwise they will form only adjacent to the individual bars. It hes been confirmed experimentally that the avezage spacing of cracks along a line parallel to, and at a distance a from, main reinforcing bar depends on the efficiency of bond, and may be taken a8 1.674 for deformed bars. oF 2.Ode, for plain round bars Figure 6.13, Bening straine 6.4.2 Estimation of Crack Widths I the behaviour of the member in figute 6.13 is examined, it can be seen thet the ‘overal extension per unit length at depth y below the neutral axis is given by 2 « @-x%) ‘where &, is the average strain in the main reinforcement over the length considered, and may be assumed to be equal 10 f,jF, whore fis the steel stress at the cracked. sections. Hence assuming any tensile strain of concrete between cracks as small, singe full bond is never developed Js <3 —9 6 ‘whore Ew = sum of erack widths per unit Fongth at level». The actual wldth of individual cracks will depend on the numberof cracks in this unit iength, the axerage number bein given by length/average spacing where average spacing, sgy~ |.67q for deformed bars; as0 Syy © 1.67(H ~ x), the Spacing of psimary cracks, Thus brags ak wah ,g —_— 2 ay. number of cracks 1" ‘The designer is concerned however with the maximum crack wideh, and it has been shown experimentally that if this is taken as twice the average value, the ethance of this being exceeded is about 1 in 100, hence for deformed reinforcing SERVICEABILITY, DURABILITY AND STABILITY REQUIREMENTS 137 bors. the maximum likely crack width Wpax at any level defined by y in a mem ber will tus be given by wimax * 12S 13.33, Of Wma provided that the rot exceeded. ‘The positions on amember where the surface crack widths will be greatest, depend on the relative values of strain (¢,) and the distance to point of zero strain (2e,). Despite the effects of bond slip adjacent to cracks. and the steel strain across cracks, the crack width at the surface of a reinforcing bar is very small and may be assured to be zero. This may therefore be taken as a point of zero strain for the purposes of measuring a... The neutral axis of the beam will also have zero strain, and hence ace may also relate to this if appropriate. €13.33(h ~ x) based on the primary cracks is 3 2 position 9 Nee nd raintorermant) zr 2 Figure 6.14 Celta! crack postions Critical positions for maximum crack width will on 2 beam generally occur at the positions indicated in figure 6.14. These occur when the distance to points of 2z2r0 strain, that is. reinforcement surface or neutsal axis, ave as large as possible Positions | and 2 will have a maximum value of strain, while at position 3, although the strsin is sate, ais considerably larger. The expression for Wings at any Point may thus be expressed in the general form ‘maximum surface erack ‘width at a point = constant x distance to the surface of the nearest reinforcing bar or neutral axis x apparent tensile strain in the concrete at the level considered ‘The expression for meximum surface crack width given in BS 8110 is basically of this form. with the constant based on a probability of the calculated value being. ‘exoeoded of somewhat greater than ] in 100. The expression is given as 3erém ‘where ¢min isthe minmum cover 10 the main telnforcement and eq is the average Concrete strain and is sed on e, but allows for the stiffening effect of the cracked concrete in the tensioa zone. The value of em, is given by an empiries] expression REINFORCED CONCRETE DESIGN _ bi =x) a = 9) 364, -3) ‘where By isthe width of section at centroid of tensile steel and a the distance ftom ccompresive face to the point at which crack is calculated, This expression allows for variations of steel stess between cracks, and results in correspondingly reduced maximum crack width estimates. A negtlvevelue of indicates thatthe section fsuncracked, 6.4.3 Analysis of Section to Determine Crack Width ‘Whatever formula is used, it is necessary to consider the apparent concrete strain at the appropriate position. This must be done by elastie analysis of the cracked section using half the instantanzous value of £, to allow for creep effects as dseussed in section 6.4. ‘The methods discussed in section 4.10.1 should be used to find the neutral axis position x and hence f, the stress in the tensile reinforcement, Then Yh @-a hhence é_y may be obtained. JH lo ete 2°90 4omm bers (nin cover =50m) (©) Detail of Reiatercamant Position (a) Gross. Section Figure 6.15 Example 64 Calculation of Flexural Crock Widths Estimate the maximum flexural crack widths for the bear section shown in figure 6.18a when subjected to a moment of 650 KN m. Characteristic strengihs of concrete foy of steel fy Modulus of elasticity of steel £, = 200 kN/mm? ‘SERVICEABILITY, DURABILITY AND STABILITY REQUIREMENTS 139 Caleulate Neutral Axis Position and Steel Stress From table 1-1, instantaneous modulus of elasticity = 26 kN/mm?, therefore cen 8 eran? ‘Then from seetion 4.10.1 the neutral axis position is given by Aw - 2 Aa=0 rea of three no. 40:mm bars = 3770 mm? fxt+ In this case A, = 1000 ~ (20 +50) = 930 mm thus 1.490 xx? + 29 4.3770 ~ 2 x 3770 x 930 3 B therefore sex _ 204 V(200" + 4 269 700) 2 394mm (Alternatively cherts may be used, as in figure 4.29 in which ease Ay, 20 3710 156 bd” 13 400% 930 taking A = 0, s/d = 0.42 from charts and hence x = 391 mm.) ‘The stress in the reinforcement ge 650 x 108 *” @—x/3)4g 798 3770 216 N/mm? thus ae PX 2.04 x 107% and tn =e, — = 2) G@' =x) m3 Bly (d —§) ‘The maximum crack width will occur either at position 1 or 2 indicated on figure 6.135 thus. 140 REINFORCED CONCRETE DESIGN Position ¥ Ger ae xi0-* 1 606 470? + 707} — 1000 124 ° «a0? + 268%) — 20 652 ass Minimum eater, cin = 50min thus a position 1 a ( : ‘epg 50.03) dre 3 x 200 x 3770 (930 — 394) =112x 107 nnd we ieee (2 we BX79M 112x107 50 rae ) i000 — 398 and similar at position 2 [oss - ‘tora — sn (60298) ei? 3 x 200 x 3770 (930 — 394) =0.50 «10 = 0.24 mam thus = 34257 «0.50 « 10 Ml 2 = 0.23 mm (82) 606, ‘The maximum exack width of 0.24 mm is therefore likely to occur at the bottom. comers of the member, and the cracks are likely 1o be at an average spacing of 1.67a, = 1,67 x 79 © 130 ram at these positions. Cracks of similar width may coveur on side Faces at a spacing of approximately 1.67 x 257 = 430 mm. 6.4.4 Control of Crack Widths It is apparent from the expressions derived above that there are two fundamental ways in which surface crack widths may be reduced (1) Reduce the stress in the eeinforcement (/.) @) Reduce the distance to the nearest bar (ae) The uso of stol at reduced stresses is generally uneconomical, and although this approach is used in the design of water-etaining structures where cracking must often be avoided altogether. itis generally easier to limit the bar cover and spacing SERVICEABILITY, DURABILITY AND STABILITY REQUIREMENTS 141 and hence deg, Durability requirements limit the minimum value of cover: however bars should be es close to the conerete surface as is allowed. Reinforcement spacing may be reduced by keeping bar diameters as small a i reasonably possible. ‘Since the side face of a beam is often a critical erackwidth position it is good practice to consider the provision of longitudinal steel in the side faces of beams fof moderate depth. Recommendations regarding this, and spacing of main rein forcement, are given by BS 8110 and are discussed in section 6.1, If these recom ‘mendations are followed, itis not necessary to calculate crack widths except i unusual circumstances, Reinforcement detalling however. hss been shown to have ‘large effect or flexural cracking. and must in practice be a compromise between, the requirements of cracking, durability and constructional ease and costs b fe fe. 1 a Figure 6.16 Forces adjacent 10 @erack 6.5 Thermal and Shrinkage Cracking ‘Thermal and sivinkage effects. and the stresses developed prior to cracking of the concrete were ciscussed in chapter I. After cracking, the equilibrium of concrete adjacent to a erick is illustrated in figure 6.16. [Equsting tersion and compression forces Asha * Ache ~ Ashe or fam FE On So) ‘i the condition is considered when steel and concrete simultaneously seach thei limiting values in tension, tha is, fa = fan foe = fy = tensile strength of concrete A appropriate «ge — usualy taken as thee days: Then An fi i+ fee here ris she sel ratio. The vale of fg canbe calculated but is generally very small and may be taken 4 zero without introducing undue imaceuriey hence the cvitical vale of steel ratio fi £ spproximate 6.109" G55 E somoxiaey (6.107 IE the stel ratio is es than this value, the steel wil yield in tension cesulting in few wide cracks; however i is greater then move cracks wil be formed when the erie We REINFORCED CONCRETE DESIGN ‘SERVICEABILITY, DURABILITY AND STABILITY REQUIREMENTS 143 tensile sivess eaused by bond between the steel and concrete exceeds the concrete nerete strain) x crack spacing with the maximum width corresponding to the tensile srength, that fs maxim Sp2cing Of Sng fosbus > he Wanax * (Coy + Tae ~ 26a) Snax 12) whore fy = average bond stress wae y= tinksee eats 1 = development length along » bar : Du, = sum of perimeteis of reinforcement, T= fallin temperature from hydretion peak For a round bar a, = coefficient of thermal expansion of concrete — taken as ‘ (5 x the value for mature concrete. to allow for creep effects ® In practice, variations in restraints cause large variations within members and between otherwise similar members, with ‘full restraint seldom occurcing. The ‘behaviour depends considerably on this and temperatures atthe time of casting. Guidance concerning possible “restraint factors’ i given in Part 2 of BS 8110. Hence, since for similar bars then Example 6.5 Caleulation of Shrinkage and Thermal Crack Widths ‘A fully restrained seotion of seinfozoed concrete wall is 150 may thick, and drying ® slrinkage strain of 50 microstrain (eq) is anticipated together with a temperature dcop (7) of 20°C after setting, Determine the minimum horizontal reinforcement to conivol cracking snd estimate maximum crack widths and svorage spacing for a fe suitable reinforcement arrangement. me | Thuee-day ultinare tensile stength of concrete (f,) = ultimate average bond stress (fo) = 15 Nina Modulus of elasticity of eonerete (#:) = 10 kNjmnm? and thus "The maximum eck spacing istic thi ylu innmeditely prior to the formation Of anew rack, whan the development length on both sess ig, hat Is Coeficent of theymol expansion for mtu eoneret (a) Fn Ee any = 12 mictostrain°C ms Cack spacing nd hence width, therefor, s governed both by the reinforcement sieand quanti Zor tio above the rita alo, which shouldbe ‘aen ata Modul of elastic of seinfrcement (6) tninnu teguirement for conrad ercking Empirical values fr genera se ae Characteristic yield streagth of reinforcement (f,) = 460 N/mm? 200 kN/mm? 1s ven n section 6-1 Crit tel ratio =033 po cent f fay 033 933 150 1000 6.5.1 Gack Width Cteuaion 100 ‘The expressions fr erick spacing asume that the tte thermal and shvinkage 25 mn? strains are sufficient to cause cracking, although in practice itis found that pre dicted eracks may not always occur. Its possible to estimate however the maxi- rmurn erack width likely to occur by considering total concrete contraction in conjunction with the maximum likely crack spacing. For steel ratios greater than ‘This could be conveniently provided as 10 mm bars at 300 mm centres in each face f the member (524 mm! /m), FFor this reinforcement, the maximum erack spacing is given 2s the critical value, and when the total contraction exceeds the ultimate tensile hs fe 13x10 strain for the concrete (éyye. the tensile stress in the concrete increases from zero mes ee at acrack to maximum value at mid-distance between cracks. Hence the mean 24th ax Ae 130000 430 mm tensile strain in the uncracked length is eqy/2 when a new crack f just about 10 form, The crack width is thus given by crack width = (total unit movement — 14a REINFORCED CONCRETE DESIGN Since the minimum spacing is given by one-half of this valve, the average spacing. will be Syy = 0.79 x 1430 = 1072 mam. "The maximum crack width corresponds to ymax and is given by nn) 4s piven in equation 6.12 where ultimate tensile strsin for the concrete wna tax (0a 17 SE - 50 microstrain therefore 20x =0.14 mm 6.6 Other Serviceability Requirements ‘The two principal other serviceability considerations are those of durability and resistance to fire, although aecasionally a situation arises in which some other factor may be of importance to ensure the proper performance of a structural ‘member in service. This may include fatigue due to moving toads or machinery, ot specific thermal and sound insulation properties. The methods of dealing with such requirements may range from the use of reduced working stresses in the materials, to the use of special concretes, for example lightweight aggregates for good ther- smal resistance 6.6.1 Durability Deterioration will generally be associated with water permeating the concrete, and the opportunities for this to occur shouldbe minimised as far as possible by pro viding good architectural details with adequate drainage and protection to the conctete surface Peimeabilty isthe principal characteristic of the concrete which affects dur ability although in some situations iti necessary to consider also physical and chemical effects which may cause the concrete to decay For teinforeed concrete a further important aspect of durability isthe degree of protection which i given to the einforcement, Carbonation by the atmosphere will in ime. destioy the alkalinity of the surface zone concrete, endif this reaches the level ofthe reinforcement will render the steel vulnerable to corrosion in the presence of moisture and oxygen. If concrete is made with a sound inert aggregate, deterioration will nt occur in the absence of an external influence. Since concrete i highly alkaline materia SERVICEABILITY, DURABILITY AND STABILITY REQUIREMENTS 145, its sesistanee (0 other alkalis s good, but it is however very susceptible to attack by acids or substances which easily decompose to produce acids. Concrete made with Portland cement is thus not suitable for use in situations where it comes into contact with such materials, which include beer, milk and fats, Some neutral salts ‘may also attack concrete, the two most notable being calcium chloride and soluble sulphates. These react with 2 minor constituent of the hydration products in Giflerent ways. The chloride must be in concentrated solution, when it has a sel. vent effect on the concrete in addition to corrading the reinforcement, while sulphates need only be present in much smaller quantities to cause internal expan- sion of the conerete with consequent cracking and strength loss, Sulphates present the most commonly met chemical attack problem for eanerete since they may ‘cour in groundwater and sewage. In such cases cements containing reduced pro- portions of the vulnerable tricalcium aluminate, such as Sulphate Resisting Portland Cement ot Super Suiphated Cement, should be used. Te addition of ulvorised Fuel Ash (Pf) or ground granulated blest furnace slag (ggbf8) may’ also be beneficial. Table 6.10 indicates minimum concrete mix requirements for use in situations where sulphates are present, Both chlorides and sulphates are present in sea water, and because of this the chemical actions are different, resulting in reduced sulphate damogp, although if the concrete is of poor quality, serious damage may occur from reactions of soluble magnesium salts with the hydrated compounds, Well constructed Ordinary Portland cement structures have neverthe- less been found to endure for many years in sea water, Table 6.10 Concrete exposed to sulphate attack Concentration of sulphates Min, (803) total Mex Cas, "Cement type cement free Insoil Tn groundwater conteat water! (eotalso,) gil) (gi?) cement <0.2% <03 Any = 7 2 020005% — 931012 Any 330050 oPc/RHPC+ 25-40% Pha 310 oss or 70-90% egbts SRPC orSsC 2800.55 3 OSto1o% — 1.21025 OPC/RHPC 25-40% Pfa or «3804S 70-90% agbts SRPCorssC 3300.50 4 10t020% —25t050 —SRPCorssC_— 370s 5 >20% >5.0 SRPCorSSC_ 370.85 + protection Note: These values relate to dense concrete with 20 mam max. apgegate size. 146 REINFORCED CONCRETE DESIGN Physical attack of the concrete must also be considered. This may come from abrasion or attrition as may be caused by’ sand or shingle, and by alternate wetting and drying. The later effect s particularly impostant in the case of marine struc- tures near the water surface, and causes stresses to develop ifthe movements produced ae restrained, Is also possible for crystal growth to oceur from drying. ‘out of sea water in cracks and pores, and this may cause further internal stresses, leading to cracking, Alternate freezing ané thawing is another major cause of physical damage. particularly in toad and runway slabs and other situations where water in pores and cracks can freeze and expand thus leading to spalling. It has ‘een found that the entrainment of a small percentage of arin the concsete in the form of small discrete bubbles offers the most effective protection against this f ‘of attack. Although this reduces the strength of the concrete, it i recommended that 4.5 £ 1.5 per cent by volume of entrained ait should be included in concrete subjected to regular wetting and drying combined with severe frost All these forms of attack may be minimised by the production of a dense, well compacted concrete with low permeability. thus restricting damage to the surface zone of the member. Aggregates which are likely to react with the alkali mattix ‘should be avoided, ss must those which exhibit unusually high shrinkoge character istics. IF this is done, then permeability. and hence durability, is affected by (2) aggregate type and density Q) water-cement ratio G) degree of hydration of cement (4) degree of compaction, A low water-cement ratio is nesessary to imi the voids du to hydration, which rst be well advanced with the assistance of good curing techniques. Coupled ‘ith thisis the need for non-porous aggregates which sre hard enough to resist any attrition, and for thorough compaction, Ii essential thatthe mix is designed to have adequate workability fr the situation in which tis tobe used thus the cement content ofthe mix must be rcsonably high 3S 8170 specifies minimura cement contents or various exposure condition, a well nim strength and maximm water cement rt, related to mine Thum cover requirements as described in section 6.1.1 “The consequences of thermal eifects on durability must not be overlooked, and very high ezment contents should ely be used in conjuneton witha detailed Cracking assessment. BS S110 suggests that 550 kg/m? cement content should be regarded as an upper limit for general use Provided thas such measures are takea, and that adequate cover of sound concrete is given to te reinforcement. deterioration of reinforced concrete is unlikely. Thus although the surface conerete may be affected, the reinforcing seo il remain protected by an alkaline concrete matrix which has not heen carbonated by the §tmosphete, Once ths cover breaks down and ster and possibly chomicas can reich the sec, rusting and consequent expansion lead rapidly to cracking and spalling oF the cover concrete and severe damage ~ visually and sometimes structurally SERVICEABILITY, DURABILITY AND STABILITY REQUIREMENTS 147 6.6.2 Fire Resistance Depending on the type of structure under consideration, it may be necessary to ‘considar the fie resis:ance of the individual concrete members. Three con ust be examined (1) effects on structural strength (2) flame penetration resistance | in the ease of dividing members (3) heat transmission properties [such as walls snd slabs ‘Concrete and stet in the form of reinforcement o prestrssing tendons exhibit reduced steength ater being subjected to high temperotutes. Although concrete hus low thermal conductivity, and thus good resistance to temperature rise, the strength begins to diop significantly at temaperatures above 300°C and i has a fendeney to spall at high temperatures. The extent of this spalling is governed by tne type of aggregate, with siloeous materials being particularly stsceptibie while caareous and ighteight aggregate concretes suffer very litle. Reinforcement Syl tain about 50 per cant of its normal strengtn aftr reaching about 550°C, ‘hile for prestressing tendons the corresponding temperature is nly 400°C “Thus as the teraperatue rises the heat is transferred to the interior of a con- elo member, with a thermal gradient established inthe concrete. This gradient wil be affected by the area and mass bf the member in addition tb the Uierml properti ofthe concrete, ané may lead to expansion and loss of strength, Dependent on the thickness and nature of cover, the stel will sein temperature and lose strength, this leading to deflections and eventual structural fire ofthe rember ifthe steel temperature Becomes excessive. Design must therefore be aimed at providing ard maintaining sound cover of concrete asa protection. thas {saying the temperature cise in the stel. The presence of plaster, sereeds and other non-combustible finishes assists the cover in protecting the reinforcement and may thus be alloved for in the design BS $110 givas tabulated values of minimum dimensions and nominal covers for various types of concrete member which are necessary to permit the member to withstand fire fora specified period of time. Although these values. which have teen summarised in tubles 6.2 and 6.3, €0 not take into account the influence of aggregate type, they may be considered adequate or most notmal purposes. Mote detailed information concerning design for fixe resistance is given in Part? of BS 8110 including concrete type, member type and details of finishes. The petiod that a member is required to survive, both in respect of strength in relation to Working loads and the containment of fire, will depend upon the type and usage of the structure - and minimum requirements are generally specified by building "egulstions. Prestressed concrate beams must be considered separately in view of the increased vulnerability of the prestrssing ste 62 Stability While it would be unreasonable to expect a structure to withstand extremes of accidental loading as may be caused by collision, explosion or similar happening, {tis important that resulting damage should not be disproportionate to the cause. [t follows therefore that a major structural collapse must not be allowed to be 148 REINFORCED CONCRETE DESIGN ‘caused by a relatively minor mishap which may have a reasonably high probabillty. of happening in the anticipated litetime of the structure ‘The possibilities of « structure buckling or overturning under the “design” loads ‘ill have been considered 9s part of the ultimate limit state analysis. However, in some instances a structure will not have an adequate lateral strength even though it has been designed to sesist the specified combinations of wind load end vertical load. This could be the case if there is an explosion ora slight earth tremor, since ‘then the lateral loads are proportional to the mass of the structure. Therefore itis recommended that a structure should always be capable of resisting a lateral force not less than 1. per cent of the total eliaracteristic load acting through the centroid of the structure above any level considered, Damage and possible instability should also be guarded against wherever possi for example vulnerable load-bearing members should be protected from collision by protective features such as banks or barriers 6.7.1 Ties In addition to these precautions. the general stability and robustness of a butlding structure can be increased by providing reinforcement acting as tes, These ties should act both vertically between roof and foundations. and horizontally around, and across each floor, and all external vertical load-bearing members should be anchored to the floors and beams. Vertical Ties Verteal tes are not generally necessary in structures of less than five stozeys, but in higher buildings should be provided by reinforcoment, effectively continuous fom roof to foundation by means of proper laps, running through all vertical loade bearing members, This stel should be capable of resisting a tensile force equal to {he maximm design ultimate load carried by the column or wall fom any one storey or the roof, In in siru concrete. this requirement is almost invariably sais: fed by a nonmal design, but joint detailing may be affected in precast work, Horizontal Ties Horizontal ties should be provided for all buildings, irrespective of height, in three ways (2) peripheral ties Q) internal ties G) column and wal ties. ‘The resistance of these ties when stressed to their characteristic strength is givea. terms ofa force F, where F; = 60 kN or (20+4 x aumber of storeys in struc KN. whichever is less. This expression takes into account the increased risk of an accident in a large building and the seriousness of the collapse ofa tall structure. G@) Peripheral Ties ‘The peripheral tie must be provided, by reinforcement which is effectively con- tinuous, around the perimeter of the building at each floor and roof level. This SERVICEABILITY, DURABILITY AND STABILITY REQUIREMENTS 149 ‘column thas. Peripheral tre std oe igure 6.17. Te forces reinforcement must le within 1.2 m from the outer edge and at its characteristic stress be capable of resisting a force of atleast F (b) Internal Ties Internat ties should also be provided at each floor in two perpendicular directions and be anchored at euch end either to the peripheral te of to the continuous column or wal ties, These ties must be effectively continuous and they may either be spread evenly actos floor. of grouped at beams of walls as convenient, Where walls ae used the tie tefnforcement must be concentrated in the bottom 0.5 m. ‘The resistance required is related to the span and loading. Internal ties must be capable of resisting a foree of F, KN per metre width o [Fy(gy + qx)/7.5] L/3 kN ber metre width if this is greater In this expression, is the greatest horizontal distance in the direction ofthe tie between the centres of vertical load-bearing ‘members. or if smaller. 5 x the clear storey height messused to underside of the beams. The loading (gy + q,) KN/m? isthe average characteristic load on unit ares of the Noor considered. Internal ties parallel to cross-walls eccurring in one dicec- tion only, on plan, need only resist the force F, KN pee metre width, (©) Column and Wall Ties Column snd wall ties must be sble to resist a force of atleast 3 per cent of the {otal vertical ultimate load for which the member has been designed. Additionslly, the resistance provided must not be less than the smaller of 2F; or Fylo/2.5 KN ‘where fais the floor to ceiling height in mettes. Wall ties are assessed on the basis Of the sbove forces acting per metre length of the wall, while column ties are eon: centrated within 1 m either side of the column centre line. Particular cate should be taken with comer columns to ensure they are tied in two perpendicular directions, 180 REINFORCED CONCRETE DESIGN In considering the structure subjected to accidental loading it is assurned that no other forces are acting. thus reinforcement provided for other purposes may also act as ties. Indeed, peripheral and internal ties may also be considered to be. acting as column or wall tis, Full ancroroge tangtn (in nenerage ne stress soe) f - ey 2s | a) ' 4 a) o> te Anenorage Reguiremants for Internal ‘ies Full anchorage trgin Hy oi toy I oy OF as (op ertey — Veg j Peripneral ne Anchorage Requirerants for Colum & Wall Ties Figure 6.18 Anchorage of tes ‘As with vertical ties, the provision of horizontal ties for situ construction will seldom affect the amount of reinforcement provided. Detailing of the rein- forcement may however be affected. and panticular attention must be paid to the ‘manner in which internal tes are anchored to peripheral ties. The requirements for ‘the full anchorage of ties are ilustrated in figure 6,18. If these are not met, then the assumed stresses in the tles must be reduced appropriately. Precast concrete construction however presents a more serious problem since: ‘the requitements of tie forces and simple easly constructed joints are not always compatible. Unless the required tie forees ean be provided with the bars anchored by hooks and bends in the case of columan and wall ties, an analysis of the structure rust be performed to assess the remaining stability after a specified degree of structural damage, ‘SERVICEABILITY, DURABILITY AND STABILITY REQUIREMENTS 151 Example 6.6 Stability Ties Catoulate the stability ties required in an eight-storey building of plan area shown, in figure 6.19 Clear storey height under beams = 2.9 m Floor to ceiling height (lo) = 3.4m Characteristic dead load (gy) = 6 KNfas? Characteristic live load (qu) = 3 KNfa? ‘Characteristic stect strength (/,) = 460 N/mm? Fy=(20 + 4x number of storeys) = 2044 8=S2KN <6OKN ronsvarse Figure 6.19, () Periphera ties Force to be resisted = Fy = 52 KN Bor area required = 521° 2443 sum? 0 This could be provided by one T12 bar. (6) internat res = Fran) L Forse to be esisted = FBR 26) Ly por mete 1 St he KN per metre (1) Transverse direction Force = xD =s74enim> Fh 735 Force per bay =87.4 x 6.5 = 568.1 KN ‘Therefore, bar area required in each transverse interior beam is 152 REINFORCED CONCRETE DESIGN 568.1 x 10° 460 This could be provided by 4120 bars. = 1235 mm? (2) Longitudinal direction 52(643) 65, 7S Force = 1.1 kN/m > F, ‘Therefore force along length of building = 81.1 x 7 = $67.7 KN, hence bar area required in each longitudinal bear is 567.7 x10 2x 460 This could be provided by 2120 bars. =617 mm? 8) Column ties Force to be designed for is by 3a) te) r= (34) 52-70. (4) @)= OT EN <2; : 4 75x 10" 163 mm? Area of ties requited = (©) Vertical tes Maximum column load from one storey is approximately equal to (1.6 x3 + 14x 6)x3.5 x 6.5 = 3003 KN ‘Therefore bar area required throughout each column is equal to 300.3 x 10° “460, ‘This would be provided by 4T16 bars. = 653 mm* SERVICEABILITY, DURABILITY AND STABILITY REQUIREMENTS 153 6.7.2 Analysis of ‘Damaged’ Structure ‘This aust be undertaken when @ structure has five or more storeys and does not comply with the veticaltie equizements, or whea every precast floor or roof unit {oes not have sufficient anchorage to resist a force equal to F, KN pat metre width Sting in the direction ofthe span. The analysis must show that each key load tearing member, its connections, and the horizontal members which provide lateral support are able to withstand a specified loading fzom any direction, If ‘his cannot e satisfied, then the analysis must demonstrate thatthe removal of any single vertical Toad-bearing element, ther than key members, at each storey in turn will not result in collapse ofa sigificant part ofthe structure ‘The minimum loading thet may act ftom any direction on a key member is recommended as 34 kN/mn* in BS 8110. The decision as to what loads should be considered acting is left to the engineer, but wil generally be on the basis of permanent and realistic live-oading estimates, depending on the building usage ‘This method is attempting therefore to asess quantitatively the effects of excep- tional loading such as explosion. The design ‘pressure’ must thus be regarded asa somewhat arbitrary value The ‘pressure’ method will generally be suitable for application to columns in precast framed structures; however, where precast load-bearing panel construction is being used an approach incorporaung the removal of individual elements may be more appropriate, In this case, vertical loadings should be assessed as described, and the structure investigated to determine whether it is able to remain standing bya different structural action. This action may include parts ofthe damaged structure behaving as a cantilever ora catenary, and it may aso be necessary to consider the strength of non-load-beering partitions or cladding Whichever approach is adopted, such analyses are tedious, and the provision of eifective tie forces within the structure should be regarded asthe preferred solu tion both from the point of view of design and performance Continuity reinforcement an¢ good detailing wil gretly enhance the overall fe resistance of a structure with respect to collapse. A fire-damaged structure ‘with reduced member strength may even be likened to a structure subjected to sccidental overload, end analysed accordingly. 7 Design of Reinforced Concrete Beams Reinforced concrete beam design consists primarily of producing member dotails which will adequately resis the ultimate bending moments, shear forces and tor- sional moments. At the same time serviceability requirements must be considered to ensure that the member will behave satisfactorily under working loads, It is difficult to separate these two criteria, hence the design procedure consists of a sates of interrelated steps and checks. These steps ate shown in detail in the flow chart in figure 7.1. but may be condensed into three basic design stages, (1) preliminary analysis and member sizing (2) detailed analysis and design of reinforsement (3) serviceability calculations ‘Much of the matecal inthis chapter depends on the theory and design specifics tions from the previous chapters. Ths loading and caleulation of moments nd shear forces should be cartied out using the methods described in chapter 3. The equations used for calculating the eas of reinforcement have been derived in chapters 4 and 5. Full details of serviceability requirements and calculations are given in chapter 6. but it is normal practice to make use of simple sles which are specified inthe Code of Practice ard are quite adequate for most situstions, Typical of these are the span-efective depth ratios to ensue aoeptable deflections. and the rules for maximum bar spacings. nd minimum quantities of reinforeement, which ae 10 limit cracking, as desribed in shapter 6. Design and detailing of the bending seinforcement must allow for factors such as anchorage bond between the steel and concrete. The area of the tensile bending reinforcement also affects the subsequent design the shear and torsion reinforce iment. Arrangement of reinforcement is constesined both by the requirements of the codes of practice for concrete stuctutes and by practical considerations such as construction tolerances, clearance between bars and available bar sizes and lengths. Many of the requirements for cortect detailing ae illustrated in the ‘examples which eal with the design of typical beams, 14 esen cLause ae gat Poaae Pease DESIGN OF REINFORCED CONCRETE BEAMS ease Loxns CONCRETE GRADE ? + ESTIMATE SELF WEIGAT CONCRETE CovER wonnusd secrion bursary & FIRE t rrfurmany amr RES ain & ESTIMATE @ FROM Mest feu 0186 SINGLY REINFORCED G86 372mm For mild conditions of exposute the cover = 25 mm (table 6,1). So for 10 mm. links and, say, 32 mm bars overall depth =a + 25 + 10+ 32/2 DESIGN OF REINFORCED CONCRETE BEAMS 159 ‘Therefore make & = $25 mm as an integer number of brick courses. $e that = 525 ~51=474 mm shear stress y= 0 = U2x 10% bd ~ 230x474 = 1.03 Nérum™ Por grade 30 concrete, maximum v allowed = 0.8 30= 4.38 N/mm®. Therefore < 438 2 Basle span-effective dept 84<20 474, A bean size of 230 mm by $25 mm deep would be suitable Weight of beam = 0.23 x 0.525 x 4.0 x24 E16 ‘which is sufficiently close to the assumed value ‘The calsulation of main bending reinforcement is performed using the equations and charts derived in chapter 4. In the cae of rectangular section which require only tension see, the leverarm curve method is probably te simplest. Where Compression ste is equired, either design charts or * manvsl epprosch with the simplified design formulae may be used. When design charts are not applicable, as in the ease of non-fecangular sections, the formulae based on the equttalent reotangular tess bloc wil simplify caleustions considerably. The type ofreinfercing steel to be used must be decided initially anes this, in conjunetion with the chosen conerete grade, will affect the areas requied and also {influence bond ealevations. in most ctcumstances one of the avaiable types of highsielé bars wll b used unlss ercking i critical, a or example in water. "staining structures. when mild stel may be preferred. Areas of reinforcement ate aleulated tthe sections with maximum moments, and sitable bar sizes selected, (Tables of bar areas are given in the appendix.) This permits anchorage 0.13 for highyteld or 0.24 for mild steel DESIGN OF REINFORCED CONCRETE BEAMS 161 085 $080 2 7 Comoression fl reintorcement| 88 f Fequred oof +} ——-} + nox rom MI om 0-08 ow Gas OE Med? fea The % values on the K oxis mork the limits for singly reinterced sections mith moment redistribution upplied [see Section £.7) Figure 7.5 Lererarmoune Example 7.2 Design of Tension Reinforcement for a Rectangular Section ‘The beam section shown in figure 7.6 has charaeterstie material stengths of ‘fox 30 Nimm? for the concrete and f, = 460 N/mm? for the stel, The design ‘moment atthe ultimate limit state is 165 KN m which causes sagging of the beam 25230 a ij t 4-490) | t+ |e @ L gure 76 162 REINFORCED CONCRETE DESIGN M165 x10 bd*foy 230 x 4907 x30 ‘This is less than 0.156 therefore compression steel is not required. From the leverarm curve of figure 7.5 l, = 0.87, therefore 87 x 490 = 426 mm and Me 165 x 10% OB7e 087 x 460% 426 Provide two T25 bats, area = 982 mm?. For the steel provided 1004, _ 100x982 bh 230x550 968 mm? Ay 0.78 and 913 < 104s <40 bh therefore the stee] pereentoge is within the limits specified by the code. 7.2.2 Rectangular Section with Compression Reinforcement Compression stel is required whenever the eonerete in compression is unable, by itself, te develop the necessary moment of resistance. Design chorts such asthe One in igure 4.9 may be used to determine the stel areas but the simplified equations based on the equivalent rectangular stress block are quick to apply. ‘The maximum moment af resistance that con be developed by the concrete the neutral axis at the maximum depth allowed by the code of practice 1 Gepth is given as x= 04) a> 05a 02 rnoment atthe section after redistribution ‘moment at the section before redistuibution where By = This reduction is due to the designer redistributing the moments from an elastic analysis of the structure. as described in sections 3.4 and 4,7. With x less than d/? the stress in the compression steel may be considerably less than the yield. therefore, the design procedure is somewhat diffesent if Bi less than 09. It should also be noted that. in order to meintain the limitation on the depth ‘of neutral axis as specified in oquation 7.2, the areas of reinforcement required and provided should mect the following requirement Aicproe — Aiseg) > Asproe ~ Assea) 3) This sto ensure a gradual tension type failure with yielding ofthe tesion rein- forcement as explsined in chapter & DESIGN OF REINFORCED CONCRETE BEAMS. 163 ° cog YL Hel fe| exis 4 Ps Equivalent rectongulor Section strains stress block Figure 7.7. Beom doubly ebvfoveed to resist @saging moment Moment Redistribution Factor By 30.9 and did > 0.2 Waa is not greater than 0.2, as is usually the case. the proportions of the strain diagram wall ensure that the compression steel will have yielded. Compression reinforcement is required iF > 0.156 fy bd? and the design equations as given in section 4.5 are (1) Area of compression steel ie orseste oy O87F, (dd) (2) Ares of teasion steel As 0.156 foobd® gs D156 Soubd? 4. 41 os 7 O87 fz 2 with bver ann 2 =0.775d afd is greater than 0.2 the stcess in the compression steel should be determined 4s outlined in part (2) of example 7.4 Moment Redistribution Factor fy <9 per cont ‘The limiting depth ofthe neural axis can be calslated from equation 7.2 and compresion steels required if a>oastids (1-2) 08) 164 REINFORCED CONCRETE DESIGN where s= depth of stress block = 0,9, The design procedure is (1) Calculate K = abd fy (2) Caloulate K" = 0.402 (By ~ 0.4) ~ 0.18 (Fy ~ 0.4)" ICK SK". compression steel i not required s0 proceed as fora singly reinfoiced section asin example 7.2 IK > £’. compression stel fs requited () Calgulste x= (By — 04) Id fx <0.43, the compression stel has yielded and foc = 0.87 fy If'd'fx> 0.43, ealulate the stel compressive stain ée and hence the stios fain example 7.4, (4) Calculate the area of compression steel from Ate ROK) fosb?™ SOT, doa’) om (5) Calculate the area of tension steel from: Age Kfestil® y 4 So 78) 0.87 fz Where z =d - 0.9x/2 Links should be provided to give lateral restraint to the outer layer of compres: sion steel. according to the following rules. (1) The links should pass round the corner bars and each alternate bar. (2) The link size should be at east one-quarter the size of the largest compression bar. (3) The spacing of the links should not be greater than twelve times the size of the smallest compression bar. (4) No compression bar should be more than 150 mm from a restrained bar. Example 7.3 Design of Tension ond Compression Reinforcement, By > 0.9. ‘The beam section shown in figure 7.8 has characteristic material strengths of fey = 30 Nimm? and fy = 460 N/mm*. The ultimate moment is 165 kN m, causing hogging of the beam. Mu 165 x 10 ba fey T0330" X30 =0.22 > 0.186 so that compression stee] is required, and d'Id = 50/330 = 0,15 <0.2 therefore fn = O87 fy DESIGN OF REINFORCED CONCRETE BEAMS 6s ob: 230 ne 390 s Figure 78 Beam doubly reinforced to rest a hogging moment From equation 7.4 Compression steel AS = oer = (165 « 10° ~ 0.196 30230 x 3304) 87 x 450 (330 - 50) = 427 mm? And from equation 7.5 0.156 foybd® 4s tension stella, = RE +Ah 156 x30 230% 230? , jog 0.87 x 460 x 0.775 x 330 = 1572 mn? Provide two 720 bars for 4;, area = 628 mm? and two T32 bars for As, area = 1610 man? so that for the ateas of steel required and provided in equation 7.3 628 — 427 > 1610 ~ 1572 Also 004; _ 100x628 bh 230x390 load, _ 1001610 | bh 230390 ‘thesefore the bir areas are within the limits specified by the code. ‘The minimum link size = 20/4 = 5 mm, say 8 mm links, and the maximum link spacing = 12 x 20 = 240 mam, centres. The link size and spacing may he governed by the shear calculations. Figure 7.8 shows the arrangement of the reinforcement 1 resist a hogging moment 0.70 179 166 REINFORCED CONCRETE DESIGN Exomple 7-4 Design of Tension and Compression Reinforcement, fy = 0.7, ‘The beam section shown in figure 7.9 has characteristic material strengths of fou = 30.Nimm? and fy = 460 N/mm? The ultimate moment 3s 370 kN m. causing hhogging of the beam. fw ru ey a =sea] no 300 ae a 4 od035 section strains Figure 7.9. Beam doubly reinforced o reste hogetg moment ‘As the moment reduction factor By = 0.7. the limiting depth of the neutral X= 04d K= Mfbd? fn, = 370 10% (300 x 540® x 30) 1a K’=0.402 (8, ~0.4) —0.18 (6, 04)? = 0.108 K> K" therefore compression steel is equited: d'fx= 100/162 = 0.62 >0.43 therefore fie < 087 fy co) = 0.0085 (x = d') x = 2.0085 (162 ~ 100) 162 Steel compressive strain ey. = 0.00134 DESIGN OF REINFORCED CONCRETE BEAMS 167 (2) From the relevant equation of section 4.1.2 Steel compresive stess = Ey ec 100.000 x 0.00134 = 268 Némm? @ (KEK fest? Fed’) = (N41 ~ 0.164) 30 x 300 x 5407 268 (540 — 100) 823 mm? Compression steel Ay @ Tension steel Ay = 87f,2 = —0:104%30%300%5407 gry, _ 268 087 x 460(540 0.9% 1632)" 087» 460 = 2011 mm? Provide two T25 bai for Az. area = 982 mm? and two T32 plus one T25 bars for Ag.atea = 2101 mm, which also meet the requirements of equation 7.3, ‘These areas lie within the maximum and sninimum imsts specified by the code To restrain the compression stel. atleast 8 mm links at 300 mm centres should be provided 7.2.3 Tebeams Figure 7.10 shows sections through a T-beam and an L-beam which may form part ofa conerete beam and slab floor. When the beams are resisting sagging moments. part of the slab actsas a compression flange and the members may be designed as T: or L-beams. With hogging moments the slab will be in tension and assumed to be cracked. therefore the beam must then be designed as a rectangular section af Width By and overall depth f ‘When the slab does act asthe flange its effective width is defined by empirical rules which are specified in BS 81 10 a5 Follows, (1) Tsection — the lesser of the actual flange width, oF the width of the web plus one-fifth of the distance between zero moments (2) Lsection — the lesser of the actual flange width or the width af the ‘web plus one-tenth of the distance between zero moments. AAs a simple rule, the distance between the points of eco moment may be taken as, 0.7 times the effective span for a continuous beam. Singe the slab acts a5 a large compression atea, the stiess block for the T- or [section usually falls within the slab thickness. For this position of the stress los REINFORCED CONCRETE DESIGN i & _ a ee | a oo Figure 7.10. Tdeamend t-bearn block. the section may be designed as an equivalent rectangular section of breadth Br. Transverse reinforcement should be placed across the top of the flange to preva ‘tacking, The area of this reinforcement should nor be less than 0.15 per cent of longitudinal eross-section of the flange. Design Procedure 1) Calculate Af/bqd fey ond determine fy from the leversarm curve of figue 7.5 Leverarm2= td or from equation 7.1 (2) Wd — 2 94 4kh eran SF Diogrer 5-R10 at 5-R10 ot 220 R10 links ot 260 cd yp is 280 —4 =) 8 Tt 2002-10 nanger bars om Figute 7.19 Nomconttnucus beaorshear roinforcament Wy 4x10" bd 300x550 = 1.3 Némm? <08 Vfey shear stress,» (b) Shear links Distanced from face of support shear Va = Ve Wad #214 752.x055=173 kN shear stress = 1310 2.05 nf? 300 « 550 Only two 25 mm bars extond a distance d past the critical section. Therefore for determining ¥ 1004s _ 100% 982 Lg gy bd -300* 550 From table, $.1,y¢ = 0.56 Njmm? ye) _ 300(1.05 ~0.56) _ ‘0.87 * 250 0.68 178 REINFORCED CONCRETE DESIGN Provide R10 links at 220 mm centues Agr 2 2X75 oy sy 220 (e) Nominal links For mild stee links Ay, 046 _ 04%300 Sy OBTfy 0.87 250 Provide R10 links at 280 mm contres 2a BS 280 = 056 (@) Extent of shear links Shear resistance of nominal links + concrete is tye (Aeasra. vi) Sheur ceinforeement is required overa distance s given by ge Bola = 24-1589 Wa 52 = 0.73 metres froin the face of the support Number of R10 links a1 220 mi required at each end of the beam is 1+ (9/220) = 1 + (750/220)= 5 7.3.2 Renvup Bors In regions of high shear forces it may be found that the use of links ta carry the full force will cause steel congestion and Jead to constructional problems. In these situations, considerstion should be given to "bending up" main reinforcement wl is no longer required to resist oonding forces. At least 30 per cent of the shear resistance provided by the steel should be in the form of links. Fora ‘double system’ of bent-up bars at 45” and spaced (d ~ a“) apart, as described in section 5.1.2, the shear resistance is VELe Bhd where Ags isthe cross sectional area of a bent-up bar. Bentp bars must be fully anchored past the point at which they are acting as tensioa members, as Indicated in figure 5.3. To guard against possible crushing of DESIGN OF REINFORCED CONCRETE BEAMS 19 Table 7.1 Shear resistance in kN of hent.up bars, ‘double system’ Bar size ty Sea (inn?) 12 le 20 25 32 40 250 35 62 36 151 247 387 460) 64 ng 7 78 458 713 the concrete it may also be necessary to check the bearing stress inside the bends fof» bar. This stress is given by Bearing stress = fe vere Fog i the tense force inthe barr ste internal rads of the bend, nd & isthe barsize, Thi stress should net exceed _2Sey 14 30ia, Whete wy is the conte to cente distance between bars perpendicular io the plane af th bend, but Fors bar adjacent tothe fae ofa member ty = + side cover Example 7.8 Bearing Stresses inside a Bend Determine the inside radius required for the 25 mm bent-up Use shown in figure 7.20, thet the ultimate bending stress is not exeweded. The bar hos side cover ‘of 50 mm, Assume the bar i at theultimate tensile stress of 0.87, and the characteristic material strengths are f, = 460 Ninmn? and fey = 30 N/mm? ay = + cover=25-+50=75 mm therefore fs 2530 a6 Nim? 1+2@ja, 142% 25/75 Fy, 087x460%d, | 087 460 x 491 ro 1x25 Br = 2860 thos 7360 ue <36 180 REINFORCED CONCRETE DESIGN rp BO sr9mmorso 36 rtm var Figure 7.20 Radius of bond fora bent.up bar 7.4 Bar Spacing ‘There are limitations on the minimum and maximum spacing of the veinfoscing bars. In the case of minimum values ti is governed by constructional require- ments to allow for the access of poker vibrators and the flav af concrete te obtain a well-compacied and donse concrete, The maximum limitations on spacings are to prevent excessive cracking caused by shrinkage of the concrete and thermal ‘expansion and contraction of the member, These serviceability requirements are ‘dealt with in chapter 6. 7.$ Continuous Beams Beams, slabs und colurans of a cast in situ structure all act together to forma continuous Joad-bearing structure. The reinforcement in 2 continuous beam must be designed and detailed fo maintain this continuity by connecting edjacent spans and tying together the beam and its supporting columas. There must also be trans- verse reinforcement to unite the slab and the beam, The bending-moment envelope is generelly a series of sagging moments in the spans and hogging moments at the supports as in figure 7.21, but occasionally the hogging moments may extend completely over the span. Where the sagging moment ‘occur the beam and slab act together, and the beam can be designed as a T-ection. At the supports, the beam must be designed ss a rectangular section — this because the hogging moments cause tension in the slab. ‘The moment of resistance of the concrete T-beam section is somewhat greater than that of the rectangular concrete section at the supports, Hence it is often advantageous to redistribute the support moments as described In chapter 3. By this means the design support moments can be reduced and the design spaa_ ‘moments possibly increased, DESIGN OF REINFORCED CONCRETE BEAMS INL Desiga of the beam follows the procedures and rules set out in the previous sections. Other factors which have to be considered in the detailed design ste as fallows. () Atan exterior column the beara reinforcing bars which resist the design moments must have an anchorage bond length within the column, @) A minimum area of transverse reinforcement must be placed in the top Di the slab, across the effective flange width as described in section 7.2.3, (3) Reinforcement in the top of the slab must pass over the beam steel and sill have the necessary cover. This must be considered when detailing the beam reinforcement and when deciding the effective depth of the beam at the suppart sections. (4) The column and beam reinforcement must be carefully detailed so that the bare can pass through the junctions without interference, 45m Bom 2725 q 2y25 prs 3 | 38 mo T Tae | Ul Figure 7.21, Arrangement of bending reinforcement Figure 7.21 illustrates a typical arrangement of the bending reinforcement for ¢ {wo-span continuous beam. The reinforcement has been arranged with reference to the bending-moment envelope and in sccordance with the rule for anchorage and ‘curtailment described in sections 7.2.4 and 7.2.5. The bending moment envelope has been divided into sectors equivalent to the moment of resistance of each rein forcing bar. Ths establishes the cut-off points beyond which the bars must extend atleast a curtailment anchorage length. It should be noted that atthe external Columns the reinforcement has been bent down to give » full anchorage bond length Is2 REINFORCED CONCRETE DESIGN Sf concrata + re ae tl y im an i Toa llateal onl of 00 Figure 7.22 Arangement of soo reinforcement “The shear-force envelope and the arrangement of the shear reinforcement for the saime continuous beam is shown in figure 7.22. On the shear-force envelope the tesistance of the concrete plus the nominal stircups has been marked and this shows the lengths of the beam which need shear reinforcement. Wien designing shear reinforcement, reference should be made to the arrangement of bending nforcemient to ensure that the longitudinal tension bars used to establish Ye extend at least an effective depth beyond the section being considered. Example 7.9 Design of a Continuous Beom “The beam is 300 mm wide by 660 man deep with three equal 5.0 m spans. In the transverse direation, the beams are at 4.0 ra centres with 2 180 mm thick slab, as shoven in figure 7.24. The live load gy on the beam is SO kN/m and the dead tosd gy, including seli= weight, is 85 KN) Characteristic material strongths are fay = 30 N/mm. fy = 460 N/mm? for the longitudinal steel and fy = 280 N/mm* for the links. For a mild exposure the ‘minimum concrete cover is to be 25 mum, For each span ultimate load wy = (1.4g4 + 1.6q,) kN/motre (1.4 x85 + 1.6 x 50)= 199 N/metre DESIGN OF REINFORCED CONCRETE BEAMS 183 ° so-tee sontet ° Moment = oor. ore o9er Shecr ¥ = O45F OGF OSSF O55FO5F O4SF 4s 5:0m 4) t+ 50m £2 5:0 Fr tq 160, Figure 7.23. Continuous Bear: with ultimate bending moment and shear force coefitents ‘Total ultimate load on a span is F199 5. = 995 kN As the loading & uniformly distributed, qy, > gx. and the spans are equal, the co- efficients shawa in figure 7.23 have been used to calculate the desiga moment and shears. Bending (2) Mid-span of Ist and 3rd Spans ~ Design as @ T-soction Moment M = 0.09 FL = 0.09 x 995 x $= 448 KN m Effective width of flange = by + 0.7L/5 0.7 x 5000 5 =3004 therefore M8518 song, BP fe” 1000 6008 x30 From the leverarm curve, fy = and d= 600 ~ 570=30-0.156 Thus. compression steel fs required. M ~ 0.186 fay bd? 087), 4-4) _ S47 x 10% — 0.156 x 30 x 300 x 5807 0.87 x 460 (580 — 50) Ay = 352 mm? ‘This area of stee! will be provided by extending the span reinforcement beyond supports, 0.156 feybd? O87 fyz = 9.156 x 30 x 300 x $80 087 x 460 x 0.775 x 580 4 Ay +352= 2977 mm? Provide two T32 plus three T25 bars, aroa = 3080 mm? (top steel). (c) Mid-pan of 2nd Span — Design as a Tsection M=0.07 FL = 0.07 x 999 x 5=348 kN m Using the lever-arm curv, itis found that J = 0.95 M__ 348x108 DSIRE ~ 087x460 (0.95 x 600) Provide one T32 plus to 725 bars, area = 1786 mms® (bottom steel. 4 525 mn? Shear (@) Check maximum shear stress Moximum shear at face of support is V, = 0.67 — wy x support width/2 = 0.6 x995 ~ 199 x 0.15 = 567 KN Ve 367 x10? ba” 300% S80 26 Nim? vein in table 72, then torsional reinforcement i required. _ ‘Refer to lable 73 for the reinforcement requrements witha combine roe ‘tion of torsion and shear stress. = 30 2” (4) ¥ +», must not be greater than yy in table 7.2 where » isthe shear outa stress due to the shear force. Also for sections with yy <550 mm. y Pein 033 037 040 me oar vu 4.00 4.38 $.00 ‘where y, is the larger centre-to-centte dimension of link. 45) Caleulate the additional shear reinforcement required from Aw r sy Bun ORTH) where x, is the smaller centre-to-centre dimension of the link. This ‘Table 7.3 Reinforcement for shear and torsion value of 4y/sy added to the value from step 1, and a suitable link = size and spacing is chosen, but 5 ?.min > Me mia 5 <200 mm of x1 ¥Sy_+0.4 Nominal shear reinforcement, Designed torsion reinforcement n reinforcement but not less t ‘The links should be of the closed type shown in figure 7.26 no terson reinforcement oe reinforcement (6) Calculate the additional area of longitudinal steel shear reinforcement »>ve+04 Designed shear reinforcement, Designed shear and torsion A te (Ben +n) no torsion reinforcement reintorcement uy 190 REINFORCED CONCRETE DESIGN Example 7.10 Design of Torsional Reinforcement ‘The rectangular section of figure 7.26 resists a bending moment of 170 KN m, shear of 160 KN and 8 torsionel moment of 10 KN m. The characteristic material strengths are foy = 30 Nimm*, fy = 460 Némun? and fyy = 250 Nimm®. (1) Cateulations for bending and shear would give y= 1100 mm? and 4s <0 . (2) Torsional shear stress or, Hin Bmax ~ Maia!) 22x10 x 10 300" (S00 300/3) (3) 0.56 > 0,37 from table 7.2, Therefore torsional reinforcement is required. % = 0.56 N/mm* (6) ¥ _ 160x108 bd ~ 300x450 therefore vty = 119+ 0.56 1.75 Nim? ¥ty from table 7, 38 N/mm. therefore 4.38 x 440 550 $0 Ghat vy <9 /S50 05 roquited. () Additional 4x = ___T __ fe 08 ORT) 100% 108 0B 1240 x 440 x 087 250 = 119N/mmt 5 DESIGN OF REINFORCED CONCRETE BEAMS 191 therefore « ‘The links are of the closed type with their ends fully enchored. (6) Additional longitudinal steel (8) > 250 = 055 x 52 (240 + 440) = 203 mn? % Tap (240+ 440) = 203 mar therefore total steel area = 1100 +203 = 1303 smn? Provide the longitudinal steel shown in figure 7.26 (2) The torsional reinforcement should exterd at leest imax Beyond where itis required to resist the torsion. 8 Design of Reinforced Concrete Slabs Reinforced concrete slabs are used in floors, roofs and walls of buildings and a8 the decks of bridges. The flo system ofa structure can take many forms such as init solid slabs, sibbed slabs or precast units. Slabs may span in one direction or in two directions and they may be supported on monolithic concrete beams, steel beams, walls or directly by the structute’s columns. Continuous siabs should in principle be designed to withstand the most un- favourable arrangements of loads, in the same manner as beams, Beceuse there ere greater opportunities for :edistribution of loads in slabs, analysis may however often be simplified by the use ofa single load case, provided that certain con- ditions are met as described in section 8.1. Bending moment coefficients based on this simplified method are provided for slabs which span in one direction with approximately equal spans. and also for lat slabs. “The moments in slabs spanning in two directions can also be determined using coefficients tabulated in the code of practice. Slabs which are not rectangular in plan or which support an irregular loading arrangement may be analysed by techniques such asthe yield line method or the Hileborg strip method, as| described in section 8.10. Conerate slabs behave primarily as flexural members and the design is similar to that for beams. although in general it is somewhat simpler beceuse (1) the breadth of the slab is already fixed and a unt breadth of 1 m is used in the caleulations (2) the shear stresses are usually low ina slab except when there are heavy concentrated loads, and @) compression reinforcement is seldom required. 8.1 Simplified Analysis BS 8110 permits the use of a simplified load arrangement forall slabs of max- ‘mum ultimate design load throughout all spans or panels provided that the following conditions are met: 192 DESIGN OF REINFORCED CONCRETE SLABS 193 Stab definitions (2) Ina one-way slab, the area of each bay £30 m* (see figure 8.1). (b) Live load gy > 1.25 Dead load gy (@) Live load a, > 5 kN/m? excluding partitions, Tf analysis is based on this single load case, all support moments (except at a cant lever) should be reduced by 20 per eent and span moments increased accordingly [No further redistribution is then permitted, but special attention must be given (0 cases where a cantilever is adjacent to a span which Is less than three times that of the cantilever. In this situation the condition where the cantilever is fully loaded and the spa unloaded must be examined to determine possible hogging moments in the span, Tabulated bending momeat and shear force coefficients for use with approxi- ‘mately equal spans and when these conditions are satisfied are given in section 8.5.2 For one-way spanning slabs and in seetion 8.7 for fat slabs 8.2 Shear in Slabs The shear resistance of a solid slab may be ealeulated by the procedures givea in chapter S. Experimental work has indicated that, compared with beams, shallow slabs fal at lightly higher shear stresses and this is incorporated into the values of Aesign ultimate shear stress ve given in table 5.1 ‘The shear stress ata soction in a solid slab is given by ‘where is the shear force due to the ultimate load, dis the effective depth of the slab and b is the width of section considered, Calculations are usually based on & Strip of slab I'm wide. 194 REINFORCED CONCRETE DESIGN ‘The code requires thet for a solid slab (1) »> the eter of 08 Vf or § Nima? Q) »}y, fora slab thickness less than 200 mm. (3) Ifv > vc, shear reinforcement must be provided in slabs more than. 230 mi tek If shear reinforcement is required, then nominal steel, as for beams, should be provided when » < (ve + 0.4) and "designed’ reinforcement provided for higher ‘alues ofp. Since shear stresses in slabs due to distributed loads are generally smal, Shear reinforcement will seldom be required for such loads. Localised ‘punching’ sctions due to heavy concentrated loads may, however, eause more critical eon- ditions as shown in the following sections, Practical difficulties concerned with bending and fixing of shesr reinforcement lead to the recommendation that it should not be used in slabs which are loss than 200 mm deep. 6.2.0 Punching Shear ~ Analysis ‘A concentrated load (V) on a slab causes shearing stresses on a section around the load: this effect i referred to as punching shear. The inital critical section for shear js shovin in figure 8.2 and the shearing stress is given by a Perimeter of the section xd (Qa 426+ 12d)a where @ and b are the plan dimensions of the concentrated load. No shear forcement is required ifthe punching shear stress, v v, and shear rein- 145 mm = 0.80 Nim? From table 5.3.¥ forcement is required For vertical inks = vedud O87 hye ~ ¥c) = 03 N/mm? isles than the minimum 0.4 N/mm? required, thus take (e— 9) =Odand A 0.4% 3780x215 A ‘O87 x 250 = 1498 im? A 1495 ‘oval numberof 8 mm links required = Aa = _1495_ sore ns ad8]4 2x 503 =1s ‘The links must be distributed evenly between two perimeters within the failure Zone. The spacing between the legs of the links must not be greater than L3d= 15 x 215320 mm, Position the links on two perimeters 150 mm and 300 mm from the face of the load, The kngths of these perimeters are uy = 4x 600 = 2400 mm and us = 4 «900 1600 min 198 REINFORCED CONCRETE DESIGN 2100 (2400 + 3600) ‘Nuraber of links on perimeter, us = 15 —6 Spacing of legs of the links = (2400 + 3600)/(2 x 15) = 200 1am < 320 men [Number of links on perimeter, uy = 15 x (€) Check secone perimeter at (1.5 + 0.75)d from load face Perimater side = 300 +2 x 2.25 x 215 = 1268 mm and perimeter w= 4 x 1268 = $072 mm. 650108 ud ~ 30722218 ‘As > ye, nominal reinforeement is tll required within the fallure zone associat ‘with the second pesimeter = 45072 x 215 Thus = 0.60 Nimm* A = 2006 mm? ¥ 0.87 x 250 ‘006 for8 ram inks 295 = 20 are requied 35503 1m pat (b), on the perimeter at 300 mm from the load face 9 inks ae already provided, thus at least L1 further links ae required. These could be provided at 450 mm irom the load face by similar links at approximately 400 mm centees. (2) Check third perimeter at (1.5 + 1.5)d from the load face Perimeter side = 200+ 2 43x 218 = 1590 mm and perimeter u= 4 x 1590= 6360 mm. Thusy = Le 8010 29.68 Nin? ud * 6360215 [As this is ess than eno further reinforcement is required. It should be noted, however that wherever links are required. top steel must slo be provided in the slab at 200 mm centres to ensure proper fixing and anchorage of the shesr links. 8.3 Span-Effective Depth Ratios Excessive deflections of slabs will cause damage tothe ceiling, ocr finishes and other architectural details, To avoid this, limits are set on the span-depth ratlos. These limits are exactly the sume as those for beams es described in section 6.2. ‘Asa slab is usually a slender member the restrictions on the span-depth ratio DESIGN OF REINFORCED CONCRETE SLABS 19 ‘become mote important and this can often control the depth of stab required. In + terms of the span-effective depth ratio the depth of the slab is given by ‘minimum effective depth = ——__ Pam _ basic ratio x modification factors ‘The modification factor is based on the area of tension steel in the shorter span when a slab issingly roinforced st midspan but if a slab has both top and bottom steel at mnid-span the modification factors for the areas of tension and compression steal a5 given in tables 6.7 and 6.8 are used. For convenience, the factors for tension siel have been plotted in the form of «graph in figure 8.4 It.can be seen from the figure thats lower service stress gives a higher modifica, tion factor and hence a smaller depth of slab would be required, The service stress may be reduced by providing an area of tension reinforcement greater than that required to resist the design moment, or alternatively mild steel reinforcement ‘with its lower service stress may be used. The span-depth ratios may be checked using the service stess approprlate to the charactersti stess of the reinforcement. as given in table 6.7, Thus a service stress of 288 N/mm? would be used when f, is 460 N/mm®, However, ia more accurate assessment of the limiting span-depth ratio is required the service stress 4Jecan be ealeuated from toctor Modificotion 00 30, 20 30 40 50 60 wreat igure 8.4. Modification fectors for span-effectve depth rtto 20 [REINFORCED CONCRETE DESIGN Sip, dena BO Asso By where Aigg = the area of reinforcement required st midspan Jee Cate art af atest rovied ed sen AP Ins ati ugar ors ay ‘edsibtion ‘The second patt of example 8.3 illustrates the calculations to determine the servi stress, and how the provision of extra reinforcement reduces the depth of slab aquired fe ‘8.4 Reinforcement Details To resist cracking of the concrete, codes of practice specify details such as the ‘minimum area of reinforcement required in section and limits to the maximum and minimum spacing of burs. Sore of these rules re as follows: (2) Minimum Areas of Reinforcement ; 13h for high yield steel 100 24h or for mild steel in both directions. (@) Maximum Spacing of the Bars These requirements are described in detail in section 6.1.3 and are similar to beams ‘except that for thin slabs, or if the tensile steel percentage is smal, spacings may be increased from these given in table 6.4 to a maximum of 3¢. (0) Reinforcement in the Flange of @ T- or L-beam ‘When the slab forms the Mange of a T- or L-bear the area of reinforcement in the flange and at right angles to the beam should not be less than 0.15 per cent of the: longitudinal crass section of the flange. (4) Curtailment and Anchorage of Reinforcement ‘The general rules for curtailment of bers in a flexural member were discussed in soction 7.2.5, Simplified rules for curtailment in different types of slabs are illustrated in the subsequent sections of this chapter, At a simply supported end the bats should be anchored as specified in figure 7.14 or figure 8.5. DESIGN OF REINFORCED CONCRETE SLABS 201 Greoter of "or 30mm l £ L|_ = o Wy sin Figure 8.5. Anchorage or simple support fora stab 8.5 Solid Slabs Spanning in One Direction ‘The slabs are designed as if they consist ofa series of beams of | m brent, The ‘main stel isin the direction of the span ard secondary or istibution steel is requited in the transverse direction. The main stel should form the outer layer ‘of teinforeement to give it the maximum lever arm ‘The calculations for bending reinforcement follow a similar procedure to thet used in beam design. Th lever-arm curve of figure 75 is used to detersine the lover arm (z) and the are of tension reinforcement i thea given by My oa7hz Forsolid slabs spanning one way the simplified rules for curtailing bars as shown in figute 8.6 may be used provided the loads sre substantially uniformly distributed, With a continuous sla itis also necessary that the spans are approximately equal 4nd the simplified single Joad case analysis has been used. 8.5.1 Single-span Solid Slab The affective span of the slab is taken asthe lesser of: (a) the centreto-centre distance of the bearings. or (b) te elear distance between supports plus the effec ‘se depth ofthe seb. The Base span-effestive dept ratio fo his typeof slab iso 202 REINFORCED CONCRETE DESIGN 407 100% 40% i a as Simply Supported 50% of midspan steel e203 620154, ctas p 08 100% ~—— som 49% 100% anv or oz ‘ Continuous Sia Figute 8.6. Simplified rules for curtalment of bare stab spanning in one dvection 10-30 20 DESIGN OF REINFORCED CONCRETE SLABS 203 Example 8.3 Design of a Simply Supported Siab ‘The slab shown in figute 8.7 s to be designed to carry a live load of 3.0 kN/m*. plus floor finishes and ceiling Jods of 1.0 kN/m®. The chacacterstic material Strengths ate fey ~ 30 N/mm* and fy = 460 N/men*. Basic span-effective depth ratio= span 20 x modification factor m.. 4300 _ 225 Dxmf. me. ‘therefore minimum effective depth el = (1) Estimating the modification factor to be of the order of 1.3 fora lightly reirforced slab. Try effective depth d = 170 mm. For a milé exposure the cover = 25 mm. Allowing, say,$ mm as half the diameter of the reinforeing bar ‘overal depth of sab h = 170 + 25 +5 = 200 mm self-woight of slab = 200 x 24x 10-* = 4.8 kN/m? total dead load = 1,044.8 = 5.8 kN/m? For a 1 mwidth of slab ultimate load = (1.4gy + 1.64.) 4.5 = (14x 5.8 + 1.6 x 3.0) 4.5 = 58.1 KN Mm 53.1 4.5/8 = 32.7 KN in ‘Span-Effective Depth Ratio UM 32.7108 bd? 1000 x 1707 From table 67. for f= 288 Nim? the span-effective depth modification fector= 1.34. Therefore - limiting 88" _. = 20x 1.34268 cdfestve depth actual —S82____, $500 Logs fective depth ~ 170 Thusd= 170 mm is adequate 13 Bending Reinforcement _M_ _ 327x108 BaF, 1000x 170" x30 From the lever arm curve of figure 7.5, fy = 0.95. Therefore 95 x 170= 161 mm 038 lever arm z= lad 2 REINFORCED CONCRETE DESIGN 32.7 x10! O8Tfz 087 x460% 161 = 508 mm?/m Provide T10 bars at 150 mm centres, Ay =523 mm?/m, ‘Shear At the face of the support 58.1 (24 shear r= 582 ( 2 V . 276x10 2 gr stress,y= Vw 27.6% 10% <08 Vou Shear st Bi” Goon e179 70°16 Nii Ve 100A, | 100%323 4 5, bd 1000 170 fiom table 5.1.1— = 0.58 Nima? and since » <¥, no further shear checks ot reinforcement ae required End Anchorage (figuee 8.5) 016 30 mm or aes end bearing = 230 mm. therefore x anchorage length » 27mm beyond the centee line of the support. Distribution Steet Area of transverse high-yield reinforcement = 2 100 = 260 mm fn Provide T10 at 300 mm centres, DESIGN OF REINFORCED CONCRETE SLABS 208 (2) The second part of this example illustrates how a smaller depth of slab is adequate provided it i reinforced so that there isa low service stress in the steel and therefore a high modification factor for the span-effective depth ratio. Try a thickness of slab. t= [70 mm and d= 140 mm, Selfeweight of slab = 0.17 x 24 = 4.08 kNfm? total dead load = 5.08 KN/mn? Ultimate load = (1.4¢,, + 1.6q,) 45 = (14 x 5.08 + 1.6 x 3.0) 4. =536kN Bending _M_ 302710 495) bd* fy 1000 x 1407 x 30 From the leverarm curve igue 7.5 ‘Therefore using mild ste! bars = 1130 mim? /m, Provide R12 at 100 mm centres, A, Span-Effective Depth Ratto M 302108, ba? ~ J000% 140" 154 Service stress fis given by the equation of section 8.3 se Sp Aaseg 1 en Bhyx Sua 87 Aapoe * By 5 1055 3 280 x 1055 3* 1130 From figure 8.4, for M/bd? = 1.54, span-effective depth modification factor = 1.7. Therefore = 146 Nim? limiting = 20x 1 seen 934, effective depth 2 206 REINFORCED CONCRETE DESIGN eh _, fective depth 140 ‘Therefore d= 140 mm is adequate actual 8.5.2 Continuous Solid Slab Spanning in One Direction For a continuous slab, bottom reinforcement is required within the span and top reinforcement over the supports. The effective span isthe distance between the | centte lines of supports and the basic span-eitective depth ratio is 26:1 If the conditions of section 8.2 are met for the single load case analysis, bend ing moment snd shear force coefficients as shown in table 8.1 may be used. Table 8.1 Ultimate bending moment and shear force coefficients in one-way spanning slabs DESIGN OF REINFORCED CONCRETE SLABS 207 1 T 1 TT 1 4 t 7 r i rl ri ry r 1 1d i It 1 Al id A 1 | polg SO gl Ege ee 4 a ei a is 1 it ia 1 I 1 i tl ty 1 4 al us ui L. 1 rH mH tH t Outer Middle of Fist ierior Middle of —_Inerior support end span support itetior sian supports op a i bts jam | asm asm Moment = 0 0.086 FE 0.086 FL 0.063 FL 0.063 FL 7 Elevation Sher OF osF : osF ‘Nore: F's the total design ultimate load om the span, and L is the effective span Example 8.4 Desist of a Continuous Solid Slab ‘The four span slab shown in figure 8.8 supports live load of 3.0 XN jm? plus p floor finishes and a ceiling load of 1.0 KN/m®, The characteristic materia strengths ate foy * 30. N/mm? and fy = 480 Nim. Base spree depth ato #26 S100 5 $500 2175 99 % ‘Try effective depth ¢= 140 mm, and with a mild exposure overall depth, = 170mm. self-weight of slab = 170 x 24 x 107? = 4.08 kNfim* total dead weight = 1.0% 4.08 08 kN ultimate load F per span = (1-4gy + 1.6qx) 4.5 = (14 x 5.08 + 1.6 x3.0) 45 = 53.6 KN per metre width Figure 8.8 Continuous sled Bending Since the bay size > 30 m®. the spans are equal and gy 1.28 g, the moment co- efficients shown in table 8.1 may be used. Thus for the first span ‘= 0.086 FL = 0.086 x $3.6 x 4.5 = 20.8 KN m Span-Effecive Depth Ratio M 28x10 | bd? 1000x140" es From table 6.7. span-depth modification factor = 1.26. Therefore limiting P88 nag y 1 362 e sffective depth LagPks: actual —SPan___ 4500 _ 55 ifective depth ~ 140 208, REINFORCED CONCRETE DESIGN ‘Thus d = 140 mm is adequate, Bending Reinforcement M 208x108 | bd? fog 1000 x 140° 30 From the lever.arm curve, igure 75,5 = 0.95. Therefore lever arm 2 = fyd'= 0.95 « 140 133 mm _M 20.8 x 108 Oa7jyz 087 x 460x133 = 391 mm? per metie 0.035 Provide T10 at 200 mm centres, A, = 393 mm?/n, Simi calesaton forthe supprts nd the nto pan gv the tel reas shown in gure 8 (verte interior support beams 1004, (by > 0.15 forthe enforcement provid and therefore exta tes not egued forthe flange ofthe beam, ‘AL the end shpports there sa monolhie conection between the lab and th team, therefore top ste! should te provided to ret any negative mont. The area ofthis ste! shuld not be es than fll the sen of tea at midspan, aft to provide the 0.13 per cont of sc! forte flange ofthe tea, T10 bas a 500 mm entes have teen spc. "he yout ofthe reinforcement in gue 8.9 i uoordng to the simpiea rues forthe curtallnent of ats in sabe as usted in igure 86 O13 be 100 0.13 « 1000 x 170 100 Provide T10 at 350 mum centres top an bottom, wherever there is main rei forcement. ‘Transverse reinforcement = 221 mm? /m 110-300 10-200 110-350 710-280 \ | | \ ‘ 2 10-2090 110-350 110-250 110-250, DESIGN OF REINFORCED CONCRETE SLABS 209 8.6 Solid Slabs Spanning in Two Direetions ‘hen a slab is supported on all four ofits sides it effectively spans in both dizec- tions, and itis sometimes more economical to design the slab on this bass. The [mount of beading in each direction will epend on the ratio ofthe two spans and the conditions of restraint at each suppo IF the slabis square and the restraints are similar along the four sides then the joad will span equally in both dreetions. Ifthe slab is rectangular then more than ‘one-half ofthe lozd will be carried inthe stiffer, shorter direction and les in the Jonges direction. Ifone span is much longer than the other, lage proportion of the Load wil be carted in the short direction und the slab may a5 wel be designed as spanning in only one direction. Beam ¢ Figur 10 Loads coried by supporting beams ‘Moments in each direction of span are generally calculated using coefficients ‘which are tabulated in the codes of practice, Areas of reinforcement to resist the ‘moments are determined independently for each direction of span, The slab is reinforced with bars in both gicections parallel to the spans with the steel for the shorter span placed furthest from the neutral axis to give it the greater effective dopth. ‘The span-effective depth ratios are based on the shorter span and the percent ge of reinforcement in that direction, With a uniformly distributed load the loads on the supporting beams may {enerally be apportioned as shown in figure 8.10 8.6.1 Simply Supported Slab Spanning in Two Directions ‘A-slab simply supported on its four sides will deflect about both axes under load and the comers will tend to lift and cufl up from the supports, causing torsional ‘moments, When no provision has been made to prevent this lifting or to resist the torsion then the moment coefficients of table 8.2 may be used end the maximum ‘moments are given by 210 REINFORCED CONCRETE DESIGN My, * oggnl? in dection of span iy and My = yt? in direction of span ly where May and May are the moments at mid'span on strips of unit width with 1, andl, respectively, and 1 4gy + 1.6q4)5 tat isthe total ultimate Load per unit area 2 = th length of tho Tonger side 1, = the length ofthe shorter sige and dy Gy Are the moment coefMiients from table 8.2. ‘The area of reinforcement in directions 2y and ly respectively are Agg= 5 tl — per metre wiath 08742 and My — ss per metse wi An gary Pet mete width The slab should be reinforced uniformly scross the full width. in each direction. ‘The effective deptid used in calculating 4,y should be less than thet for Aye ‘because of the different depths of the two layets of reinforcement, Ac least 40 per cent of the midspan reinforcement should extend to the supports and the remaining 60 per cent should extend to within 0.1", oF O.1ly of the appropriate support. ‘Table 8.2 Bending moment coefficients for slabs spanning in two Girections at right angles, simply supported on four sides bie 10 WW 12 13 i418 tm, 0.062 0.074 0.084 0.093 0.099.104 0.113 ay 0.052 0.061 0.089 0.05 0,051 0.046 0.037 _ 00 Example 8.5 Desig the Reinforcement fora Simply Supported Slab 220 mm Thick axd Spanning in Pwo Directions ‘The offective span in each direction is 4.5 m and 6,3 m and the slab supports @ live losd of 10 kN/m®, The characteristic material strengths are fay = 30 Nm! and fy = 480 N/m? F Ma DESIGN OF REINFORCED CONCRETE SLABS 21 Iyfly = 6.348 = 14 From table 8.2, a = 0.099 and ayy = 0.051 Sell weight of slab = 220% 24x 10% ultimate toad n S.3kNIm? Ane + 1.6ay = 145.3 41.6 x 10.0= 23.4 kNIm® Bending ~ Short Span With mild exposure conditions take d= 185 mm, Mg = Agri? = 0.099 23.4 x 4.5? 15.9 KN m 46.9 x 108 1000 « 185" x30 95. Therefore 76 mm, May bd fas From the lever-arm curve, figure 7S, fy lever arin? = 0.95 x 184 = 0.046 and 46.9 x 108 087 1 460 x 176 = 666 mm fin Provide T12 at 150 mm centres, A, = 754mm? am Mex O87 he My, 46.9010 bd® 1000 % 1857 Fron ble 67, for f,# 288 N/mm? the spanefectie depth modiiaton factor =137 mn Unntlag: ctve dipib ctu) 2280 4500 effective depth 185 Thus d= 185 mm is adequate Bending — Long Span Mey = Oy i? = 0.051 x 23.4 x 45? = 24.2 kN 22 REINFORCED CONCRETE DESIGN Since ths reinforcement for this span will aw & reduced effective depth, take 22 176—12= 16a mm, Therefore ape Moe = 2421108 DRT AE” O87 x 460% 164 = 369 mum? Jen Provide T10 at 200 mmm centres, A, = 393 mm?/m. q 1004, _ 100x393 0x39, o.18 bh” 3000x220 ‘which is greater than 0.13, the minimum for transverse steel ‘The arrangement of the reinforcement is showa in figure 8.11. 110 - 200 112-150 t 25m Figure 8.11 Simply supported sla sponning fn wo drections 8.6.2 Restrained Slab Spanning in Two Directions When the slabs have fixity at the supports and reinforcement is added to resist torsion and to prevent the comers of the slab from lifting then the maximum ‘moments per unit width are May ~ Bach? in direction of span le and Mgy = Bry ih? i direction of span ty where By, and Pyy are the moment coefficients given in table 3.15 of BS 8110 for the specified end conditions, and m= (1.4g, + I.6gy), the total ultimate load pet unit area ‘The slab is divided into middle and edge strips as shown in figure 8.12 and reins forcement is required in the middle strips to resist Mg, and M,y. The arrangement this reinforcement should take is ilustrated in figure 8.6. In the edge strips only nominal reinforcement is necessary, such that 1004,/bh = 0.13 for high yield steel or 0.24 for mild steel. DESIGN OF REINFORCED CONCRETE SLABS 213 at 7] fee 21 2 1g, [RI saiaate steve | Male sino ay 3 3 2 is S \ ' au L L eae fe) For span ig (0) For span fy Figure 8.12 Division of slab into mite and ede siripe In addition, torsion reinforcement is provided at discontinuous corners and it should (1) const of top and bottom mats each having bars in both destion of span 2) extend from the edges a minimum distance 4/5 G) ata comer winere the sab i discontinuous in both directions have an area of steel in each of the fous layers equal to three-quarters ofthe area required for the maximum mi-span moment. (A) ata corner where the slab is discontinuous in one direction only, have ‘an area of torsion reinforcement only half of that specified in rule 3. Torsion reinforcement is not, however, necessary at any corer where the slab is contiauous in both directions. Where fy /ly > 2, the slabs should be designed as spanning in one direction only Shear force coefficients are also given in BS 8110 for cases where torsion commer reinforcement is provided, and these are based on a simplified distribution ‘of load to supporting beams which may be used in preference to the distribution shown in figure 8,10. Example 8.6 Moments in a Continuous Two-way Slab The panel considered is an edge panel, as shown in. figure 8.13 and the uniformly distributed load, n= (1.4g, + 1.6q) = 10 KN/m?. The momen ores ato om eee of ble 1S of BS 110, 4. 60 ie 50 Positive moments at mid-span Mg, = Bgl? = 0.042 x 10 x 5? ~1os wai atin, 12 a REINFORCED CONCRETE DESIGN My = Boy? = 0.028 x 10x S* = 2OKN min direction fy Negative moments Support ad, Mi = 0.086 x 10 x5* = 14 Nm Supports ab and de, M, = 0.037 x 10.xS? = 9.3 kN m support @ 5 Discontinuous supportee eage =60m support y ‘suppert Figure 8.13 Continuous panel spanning in eo dtections ‘The moments calculated are for a metre width of slab. “The design of reinforcement to resist these moments would follow the usual procedure, Torsion reinforcement, according to rule 4 is required at comers b and 5. A check would also be required on the span-effective depth ratio of the slab. 8.7 Flat Slab Floors ‘A fat slab floor is reinforced conerete sib supported directly by concrete columns without the use of intermediary bears The slab may’ be oF constant thickness throughout or inthe eres ofthe column it may be thickened esa drop panel, The coluri may also be of constant section or it may be flared to forma Eolumn head or capital, These various forms of construction ae ilutrated in gure 8.14. “The drop panels ace effective in reducing the shearing stresies where the coli is able to punch through the sab, and they also provide an increased moment of resistance where the negstive moments are greatest. They are generally used with live Toads in excess of 7 KNim®, or thereabouts. DESIGN OF REINFORCED CONCRETE SLABS 2s mT LT, ae column need but poral ana cluren ro arep panel eo 9) Floor without column neos Figure 8.14 Drop pone and eotumn hese ‘The fat slab floor has many advantages over the besm and slab floor. The simplified formwork and the reduced storey heights make it more economical ‘Windows can extend up to the underside of the slab, and there are no beams 10 obstruct the light and the circulation of air. The ebstnce of sharp corners gives ‘reater fre resistance as there is less danger of the concrete spalling and exposing the reinforcement, Deflection requirements will generally govern slab thicknesses ‘which should not bo less than 125 mm. b————_ Postion ot mon mun Tapotive mamanta Position of maximum positive moments width of Ral! colurm step = fig with 90 areps whan drovs ere ised Figure 8.18. ar slab dteided mo stripe 216 REINFORCED CONCRETE DESIGN ‘The analysis of «flat slab structure may be carried out by dividing the structure into a series of equivalent fremes. The moments in these frames may be det (@)_@ method of frame analysis such as moment distribution, or the stiffness method on a computer or (b) 2 simplified method using the moment and shear coefficients of table 8.3 subject to the following requirements: () the lateral stability is not dependent on the slab.columa conneetions (Gi) the conditions for using the single load case described in section 8,1 are satisfied (Gil) there are at least three rows of panels of approximately equal span in the direction being considered, ‘Table 8.3 Simplified moment and shear coefficients for flat slabs Outer support First interior Interior Interor Col, Wall_First span support span support Moment -0.04FL* ~0,02FL +0.083FL* -0,063FL +0.071FL —0.05SFL Shear O4sF 04F — O6F - OsF Column mom, O.04KZ —— ~ 007FL— _oomrL ‘Check columa mementtranster capacity (see BS 8110). In thiscaleulation & isthe eifective span and F's the total ultimate lord on the Sab str ‘between columns The eifective span i the distance berween column contre lines ~ 2hc/3 where is the effective discter of the column or column feeds. Interior panels of the fat slab should be divided as show in figure 8.15 into coluran and middie srips. Drop panels should be ignozed if their smaller dimen- sion is less than the 3 of the smaller panel dimension /,. Ifa panel is not squate, strip widths in both directions are based on dy Moments determined from a structural analysis or the coefficients of table 8.3 are distributed between the strips as shown in table 8.6, Reinforcement designed to resist these slab moments may be detailed accord. ing tothe simplified rules for sibs, and satisfying normal spacing limits. This shouldbe spread across the respective strip, but stel to resist negative moments in columm strips should have eworthirds of the area located in the central 1/2 strip width. Ifthe color strip is narrower because of drops, the moments resisted by, the column and middle strips should be adjusted proportionally as iustated in example 87 DESIGN OF REINFORCED CONCRETE SLABS 217 Table 84 Division of moments between strips Columma stip Middle strip ‘Negative moment 15% 25% Positive moment 55% 45% When considering bending moments induced in the columns, the total moment acting at the ends of an upper and lower colurnn should be taken as SO per cent of the negative moment in the column strip for an interior column, and 90 per cent for an exterior column. These moments should be divided between the coluran lengths above and below the Noor in proportion to thet stiffness, Particular care is needed over the transfer of moments to edge columns. This is to ensure that there is adequate moment capacity within the slab adjacent to the column since moments will only be able to be transferred to the edge column by @ strip of slab considerably narrower than the normal internal panel column strip, width, ‘The reinforcement fora lat slab should generally be arranged according to the rules illustrated in figure 8.6. ‘An important featuze in the design of the slabs are the calculations for punch- ‘ng shear at the head of the columns and at the change in depth of the slab, if dcop panels are used. The design for shear should take the procedure desexibed in ‘the previous section on punching shear excopt that BS 8110 requires that the ‘design shear force be increased above the calculated value by 1S per cent for internal colurans and up to 40 per cent for edge columns to allow for the effects ‘of moment transfer If spans are tot approximately equal, reference should be ‘made to BS 8110. In this respect it can be advantageous to use mild stel in the design, as the esulting higher percentages of reinforcement will allow a corres- pondingly higher ultimate concrete shear stress. ‘The usual span-effective depth ratios may be used ifthe slabs have drop panels of widths at lest equal to one-third of the respective span, otherwise the ratios should be multiplied by s factor of 0.9. Reference should be made to codes of practice for further éetalled information describing the requirements for the analysis and design of flat slabs. Example 8.7 Design of a Flat Slab ‘The columns are at 6.5 m centres in each direction and the slab supports a live load of 5 kN/m?. The characteristic material strengths are fey = 20 N/mm? and J, = 250 Nfmm* for mild steel reinforcement. It is decided to use a floor slab as shown in figure 8.16 with 200 mm overall depth of slab, and drop panels 2.5 m square by 100 mm deep, The column heads ate to be made 1.4 m diameter. 218. REINFORCED CONCRETE DESIGN 25m sq droog a n= 200 ul & 5m cotunm centres eoen wey _| igure 8.16 Dead load Weight of slab = 0.2 24 x 65? = 203 kN Weight of drop = 0.1 x 24% 2.5? = 15 kN Total = 218 kN Live load Total = 5 x 6.5? =212KN ‘Therefore ultimate load on the floor F 4x 218+ 1.6% 212 = 645 KN per panel and equivalent distributed load n= S25 = 15.3 KN/n?, ‘The effective span L ear span ~ 2/3 2x =65— 6m A conctote cover of 25 mm has been allowed, and where there are two equal layers of reinforcement the effective depth has been taken as the mean depth of the (wo layers in calculating the reinforcement arcas, ‘The drop dimension is greater than one-thied of the panel dimension, therefore the column strip is taken as the width of the drop panel (2.5 m).. Singe the liv load is less than 1.2 x the dead load, and is not greater than 5 kN/m?, the single load case may be used. From tables 8.3 and 8.4 Bending (1) Cente of interior span forcement Positive moment = 0.071 FL = 0.071 x 645 x5.6= 257 KN m DESIGN OF REINFORCED CONCRETE SLABS 29 ‘The width of the middle strip is (6.5 ~ 2.5) = 4.m which is greater than half the panel dimension, therefore the proportion of this moment taken by the middle Strip is given by 4 45 x 2 055 o4s x 5 ‘Thus maidle strip positive moment = 0.5 « 287 = 142 KN m ‘The column strip positive moment = (1 — 0.55) x 257 = L16 KN im (@) For the middle strip _M__1a2xi0t bd? fey 4000 x 155? x30 From the leverarm curve. figane 75,4, = 0.98, therefore Age Mg NIP 5581 an? bortom tee Bifjhd 08x 2500550158 ‘Thus provide twenty-three R16 bars each way in the span, distributed evenly across the 4 m width of the middle stip. = 0.049 (6) The column strip moment will require 3622 mm? bottom steel which can be provided as nineteen R16 bars inthe span distributed evenly across the 2.5 m Width ofthe column strip, (2) Interior support ‘te monet = 655 Ft #0055 1645 156+ BAN sed ea de iste sip 20.25% 122% 921 x199=628\m sd sama sip “060.99 1K (2) For the middle str Mo 62x 108 Bfay ~ HO00% 155" x30 From the lever arm curve f= 0.95 theofore 62 x 10% DBT WIS 095K 155 Provide eighteen evenly spaced R12 bars as top ste ((b) For the column strip. M 138 x 10° Be foy 2500x255" x30 936 ram? 0.029 220 REINFORCED CONCRETE DESIGN From lever-arm curve /, = 0.95, therefore - 138 x 10¢ 0.87 x 250 x 0.95 x 255 Provide fourteen R16 bars es top steel, ten of these bars should be placed at approximately 125 mm centres within the central half of the column strip. ‘The bending reinforcement requitements are summarised in figure 8.17 619 mm? ¢ Column coumn 1B RI2-220 18 R12 -220 2316-175 a0ch wey (a) Middie strip 4.0m wide 16R15--125 ond 2500 1416-125 and 250ew SS i 19R 18-135, (b) Colurmn strip 25m wiee Figure 8.17 Devils of bending reinforcement Punching Shear (2) At the column head: perimeter u = 7 x diameter of column head = x 1400 = 4398 mm axle Shear force V=P— F144 n= 64s KIS = 621.5KN To allow for the effects of moment transfer, V is increased by 15 per cent, thus = MSW _ 115 621.510" = 0.64 Njram? ud 4398 3 255 DESIGN OF REINFORCED CONCRETE SLABS 2 which is less than 0.8-Vf., and 3 N/mm? (2) First critical perimeter is 15d'= 1.5 x 255 mm 383 mm from the column face thus the length of perimeter w= 4 (1400 + 2 x 383) = 8664 mm Ultimate shear force = 645 — (1.442 x 0.383) x 153 574.kN 1.15574 x 10% 8664 x 255, thus shear stress ¥ = 30 Nimm? By inspection from table 5.1 » vg/2 Span-effective depth ratios are limited to the values for a langed beam based con the shorter span but the web width used in determining the ratio from lable 6.6 may include the thickness of the two adjacent block-wall ‘A Teast SO per cant of the total tensile einforcement inthe span should con- tinue tothe suppor and be anchored. In some instances the slabs ae supported by steel beams and are designed as simply supported even though the topping may be continuous. Reirforcement should be provided over the supports to prevent cracking in these cases. It is recommended thet the aree of this top steel should not be let than one quarter of the area of see] requited inthe mile of the span and it should extend at least 0.15 ofthe clear span int the adjoining spans. A light reinforcing mesh in the topping lange can give added strength and urability tothe slab, particularly i thee sre concentested or moving loads, o iFeracking due to shrinkage of thermal movements is likely. An area of 0.12 per eat of the topping flange is recommended. Example 8.8 Design of a Ribbed Floor ‘The ribbed floor is constructed with permanent fibreplass moulds; it is continuous over soveral equal spans of 5.0 m.,The characteristic material stzengths are fou = 30 Nimun? and fy = 250'Nfinm?. Aan effective section as shown in figute 8.19 is to be tried. The characteristic dead load including eiF-weight and finishes s 4.5 kNm® and the characteristic live load is 2.5 kN/n? ‘The eafeulationsare for an interior span for which the moments and shears can bie determined by using the coefficients in table 8.1. 224 REINFORCED CONCRETE DESIGN 800, solid sab i 3-RI0 above ‘och rib 2-RI2 per rib t spon 50m sof = + rae a+ 200 Cross-section at midspan Figure Considering a 0.4 m width of floor as supported by each rib Uttimate load = 0.4 (1.4g4 + 1.69%) = 0.4 (14x45 +1.6x25) = 4.12kNim 12x 5.0 20.6 kN ultimate load on the span F Bending (2) At midspan: design as T-section BE= 0.063 FL. = 0.063 x 20.6 x 5.0= 6.49 KN m _M__ 649x108 Bd?foy 400 160? x30 From the lever-armn curve, figure 7.5, ly within the flange and = 0.021 .95. Thus the neutral axis lies . 6.49 « 108 O87F,2 087x250 0.95 x 160 = 197 mm* Provide two R12 bars in the ribs, A = 226 mm’. As DESIGN OF REINFORCED CONCRETE SLABS 25 (2) Atasupport: design asa rectangular section forthe solid slab ‘M=0.063 FL = 0.063 x 20.6 x 5.0" 6.49 KN masin (1) and Ag 197 mot as at midspan Provide three R1O bars in each 0.4 m width of slab, A, = 236 mm? (3) Atthe section where the ribs terminate: this Occurs 0.6 m from the centre line of the support and the moment may be hogging so that the 125 ram rias must provide the concrete ares required to develop the design ‘moment. The maximum moment of resistance of the concrete ribs is M, = 0.156 foybd? = 0,156 x 30 x 125 x 160 x 10-* =15.0kNm ‘which must be greater than the moment at this section, therefore com pression steel is not required, Spar- Effective Depth Ratio 6.49 x 10° Atmidspin joa? = $49%10" 9.63 midspan Mba 0 5 160% From table 6.7, with f, = 156 N/mm?, the modification factor = 2.0. For a ‘Tesection with web width 0.31 x flange width the basic ratio is 20.8 from table 66. limiting =208x20=41.6 effective depth span, 5000 effective depth 160 ‘Thus d= 160 mm is adequate, actual 2313 Shear ‘Maximum shear in the rib 0.6 m from the support cent line =05F 06 x4.12=05 «206 ~25=78KN Therefore shear stress = J, 0 0,39 N/mm? ba” 125% 160 1004, | 100x226 bd” 125x160 From table 5.1,%_ = 0.87 N/mm? ; therefore the section is adequate in shear, and since » 400 For and condition 2, top ond bottom tex 5 Palen, 0-89% 4000 17215 > ” 200 * Therefore: Column is slenagr about tha x x axis Figure 9.2. Braced column slendernesretios Short columns usually fail by crushing but a slender column is liable to fal by. buckling. The end moments on a slender colurnn cause it to deflect sideways and 1438 illustrated in figuee 9.3, The moment Negag causes a further lateral deflection and ifthe axial load (N) excee a critical value this deflection, and the additional moment become self-propagati until the column buckles. Euler derived the critical load for a pineended strut as, ‘thus bring into play an additional moment PEL Nea ‘The crushing load Ny, of a truly axially loaded column may be taken as Mur = 0.45 fiy Ae + 0.87 Agehy where 4¢ is the area of the concrete and ds: is the area of the longitudinal steel. ‘Values of Nert/Nuz and i/h have been calculated and plotted in figure 9.4 fora typical columa erossSection. COLUMN DESIGN Lees » tes | Figure 9.3. Slender column with erat deletion +0] | bucking crushing a fi F Figure 9.4 Column eure modes 248 REINFORCED CONCRETE DESIGN The ratio of NoMa inthe figure determines the type of failure of the coluann, With I fess than, say, 15 the load will probably couse crushing, Mes is much less than Neg. the load that causes buckling — and therefore a buckling failure will not occur. Ths is not true with higher values of hk and soa buckling failore is possible, depending on such factors the intial curvatuce ofthe column and the actual eccentricity ofthe load. When itis greater then 32 then Nae is Jess than Nye and in this case a buckling failure will occur forthe columa eon. sidered ‘The mode of failure of a column can be one of the following. (1) Material failure with negligible lateral deflection, which usually occurs with short columns but can also occur when theve are large end m ‘on a column with an intermediate slenderness ratio. (2) Material failure intensified by the lateral deflection and the additional moment. This type of failure is typical of intermediate columns. (8) Instability failure which occurs with slender columns and is liable to be preceded by excessive deflections. 9.3 Reinforcement Details ‘The rules governing the minimum and maximum amounts of reinforcement in a Toad bearing column are as follows. Longitudinal Stel (1) A minimum of four bac is required ina rectangular colurin and sx bats in circlae column ® 1004s 0.4 cot ® 1004s 6.0 in a vertically cast column Acct 1004s 5 8.0 in a horizontally cast column Aco but at laps 4204s > 10.0 for both ayes of cums cl where 4s is the total area of longitudinal steel and Ag is the cross. sectional area of the column. COLUMN DESIGN Das. Links Q) Minimum si than 6 men, (2) Maximum spacing = 12 x size of the smallest compression bar. (3) The links should be arranged so that every corner bar and slternate bar ‘or group in an outer layer of longitudinal stel is supported by a link using round the bar and having an included angle not seater thon (4) All other bars or groups not restrained by a link should be within 150 mm of a restrained bar. (5) In circular columns a circular link passing around a circular arrangement of longitudinal bars is adequate. No provision is mace in BS 8110 for eslculating the strength of a column which has helical einforcement in place of links. Ts form of spiral reinforcement is widely used in the USA and their codes take account of the added strength i gives toa column and its resistance to seismic Forces Figure 9.5 shows possible arrangements of reinforcing bas at the junction of ‘wo columns and a floor. In figure 9.Sa the reinforcemeat in the lower column is cranked so that it wil it within the smaller column above. The erank in the reine forcement should, if possible, commence above the soffit of the beam so that the i se | LN] com S bors Sib) + ae L 7 ey 0) © = 1/4 x size of the largest compression bat but not less Figure 9.5 Deets of splices in column reinforcement 26 REINFORCED CONCRETE DESIGN moment of resistance of the column is net reduced, For the same reason, the bart in the upper column should be the ones cranked when both columns are of the same size as in figure 9.Sb. Links should be provided at the points where the bars are cranked in order to resist buckling dus to horizontal components of force in the inclined lengths of bar. Separate dowel bars as in figure 9-5¢ may also be used {0 provide continuity between the two lengths of column, The column-bearn junction should be detailed so thet there is adequate space for both the columa, steel and the beam steel. Careful attention to detail on this point will greatly assist ‘he fixing of the steel during construction. 9.4 Design of Short Columns Short columns are divided into three categories according to the degree of eceen= tricity of the loading as described in the following sections. f 9.4.1 Short Braced Axially Loaded Columns ‘This type of column can occur in precast concrete construction when there is no ‘continuity between the members. Also it can be considered to occur when the columns support a symmetrical and very rigid structure ‘When the load is perfectly axial the ultimate axial resistance is N= 045 feyde + O87 fy Age where de Is the net area of the concrete and Aye fs the area of the longitudinal reinforcement, Ferivct conditions never exist and to allow for a small eccentricity the ultimate load should be caleulated from N= Od fegde + 0.75fy Age oe Fora rectangular column and to allow for the area of concrete displaced by the longitudinal reinforcement this equation may be modified to N= 04 feybh + Az(0-15 fy — OA feu) 2) Example 9.1 Axially Loaded Column Design the longitudinal reinforcement for a 300 mm square column which supports an axial load of 1700 KN at the ultimate limit state. The characteristic material strengthisave fy = 460 Nm? for the reinforcement and fey = 30 N/mm* for the concrete. From equation 9.2 1700. 10" = 0.4 x 30x 300? + 4ye(0.75 x 460 ~ 04 x 30) therefore = (1700 — 1080) 10° 333 Provide four T25 bars, area = 1960 mm? Ag = 1862 mm? COLUMN DESIGN ur 9.4.2 Short Braced Columns Supporting an Approximately Symmetrical Arrangement of Beams ‘The moments on these columns will be small and due primarily to unsymmetrical arrangements of the lie load. Provided the beam spans dotnot differ by more than 15 per cent of the longer, and the loading on the beams is uniformly distributed, the column may be designed to support the axial load only. The ultimate load that can be supported should then be taken 2s N=035 fey Ac + 087 fy Aye (2) ‘To take account of the ares of concrete displaced by the reinforcement the equa tion for a rectangular section may be written as. N=0.35 fosbh + (067 fy ~ 0.35 fex)Age 9.4.3 Short Colurins Resisting Moments and Axial Forces Tae area of longitudinal steel for these columns i determined by: (2) using design charts or constructing M-¥ interaction diagrams as in section 4.8 (2) a solution of the basic design equations. or (3) an approximate method. Design charts ore usually used for columns having a rectangular or eiteular cross. section and a symmetrical arrangement of reinforcement, but interaction diagrams can be constructed for any arrangement of cross-section as illustrated in examples 4.9 and 4.10. The besic equations or the approximate method can be used when, an unsymmetrical arrangement of reinforcement is zequired, or when the cross- section is non-tectangular as described in section 9.5 Whichever design method is used, a column should not be designed for a ‘moment less than 1 x éyyiq. WheTe eqyig Has the lesser valte of f/20 or 20 mm. This iste allow for tolerances in construction. The dimension A is the overall size ‘of the column cross-section in the plane of bending. (1) Design Charts and Interaction Diagrams The design ofa stetion subjected to bending plus axial load should be in accord ance with the principles described in section 4.8, which deals withthe analysis of {he cross-section, The basic equations derived for a rectengular section as shown in figure 9.6 and with a rectangular tess block are No Fe HF t hy (0.4) 45 feybS + feos + fay weal) ned) os he depth the ts lok = 9 ete of longitudinarelnforcerent inthe more highly compresied fa ‘Ay = the ares of reinforcement in the other face shy compressed Fase See = the stress in reinforcement Af 248 REINFORCED CONCRETE DESIGN COLUMN DESIGN 29 ous Example 9.2 Column Design Using Design Charts gee Figure 98 shows frame of a heavily loaded industrial structure for which the centre colurmns along line PQ are to be designed in this example. The frames at . as adm centres, are braved against fxieral arces, aid ‘suppont the following Moor load . » [Ao | Te dead load gx = 10 KN/m* 4 Neutrot a live load ae = 15 N/m? eee J 6 a ey section Strain Stress block igure 9.6 Column sein . foo 5 e 2 ara Ftcor & 1 ! | 2 4 j : ls5m ane Fae crs SOO FOO | | sm oe op 35m at Saeed a Ground corns figine 9.7 Coleman deg oar nt cH Seetion through the trame = the stress in reinforcement A,, negative when tensile ‘These equations are not suitable for direct solution und the design of a column ‘with symmetrical reinforcement in each face is best carried out using design charts sum to those published in Pact 3 of BS 8110. An example of one of these cl is shown in figure 9.7. Figure 9.8. Cotumss ie an tndestilstrecture 250 REINFORCED CONCRETE DESIGN Characteristic material strengths are fay * 30. N/mm? for the concrete and fy =460 N/mm? for the steel Maximum ultimate Toad at each floor = 4.0(1.4gy + 1.6q,) per metre length of bears =4(14x 1041.6 x 15) = 182 kN/m Minimum ultimate load at each floor = 4,0 x 1.08, = 4.0 x 10= 40KN per metre length of teem, Consider first the desiga of the centre colusnm at the underside (u.s.) ofthe firs floor. The critical arrangement of load whica will cause the meximaum moment in. ‘the column is shown in figure 9.93, Column loads sevond and third floors =2 x 152. 10/2 $20 kN fits floor = 152 x 6/2 + 40x 4/2 = 536 Column solf-weight, say 214 28 N= 2084 kN ‘Similar arrangements of load will give the axial loadin the column at the under (us) and top side (ts) of each floor lovel and these values of ere shown in table 9.3. ‘The moments on the column are not large and therefore equation 9.3 may be used for a preliminary sizing, Trying a 300 x 400 coluron N=035 feubh + 06TH Ay 2084 x 10° = 0.35 x 30 x 300 x 400 + 0.67 x 460 x Ase from which Ag = 2674 mm? and 100 Ag fh =2.23 ‘This provides an adequate cross-section and a 300 x 400 column isto be used. Column Moments ‘The loading arrengement and the substitute frame for determining the column moments at the first and second flooss are shown in figure 9.9. Meraber stiff kay 2g By 03X08 oy igs 1x6 4 RE = 107% 10 % Kea = 93204" 0.46 x 107% COLUMN DESIGN 251 Tq ITER 1064 (©) Critical loading orrangemam tor centre columns at 136 floor (0) Suostitute trome ond Fixed end moments Figue 9.9 therefore Ek = (0.71 41.0742 0,46) 10-* = 2.70 x 10-* and Feat 5 946 ot7 k Aistribotion factor forthe columa = Zt = 58 Fixed end moments at Bare column moment M= 0.17 (456 ~ $3) = 69 KN m 252 REINFORCED CONCRETE DESIGN AL the 3rd floor Bk=(0.71 + 1.07+0.46)107 24x10 and 046 column moment af= 256 (456 — $3) = 83 kN m oluren moment M= = ( ) ‘The areas of reinforcement in table 9.3 are determined by using the design chart of figure 2.7, Sections through the column are shown in figure 9.10. ‘Cover for the reinforcement is taken as SO mm and d/h = 320/400" 0.8. The ‘minimum area of reinforcement allowed in the section i given by Age = 0.004 bh = 0,004 x 300 x 400 = 480 mm? and the maximum area is Agg = 0.06 x 300 x 400 = 7200 mam? brat laps Age = 0.1 x 300 x 400 = 12000 mam? and the reinforcement provided is within these limits, Toble 9.3 Floor NOM NM 100d a, ) Nm) mF oh (om) ards sa OTL And ts 7 690 GAS dd 80 1536 2adus G10 009248 ses 148690129148 091080 4536 Inus 2088 690737 d 2520 1 Foundation 2098 «345 37.48— (0.72 1920 A smaller column section could have been used above the fist floor but this would have involved changes in formwork and also increased areas of reinforce: ‘meat, For simplicity in this example no reduction was taken in the total live lead although this is permitted with some structures, as shown by table 9.1 (it) Design Equations ‘The symmetrical arrangement of the reinforcement with Aj = A, is justifiable for COLUMN DESIGN 253 300_, Ra ct190 Raat 240 RB at 190 7 8] 472544716 4720 ane (a) Ground to () Ist to2nd Floor (ey 2ne to Ara Floor T3t Floor Figure 9.10. Colum sections in dvign example the columns of 2 building where the axial loads are the dominant forces and where ‘any moments due to the wind can be acting in either ditection. But some members are required (0 resist axial forces combined with large bending moments so that it {snot economical to have equal areas of steel in both faces, and in these cases the usual design charts cannot be applied. A rigorous design for a rectangular section as shown in figure 9.11 involves the following iterative procedure. () Selecta depth of neutral axis, x 2) Determine the steel strains ee and ¢, from the strain distribution. G) Determine the steel stresses fo. and f, from the equations relating to the stress-strain curve for the reinforcing bars (see section 4.1.2). (4) Taking moments about the centroid of Ay 1 Normal to the section © TERY stress Bleck Figue 911. Colum with en unsymmetreat erangement of reinforcement 254 REINFORCED CONCRETE DESIGN a (ert a) 0.002, therefore fic = 460/1.15 = 400 Nfram™ and fy = 400 N/mam?, tension, (4) Inequation 96 N (+3 =) = 0.45 feybs(d — 5/2) + fel (@~ a’) aM. 230x108 W ” Toon 09x #09 190 = 11am = 209 mm To allow for the area of concrete displaced fe becomes 400 — 0.45 fey = 400 — 0.45 x 30= 386 N/mm? and from equation 9.6 100 x 10° (209 + 140) 45 x 30x 300 « 171 (340.~ 171/2) 386 (340 ~ 80) 1069 ram? (5) From equation 9.7 N= 048 fyb +c A+ fey 00 = 978 mm? Thus Ay + Ay = 3047 mm? for x = 190 mm (6) Values of Aj + A, calculated for other depths of neutral axis, x are plotted in figure 9.13. From this igure the minimum area of reinforce: ment required occurs with x ~ 210 mm, Using this depth of neutral axis, steps 2 to S are repeated giving 256 REINFORCED CONCRETE DESIGN 790200 210-220-230 Depth of neutral os, = Figure 9.13. Deri chert for unaymmertet column example eq = & = 0.00217 > 0.002 10 Nima? and f (Alternatively separate values of Ai and A, 2s esloulated for each value of x could also have been plotted against x and their values read from the graph at x = 210 mm.) This area would be provided with A= theee T25 plus two T20 bars = 2098 mm? and Ag one 725 plus wo T20 bars = 1119. mm? With a symmewica arangement of reinforcement the azea from the design chart of figure 9.7 would be Az + 4, = 3200 mm? or 7 percent jgreater than the area with an unsymmetrial arrangement, and including no allowance for the area of concrete displaced by the steel. ‘These types of iterative caloulatons ae readily programmed for solution by a small microcomputer, which could find the optimum steel areas without the necessity of plotting a graph (ui) Simplified Design Method ‘Asan alternative to the previous rigorous method of design an approximate method may be used when the eccentricity of loading, eis not less than 2a) ‘The moment M and the axial force N are replaced by an increased moment -M, where myemen(t -a) 08) COLUMN DESIGN 257 plus a compressive force ‘V acting through the tensile steel 4, as shown in figure 9.14, Hence the design of the reinforcement is carried cut in two parts, (1) The member is designed as a doubly reinforced section to resist My act ing by itself. The equations for calculating the areas of reinforcement toresist M, are given in section 4.5 as My = 0.156 fesbd? +087f,A) (d= a’) 09) O87fyA, = 0.201 feybd + 087 f, At 19) (2) The area of caloulated in the first partis reduced by the amount, NiO8TI,. ‘This preliminary design method is probably most useful for non-rectangulat column seetions as shown in example 9.6, but the proceduce is first demonstrated with a reetangular cross-section in the following exarnple. iy: . Figure 9.16. Simplified desin mechost Example 9.4 Column Design by the Simplified Mechod Calculate the area of steel required in the 300 x 409 column of figure 9.12. N= 1100 kN, f= 230 KN m, fey = 30 N/mm? and f, = 460 Nimm? 230 x 108 (4 ) Eccentricity e= 23010 = 209 mm>( * dy Ye" 1100 108 MEN (1) Increased moment mesten( 8a) = 230+ 1100(200 — 60) 10" = 384 KN m ‘The ares of steel to resist this moment can be calculated using the 258 REINFORCED CONCRETE DESIGN formulae 9.9 and 9.10 for the design of a beam with compressive sein- forcement, that is My =0.156 feybd? 4 087 f, A, (d~ a) and 0.87 fyAy=0.201 feyhd + 0.875, AS therefore 384 x 10° = 0.156 x 30 x 300 x 340 + 0.87 x 460.4; (340 - 80) so that A= 2130 mm? and 0.87 x 4604, = 0.201 x 30 x 300 x 340 + 0.87 x 460 x 2130 Ag = 3667 mm? (2) Reducing this area by N/O.87f, 1100 x 10% 087 x 460 A, = 3667 — =919 mm* ‘This compares with 43 = 1950 mm? and A, = 1049 mm? with the design method of example 9.3. (To give a truer comparison the stress in the compressive reinforcement should have been modified to allow forthe ares of concrete displaced, as was done in example 9.3.) 9.4.4 Biaxial Bending of Short Columns For most columns. biaxial bending will not govesn the design. The loading patterns necessary to cause biaxial bending in a buiing's internal and edge colurans will not usualy cause large moments in both ditections. Cosner columns may have to resist signfeant bending about both axes, but the axial loads ae usually small and a design similar to the adjacent edge columns is generally adequate, ‘A design for biaxial bending based on vigorous analysis ofthe eross section and the strain and stress distributions woulé be according tothe fundamental principles of chapter 4, otherwise a simplified method as described in BS 8110 may be used. This method specifies that a column subjected to an ultimate toad W and moments, and Mf, about the xx and yy axes respectively may be designed for single axis bending But with an increased momen: and subject to the following conditions: ow Meat ‘then increased single axis design moment is COLUMN DESIGN 259, : i Mamet o Eom, e B My od, wy it Mec Me wit Bet them increased single axis design moment is My omy +0 why ‘The dimensions f’ and b" are defined in figure 9.15 and the coefficient fis speck fied in table 9.4, |? oy igure 9.18. Section with Baxi! bending ‘Table 9.4 Values of coefficient é for biaxial bending 0 01 02 03 04 05 B06 Bhs 6 100088 «0.77 068 «053042 0.30 200) REINFORCED CONCRETE DESIGN Example 9.5 Design of a Column for Biaxial Bending The column section shown in figure 9.16 is to be designed to resist an ultimate axial load of 1200 KN plus moments of Me = 75 KN m and Myy = 80 KN m. The characteristic material strengths are fey = 30 Nimm? and f, = 460 Nimm? eo). Figure 9.16. Bisa benng example % ” 00-60) M,{h' 1S fora braced column o > 10 for an unbraced column ‘There isa general restriction on the maximum slendeess of fa > 608" ‘and for an unbraced eoluran fo > 100% o ‘where fy is the clesr distance between end restraints and ‘ond fare respectively the smaller and larger dimensions of the column section, A slender columa must be designed for an additional moment caused by its curvature at ultimate conditions. The expressions given in BS 8110 for the ad COLUMN DESIGN 265 tional moments were derived by studying the moments’ curvature behaviour for a member subject to bending plus axial load, The equations for calculating the design moments are only applicable to columns of a rectangular or circular section and with symmetrical reinforcement ‘A slender column should be designed for an ultimate axial load (¥) plus an incceased moment given by Me M+ Mas =i Nay 14) where Mj is the initial moment in the column Maga is the moment caused by the deflection of the column 4, is the deflection of the column. ‘The deflection of a rectangular or circular column is given by 4, = Kh 3) The coefficient Ais caleulated from the equation 1 (ey = hfe! 0.16) 3000 () oo ‘with b” being generally the smaller dimension of the coluran section except when biasial bending is considered, In equation 9.15 the coefficient K isa reduction factor to allow for the fact, that the deflection must be less when there isa large proportion of the column, section in compression, The value for K is given by the equati <10 7 Naz ~ Moa Where Nyy is he ultimate axial load such that Nun = O45 foye +087 F,A sand Xgl is the axial load at balanced failure defined in section 4.8 and may be taken as Nya 0.25 fede In order to calculste K, the ates Age of the columns reinforcement must be known and hence a trial and etror approach is necessary, taking an initial conserva tive velue of X= 1.0. Values of K are also marked on the column design charts as shown in figure 9.7, 9.6.1 Braced Slender Column ‘Typical bending moment diagrams for a braced coluinn are shown in figure 9.19. ‘The maximum additional moment Mya Occurs near the mid-heightof the column ‘and at this location the initial moment fs taken a5 My=D4I, $0.6; >0.4M, 0.18) 266 REINFORCED CONCRETE DESIGN COLUMN DESIGN 207 mortent 25450 oo ‘ne 300. | aloe | 1 (a) section §21700%N1 Smalian Mtg = 70KNM moment Figure 9.19. arcced slender column dexisn moments Where Mf, isthe smaller initial end moment due to the design ultimate loads and M; is the corresponding larger initial end moment. For the usual ease sith double curvature of a braced column, Afy should be My = 1OKNM taken as negative and Mz as positive. From figure 9.19, the final desien moment should never be taken as less than Mh, MaMa My +Mygald a 2% in With ein ® A120 0¢ 20 mem (9) Axia! load and initio! moments. Figuce 9.20. Stender coluinn example Equations 9.14 to 9.18 can be used to calculate the edditional moment and ‘combined with the appropriate initial moment to design a slender column with single axis bending about elther axis, provided thatthe rato ofthe lengths of the sides i sways less thin 3 and the slendemness ratio jet for a column bent about its major axis does not exceed 20. Where these conditions do net apply and the ‘column is bent about its major axis, the effect of biaxial bending should be con- sidered with zero intial moment about the minor axisand additional moments bout both axes Slenderness ratios are dey = 6.15/03 = 22.5 > 15 ey[b = 8.00.45 = 17.8> 15 ‘Therefore the column is slender. As the column is bent in double curvature Mi = 10 KN m and My = 04M, $0.60; 04x -10+0,6%70=38 KN m and Mis therefore greater than 0.4.M, ‘The additional moment induced by deflection of the column is Example 9.7 Design of a Slender Columa A braced column of 300 450 crosssection resists atthe ultimate limit state an axial load of 1700 kN and end moments of 70 kN m and 10 kN m causing double curvature about the minor axis XX as shown in figure 9.20, The column's effects heights are gy = 6.75 m snd fey = 8.0 m and the characteristic material strengths = 30N/mm® and f, = 460 N/mm, 268, REINFORCED CONCRETE DESIGN 1190220 (830) ie 2200 (0p 29 KN ma with = 1.0 for the initial value. For the first iteration the total moment is sayete = 50x 3008 From the design chart of gute 9.7 100A,,/bh = 3.2 and K=065 7 ‘This new value for K is used to recalculate Myyy and hence Mj for the second it ion. The design chart is again used to determine 1004ye/bh and a new K is shown in table 9.5. The iterations are continued until the value of X in columns (I) and (5) of the table are in reasonable agreement, which in this design occurs after two iterations. So that the steel area required is Age =2.2bh/100 = 2.2 x 450 x 300/100 = 2970 mm? [As a check on the final value of K snterpolsted from the design chart Nya" 0.25 feud = 0.25 x 30 x 450 x 240 x 10-9 = 810 kN Nyp= 0.45 foybh + 087 Age = (0.45 x 30 x 450 x 300+ 0.87 x 460 x 2970) 107? = 301 kN Nar ~ Neat = 3011-1700 3011 ~ 810 08 COLUMN DESIGN 209 which agrees with the final value in column 5 of the table, Table 9.5 a @ @, @ 6) K Me MN? 100 Jon fe i? 42 32 065 mm 30 22 os 9.6.2 Unbraced Slender Columns ‘The sway of an unbraced structure causes lager additional moments in the ‘columns. Figure 9.2] shows how these additional moments ate added to the initial moments at the ends of the columns. The additional moment calculated from ‘equations 9.14 to 9.17 is added to the initial moment in the column at the end. ‘with the stiffer joint. At the other end of the column the edditional moment may toe reduced in proportion to the ratio ofthe stifinesses of the two joints. stiffer end pe ees Me + = ‘ M by 9 Moa ess stiff Lenina! ee end. ft this end Masa may be recuces in proportion 20 he jomnt stifinasses. at each eno. Figate 9.21 Undraced slender column desion moments 10 Foundations A building is generally composed ofa superstructure above the ground and a sub- Structure whieh fons the foundations below ground. The foundations transfer tnd spreed the loads from a structure's columasand walls nto the ground. The safe bearing capacity of the sol must act be exceeded otherwise excossive sete. ‘ment may occur, resulting in damage to the building and its service facilities, such as the water or gos mains. Foundation failure can also affect the overall stability ofa stcucture so that iti hable to slide, to lift vertieally or even overturn. ‘The earth under the foundations isthe most variable of all the materials that are considered in the design and construction of an cngineering structure. Under cone sill building the soil may vacy froma soft clay to ailanse rock. Also the nature and properties ofthe soil wll change with the seasons and the weather. For example Keuper Mar, aselacvefy common sili hard like a rock when dry but ‘when wet can change into an almost quid state. Tus important to have sn engineering survey made of the soll under a proposed structure 2 that variations inthe strata and the sol properties can be determined. Dill oles otal pits should be sunk, fv sz tests stich asthe penetration test performed and samples ofthe soil taken to be tested inthe laboratory. From the information gained it possible to recommend safe earth bearing pressures end, if necessary, calculate potsible settlements of the structure, Representatives values Of the safe Dearing pressures for typies soils are listed in table 10.1. Tn the design of foundations, the areas of the bases in eontact with the ground should be such thatthe sae bearing pressures will not be exceeded, Seitlement takes place during the working life ofthe structure, therefore the design loeding to be considered when calculating the bose areas shouldbe those that apply to the serviceability Timit state, and typical values that ean be taken are (1) dead plus imposed load = 100, +100 (2) dead plus wind load = 1.00, + LOM, (3) dead plus imposed plus wind load = 1,06, +080, +0.8M%, ‘These partial faciors of safety are suggested as itis highly unlikely that the max rvur imposed loed and the worst wind load will eccur simultaneously. 270 FOUNDATIONS. am ‘Table 10.1 Typical allowable bearing values Rock or soil “Typlal bearing value (n*) Massive igneous bedrock 10000 Sandstone 2000 10 4000 Shales and mudstone 00 to 2000, Gravel, sand and grave, compact 600 Medium dence sand 100 40300 Loose fine sand Jess than 100 Hird clay 300 10 600 Median clay 100 19300 Soft clay les than 75 +06, = i tam, Py, upiitt Figure 10.1. Uplift on fooune Where the foundation st subet to vr and horizontal a the following rule can be applied. vou 2 Sei Pye Py re were Y= he verlag Hf =the honsontat oad 2, =the aol vere! oad P= the lonablehorizntal toed init waihonzontliond wold ate coun of he pais ence of he round in conact with he ver fac the foundation pls the cin an ‘cohesion along the base. apse “ mm REINFORCED CONCRETE DESIGN ‘The calculations to determine the structual strength of the foundations, that isthe thickness ofthe bases and tho etess of reinforcement, should be based onthe loadings and the resultant ground pressures corresponding to the ultimate limit state With some structures, such es the type shown in figure 10.1, it may be necessary to check the possibility of uplift on tae foundations and the stability of the structure when itis subjected to lateral loads, To ensure adequate safety, the stability calculations should also be for the loading arrangements associated with the ultimate limit state. The critical loading arrangement is usualy the combina. ‘on of maximum lateral load with minimum dead load and no live load, that is 1.6 + 1.0G,. Minimum dead load can sometimes occur during erection when ‘many of the interior finishes and fixtures may act have been installed. For most design a linear distribution of soil pressure actos the base ofthe footing is assumed as shown in figure 10.22. This eesumption must be besed on {he soil acting as an elastic matesial and the footing having infinite rg. Tn fac, not only do most soils exhibit some plastic behaviour and all footings have a finite stiffness, bu aso tho distribution of soll pressure varies with time. The sctual distribution of bearing pressure at any mioment may take the [orm shown in figure 10.26 or c, depending on the type of sol and the stiffness of the base and the sinicture. But asthe behaviour of foundations involves many uncertaine ties regarcing the action ofthe ground andthe loading, its usually unrealste to consider an analysis which is 109 sophisticated Lt 4 mn (©) _Usitorm (©) Cohesive (©) Senay eisterbutron soi) ol Figure 10.2. Pressure dlurtburon under footings Foundations should be constructed so that the underside of the bases are below frost level. As the concrete is subjected te moro severe exposure conditions a larger nominal cover to the reinforcement is required. Its recommended that the imum cover should be not less than 75 mm when the conerete is cast against the ground, or less than $0 mm when the concrete is cast against a layer of blind: ing concrete. A concrete grade of at least fey = 35 N/mm? is required to meet the sctviceability requixements of BS 8110; ee table 6.1. FOUNDATIONS 273 10.1 Pad Footings ‘The footing fora single column may be made square in plan, but where there isa Jarge moment acting about one axis it may be more economical to have a rectangu- Jar base, Assuming there isa linear distribution the bearing pressures across the base will take one of the three forms shown in figure 10.3, according to the relative magni- tudes of the axial load Vand the moment M acting on the base. (1) In figure 10.3a there is no moment and the pressure is uniform N BD (2) With a moment M acting as shown, the pressures are given by the equation for sxial load plus bending. This is provided there is positive contact between the Pp a9.n* Eccantricitytey « Breadth of footing qn 2 a fea 5 so ee ar where © © © Figure 10.3. Ped footing ~ presue ditributions a @) REINFORCED CONCRETE DESIGN base and the ground along the complete length D of the footing, as in figure 10.3b so that whete 1s the second moment of ares of the base about the axis of bending and y is the distance from the exis to where the pressure is being calculated, Substituting for 7= BD? /12 and y = D2, the maximum pressure is x er oe, 10 ae (102 and the minimum pressor i N 6 : =X Mm 10: ae t- (103° ‘There is positive contact along the base ifp3 from equation 10 3 is postive When pressure pa just equals zero So that for p2 ‘0 always be positive, MIN — or the effective eccentricity, e— must never be greater than D/S. In these cases the eccentricity of loading is said to ie within the ‘middle third’ of the base. When the eccentricity, is greater than D/6 there is no longer a positive pressure along the length D and the pressure diagram is triangular as shown im figure 10.3c, Balancing the downward load and the upward pressures. deny = therefore 2 maximum pressure p= 2 Where ¥ is the length of positive contact. The centroid of the pressure diagram must coincide with the eveentricity of loading in order for the load and reaction to be equal and opposite. Thus FD 52 or ra) FOLINDATIONS 275 therefore inthis case af e> DIS ‘maximum pressure p= — 2 0.4) 3802-0 ‘Atypical arrangement of the reinforcement in u pad footing s shown in figure 10.4. With a square base the reinforcement to resist bending should be distributed uniformly across the full width ofthe footing, Fora rectangular base the eeinforce ‘neat in the short dizectioa should be distributed with a closer spacing inthe cegion tender and near the columa, to allow forthe feet thatthe transverse moments must be greater noarer the colurn. IF the footing should be subjected toa lrge over. turning moment so that there is only partial wearing. or if there isa cesutant uplift force, thea reinforcement may also be requied.in the top fae. Dowel or starter bars should extend from the footing into the column in order to provide continuity o the reinforcement, These dowels should be embedded snfo the footing and extend into the coluas a full lp length. Sometimes a 75 mm length ofthe coluran is constructed in the sare concrete pour as the oot {ng so as to form a ‘kicker or support forthe colum shutters. In these casos the dowel lap length should be measured from the tp of the kicker. Dewels Lap tengtr| Figure 10.8 Pat footing reinforcement deialls The critical sections through the base for checking shear, punching shear. bend ing and local bond are shown in figure 10.5. The shearing force and bending ‘moments are caused by the ultimate loads fram the column and the weight of the base should not be included in these calculations. 276 REINFORCED CONCRETE DESIGN ‘The thickness of the base is often goverened by the requirements for shear resistance. The principal steps in the design calculations are as follows. (1) Calculate the plan size of the footing using the permissible bearing pressure and the critical loading arrangement for the serviceability limit state, 2) Calculate the bearing pressures associated with the critical loading arrangement at the ultimate Kit state (3) Assume a suitable value for the thickness (h) and effective depth (¢), (Check that the shear stress at the column face is less than 5 Némm® 0108 Voy, whichever i the smaller. (4) Check the thickness for punching shear, assuming a probable value for the ultimate shear stress. ve, from table'$.1 (5) Determine the reinforcement required to resist bending. (6) Make a final check of the punching shear, having established precisely. (2) Check the shear stress at the critical seetions. (8) Where applicable. foundations and structure should be checked for ‘overall stability atthe ultimate limit state Reinforcement to resist bending in the bottom of the base should extend at least a full tension anchorage length beyond the critical seetion for bending, Punching shear perimater = column perimeter +8x 150 | shear Figure 10.5. Coicl sections for desien FOUNDATIONS. a Example 10.1 Design of a Pad Footing “The footing i quite to resist characteristic axial londs of 1000 KN dead and 2350 KN imposed from: a 400 mm square column, The safe bearing rassuce on the Soll is 200 &N/n? andthe characteristic material strengths are fey = 35 Nimm* and f, = 460 Nfmm Assume a footing weight of 150 KN so thatthe total dead load is 1150 KN. oe 10054 vo) (ot) Fe 28m se Figure 10.6 Pad footing exemple (3) For the Serviceability Lis Total design axial oad = 1.06, + LOO, = 1150 +350= 1500 kN 1500 tequited base 1500 as int quid base area = SEE = 7.5 nl Provide base 2.8 m squere, area = 7.8 m?, (®) For the Ultimate Limit State Column design axial load = 1.4Gy + 1.60, 1.4 x 1000 + 1.6 x 350 1960kN 1960 1960 = 250 N/m? oF Nf earth pressure (6) Assume a 600 mun thick footing and with the footing constructed on @ binding layer of concrete the minimum cover is taken as SO mm. Therefore take mean ‘effective depth d = $20 mm. 218 REINFORCED CONCRETE DESIGN AL th column face shear stess. = W{column perimeter x d) = 1960 x 10°/(1600 x 520) = 236 Nimm? <08-vieu (4) Punching Shear ~ see figure 10.5 critical perimeter = column perimeter #8 x 1.3 = 4.4004 12 x 520 = 7840 mm area within perimeter = (400+ 3a)? = (400 + 1560)? 3.84 x 106 mm? therefore punching shear force V = 250 (2.8? 3.84) = 1000kN v Perimeter xd = 1000 10% 7840 x 320 From table 5.1 this ultimate shear steess is not excessive, therefore h = 600mm will be suitabie, punching shear stress» = 0.25 Nfmm? (@) Bending Reinforcement — see figure 10.78 ‘At the column face which is the e section 1 M= (250% 2.8 x 1.2) x = 504 kN m for the concrete My = 0.156 foybd™ = 0,156 « 35 x 2800 x 5207 x 10 £4133 KN m > 504 Ma ORThz From the leverarm curve, figure 7.5, i, As 98. Therefore FOUNDATIONS 29 = __ 508 x10 087% 450(0.95 x 520) = 2550 min? Provide nine T20 bars at 330 mm centres, A, = 2830 mm?. Therefore 1004, _ 100 x 2830 bi 280% 600 Maxiseum spacing ~ 750 mm, Therefore the reinforcement provided meets the ‘requirements specified by the code for minimum area and maximurn bar spacing ina slab. =0.17>0.13 as required Vode 52m nem. 28m] o A, A (a) Bending tb) Shear Figure 10.7. Cricatsectons (Final Check of Punching Shear From table 5.1, for fey = 35 and 1004,/bd = 0.19 ultimate shear stress, ve = 0.4 N/mm? Punching shear stress was 0.25 Nfmm?, therefore a 600 mm thick pad is adequate. (@) Shear Stress ~ see igure 10.7b ‘At the critical section for shear, 1.0d from the eoluma face V = 250% 2.8 «0.68 = 476 KN ye Le Axe bd 3800x520 0.33 Nimm? < 04 ‘Therefore the section is adequate in shear. 280 REINFORCED CONCRETE DESIGN 10.2 Combined Footings Where two columns are close together it is sometimes necessary or convenient to combine their footings to form a continuous base. The dimensions of the footing should be chosen so thatthe resultant load passes through the centroid ofthe base atea, This may be assured to give a uniform bearing pressure under the footing and help to prevent differential settlement. For most structures the ratios of dead. and imposed loads carried by each column ace similae so that i the resultant pase through the centroid forthe serviceability limit state then this wil also be true — ‘or very nearly ~ atthe ultimate limit state, and hence in these cases uniform pressure distribution may be considered for both limit states. Cantroi of bose ong Fotultant aoe caincies | fe. — sreperoidat Rectangular Figure 10.8 Combined becor Te shape of the footing may be rectangular or trapezoidal as shown in figure 10.8, The trapezoidal base has the disadvantage of detalling and cutting verying lengths of reinforcing bars; itis used where there iss lange variation in the loads canted by the two colurans and there are limitations on the length of the footing, Sometimes in order to strengthen the base and economise on concrete a beam is incorporated between the two columns so that the base is designod as en inverted section, ‘The proportions of the footing depend on many factors If iti too long, there ‘wil be large longitudinal moments on the lengths projecting beyond the columns, ‘whereas a short base will have a Ierger span moment between the columns and the {grester width will cause large transverse moments. ‘The thickness ofthe Footing must be such that the shear stresses ure not excessive Example 10.2 Design of a Combined Footing ‘The footing supports two columns 300 mm square and 400 ram squere with characteristic deed snd imposed loads as shown in figure 10.9. The safe besring pressure is 300 kN/m? and the characteristic material strengths are fog = 35 N/mm? and f, = 460 N/mm? FOUNDATIONS 281 23m 12am Ge 1000KN 64= 840040 4+ 2004N Gus 200KN Teo Lo" i {| (CA A 116 at 180 o120 rensverse) 30059 400 54 Bow coum A | | | ose som 1950 46m Figure 10.9 Combined footing example (1) Base area: allow, say, 250 KN for the self-weight of the footing. At the serviceability limit state total load = 250-4 1000 + 200+ 1400 + 300 = 3150 kN area of base required = 325° = 19.$ m2 300 provide a rectangular base. 4.6 m x23 m, atea= 10.88 m?. (2) Resultant of column loads and centroid of base: taking moments about the line of the 400 mm square column 12003 7200+ 1700 ‘The base is centred on this position of the resultant of the column loads as shown in figure 10.9, £1.24 282 REINFORCED CONCRETE DESIGN (3) Bearing pressure at the ultimate limit state: at the ultimate limit state ‘column loads = 1.4 x 1000+ 1.6 x 200 + 1.4 x 1400+ 1.6 x 300 = 1720+ 2440 = 4160 kN therefore 4160 arth pressure = 3 7393kN mt (4) Assuming an 800 mm thick base with d = 740 mm for the longitudinal bars ‘and with a mean d = 730 mun for punching shear calculations: At the column Face shear sues vg = N/eolumn perimeter xd For 300 mm squaze columa ¥e = 1720 x 10°/(1200 x 730) = 147 Nimm? son LTE z ct Lead TY r | Z| Sheor Forcd’s a Benaing Moment Aenet upward pressure at the ltimete limit state Figure 10.11 Step footing with shearing force ond beni moments fr the strap beam 286 REINFORCED CONCRETE DESIGN and solve for R . The width 8 of the outer footing is then given by p= pD where p isthe safe bearing pressure. (3) Equste the vertical loads and reactions to determine the reuction Ry under the inner footing. Thus Rit Re tM Wt Wy t=O (10.6) and solve for Rz. The size § of the square inner footing is then given by s= ft ? (4) Check that the resultant ofall the loads on the footings passes through the centroid of the areas of the two bases. Ifthe resultant is too far away from the centroid then steps 1 to 4 must be repeated until there 1s adequate agreement. (6) Apply the loading associated with the ultimate limit state, Accordingly, revise equations 10.5 and 106 to determine the new values for Ry and Ry. Hence calculate the bearing pressure p, for this limit state, Itmay_ ‘be assumed that the bearing pressures for this case are also equel and ‘uniform, provided the ratios of dead load to imposed load are similar for both columns (6) Design the inner footing as a square base with bending in both direc- tions. (7) Design the outer footing asa base with bending in one direction and supported by the strap beam. (6) Design the strap beam. The maximum bending moment on the beer ‘occurs at the point of zero shear as shown in figure 10.11. The shear on ‘the beam is virtually constant, the slight decrease being eaused by the bbeam’s self-weight. The stirrups should be placed at a constant spacing ‘but they should extend into the footings over the supports so as to give ‘a monolithic foundation. The main tension steel is required at the top of the beam but reinforcement should also be provided in the bottom of the beam so as to cater for any differential settlement or downward loads on the bear, 10.4 Strip Footings ‘Strip footings ate used under walls or under a line of closely-spuced columas. Even ‘were it possible to have individual bases. itis often simpler and more economic 10 excavate and construct the formwork for a continuous base. ‘Ona sloping site the foundations should be constructed on a horizontal bearing and stepped where necessary. At the steps the footings should be lapped as shown in figure 10.12. FOUNDATIONS 2a7 Figure 10.12. Stepped footing om a sloping ste ‘The footings are analysed and designed as an inverted continuous beam subjected to the ground bearing pressures, With a thick rigid footing and a firm soil, « linear distribution of bearing pressure is considerec. If the columns re equally spaced and ‘equally loaded the pressure is uniformly distributed but if the Loading is not sym: metrical then the base is subjected to an eccentric Joad and the bearing pressure ‘varies as shown in figure 10.13. ee ipo? dit Wet coors =pppEr Uniform pronture on-untorm prassure Figure 10.12 Lineor presmere detibuion under a rigid 2p footing ‘The bearing pressures will not be linear when the footing is not very sigid and ‘the soil is soft and compressible. In these cases the bending: moment diagram ‘would be quite unlike that for a continuous beam with firmly held supports and the moments could be quite large, particularly ifthe loading is unsymmetrical. For ‘large foundation it may be necessary to have a more detailed investigation of the soil pressures under the base in order to determine the beriding moments and shearing Forces, Reinforcement is required in the bottom of the base to resist the transverse bending moments in addition to the relaforeement required for the longitudinal bending. Footings which support heavily lozded columns often require stirrups and bentup bars to resist the shearing forces Example 10.3 Design of a Strip Footing Design a strip footing to carry 400 mm square columns equally spaced at 3.5 m centres, On each columa the characteristic loads are 1000 KN dead and 350 kN 288 REINFORCED CONCRETE DESIGN 35m etes pom ews s1zo 18 orzo Figure 10.14 Strip footing wits bending renforcement r20'et 250 ctrs L imposed, The sae bearing pressuce is 200 kN/m* and the characteristic material strengths are fay = 35 Nfmm? and f, = 460 Nima? (1) Try 2 thickness of footing = 800 with d= 740 mm for the Longitudinal reinforcement. Assume # footing solf-weight = 45 kim, Width of Foting required = 1900-* SO*CS 235) 22.15 m Provide a stip footing 2.2 m wide. At the ultimate limit state column load = 1.4 x 1000 + 1.6 x 350= 1960 kN 1960 2aKas = 255 KN)? (2) Punching shear at column face Ye = Njcolumn perimeter xd = 1960 x 10/1600 x 740 = 1.7 Nfmmn? <08 Vfey By inspection, the nommal shear on a section at the column face will be significantly ess severe. @) Longitudinal Reinforcement Using the moment end shear coefficients for an equalspan continuous beam (Figure 3.10), for an interior span ° bearing pressure = ‘moment at the columns = 255 x 2.2 x 3.5% x 0.08 = 550kN im therefore 550» 108 a = 1955 mm 0.87 x 460 x 0.95 x 740 s FOUNDATIONS. 289, 800 13 «2200 820 = 2288 ram? 100 um Minimum 4, = 213% 100 Provide eight T20 at 300 mm centres, area = 2510 mm?, bottom steel. In the span. M= 255 «2.23.5 «0.07 = 481 kN m therefore 481x108 0.87 x 460 x 0.95 x 740 Provide eight 120 bars at 300 mm centres, area = 2510 mm, top steel. (4) Transverse Reinforcement Ly 4 = 1710 mm? Mt =255 x 12 = 155 kN mf 155 « 108 ‘ se SIE = 966 mi? fm O87 x 460 x 0.95 x 720 i 38h 800 Minimum A, 1040 mm SUSE" 5 0.13 x 1000 x 29 100 100. Provide 120 bars at 250 min centres. tea = 1260 mm?/m, botzom steel. (5) Normal Shvar wil govern asthe punching shear pecimeter is outside the footing 1.0d from column face ¥= 255 2.2(3.5 x 085 ~0.74 —0,2)*S53 kN (The coefficient of 0. is From figure 3.10.) shear sressy= 353% 10g 34 Nima? 2200740 Allowable ultimate shear stiess = 0.38 N/mnm?, from table 5.1 for Soy =35 Nim? 10.5 Raft Foundations ‘A raft foundation transmits the loud to the ground by means of a reinforced ‘concrete slab that is continuous over the base of the structure. The raft is able to span over any areas of weaker soil and it spreads the loads over a wide area Heavily loaded structures are often provided with one continuous base in pre forence to many closely spaced, separate footings. Also where settlement is @ problem, because of mining subsidence. itis common practice to use a raft founds: tion in conjunction with a more flexible superstructure. ‘The simplest type of raft isa flat slab of uniform thickness supporting the columns. Where punching shears are large the colurans may be provided with a ey 290 REINFORCED CONCRETE DESIGN pedestal at their base as shown in igure 10.15. The pedestal serves similar function tothe drop panel ina flat sab floor. Other, more heavily Toaded rat require the foundation tobe strengthened by bears to form a ribbed construc- tion, The beams may be downstandin, projecting below the slab or they may be upstanding as shown in the igure. Downstanding bears have the disadvantage of disturbing the ground below the sib and the excevsted trenches are often a nuisance during construction, while upstanding beams interrupt the clear floor area above the sab, To overcome this a sond ab is sometimes cast on top of the beams, s0 forming a cellular caft Rafts having 1 Uniform sab, and without strengthening beams, are generally analysed and designed as an inverted flat slab floor subjected to the earth bearing pressure, Wit regular column spacing and equal column loading, the coefficients tabulated in section 8 7 for fat slab floors are used to calculate the bending ‘moments inthe raft. The sab must be checked for punching shear around the solumas and around pedestals, if they are used Pesestol (@) Flot slob to» Dewnstona (1 upstona ‘beam po Figire 10.18. Roft foundations TETTTT Figure 10.16 oft foundation subject ro uplift A raft with strengthening beams is designed as an inverted beam and slab floor. The slab is designed to span in two directions where there are supporting beams on. all four sides. The beams are often subjected to high shearing forces which need to be resisted by a combination of stirrups and bent-up bars. Raft foundations which are below the level of the water table asin figure 10.16, should be checked to ensure that they are able to resist the uplift forces duo 10 the” hydrostatic pressure, This may be critical during construction before the weight of FOUNDATIONS 291 ‘the superstructure isin place, and it may be necessary to provide extra weight to the raft and lower the water table by pumping. An alternative method is to anchor the slab down with short tension piles. 10.6 Piled Foundations Piles ate used where the soil conditions are poor and itis uneconomical, or not possible, to provide adequate spread foundations. The piles must extend down 10 firm soil so that the load is eartied by ofther (1) end bearing. (2) friction, oF GG) combination of both end bearing ené friction. Concrete piles may be precast ‘nd driven into the ground, or they may be the cast i size type which are bored ‘or excavated. Asoils survey of a proposed sito should be carried out to determine the depth to firm soll and the properties ofthe soil. This information will provide 2 guide to the lengths of pile required and the probable safe load capacity of the piles. On & targe contract the safe loads are often determined from full-scale Toad tests on typical piles or groups of piles. With driven piles the safe load can be calculated from equations which relate the resistance of the pile to the measured set per blow and the driving force. ‘The load-carrying eapacity of a group of piles is not necessarily a multiple of that for a single pile — it is often considerably less. For a large group of closely, spaced friction ples the reduction can be of the order of one-third. In contrast, the load capacity of a group of end bearing piles on a thick stratum of rock or compact sand or gravel Is substantially the sum total ofthe resistance of each individual ple, Figure 10.17 shows the bulbs of pressure under piles and illustrates ‘why tho settlement of a group of piles is dependent on the soil properties at a greater depth. up of. \ dh ii VHh ‘single pila Figure 10.17 Bulbs of pressure 2 REINFORCED CONCRETE DESIGN ‘The minimum spacing of piles, centee 10 centre, should not be less than (1) the pile perimeter for friction piles, or (2) twice the least width of the pile ~ for ‘end bearing ples. Bored piles ate sometimes enlarged at their base so that they havea lager beating area or a greater resistance to uplift. A pile is designed as short column unless i s slender and the surrounding soil is to0 weak to provide restraint. Precast piles must also be designed to resist the bending moments caused by lifting and stacking, ond the head of the pile must be reinforced to withstand the impact of the driving hammer, Its very difficult if not impossible to determine the true distsibution of load ‘of apile group, therefore, in general itis more realistic to use methous that are simple but logical. A vertical load on a group of vertical piles with an axis of symmetry is considered to be distributed according to the following equation, ‘whichis similar in form to that for an eccentric load on # pad foundation: Paw Ng Mes ys Mew 5, no lee hy where Py is the axial load on an individual pile "is the vertical load on the pile group ‘isthe number of piles ae and éyy are the eccentricities of the load WN about the centroidal axes XX and YY of the pile group dng and Jyy are the second moments of area of the pile group about axes XX and YY are the distances ofthe individual pile from axes Y¥ and XX, respectively. Example 10-4 Loads ino Pile Group Determine the distribution between the individual piles of a 1000 kN vertical load acting at position A of the group of vertical piles shown in figure 10.18, Centfoid of the pile group: taking moments about line TT By, 2.042.043.0430 a 6 Xn and Yq 167m ‘where mis the number of piles. Therefore the eccentricities of the load about the XX and YY centroidal axis are eee = 2.0 ~ 1.67 = 033 m and ey = 0.2m Teg = Eyq? with respect to the centroldal axis XX = 2x 1.67? +2033? +2x 1,33 =9:33 similarly Iyy = Exq 23x 1.07 43x 1.0? 6.0 ‘Therefore FOUNDATIONS 293 nyo M a Mim yy Mey ee 6 933 * 6.0 7 = 166.7 35Ayy + 333% Therefore, substituting for Yq and xq Py = 166.7 — 35.4 1.67 +333 x 1.0 140.9 kN Py = 166.7 ~ 35.4% 1.67 -333 «1.0 74.3 kN Py = 166.7 + 35.40.33 +33.3 x 1.0% 211.7 KN Py = 186.7 # 35.4 0.33 33.3 x L0= 145.1 KN Py = 166.7 + 35.4 x 1,33 433.3 x 1L0= 247.1 KN Py = 106.7 + 35.41.33 ~33.3 x 1.0 180.5 kN Teta) 999.6 = 1000 kN 20m 1Om 204 REINFORCED CONCRETE DESIGN When a pile group is unsymmetrical about both co-ordinate axes itis necessary to consider the theory of bending about the principal axes which is dealt with in ‘most textbooks on strength of materials. In this cese the formulae for the pile loads are Py = N Ayy #Bxy where Aa NoxEx} ~ eyyExa¥n) t Exp Dyn — (Dewy)? and MeyyZ9% ~ eexE a) Zxalyh —(ZXwin)? Note thet egy isthe eccentri about the ¥¥ axis, asin figure 10.18. Piled foundations are sometimes required to resist horizontal forces in addition to the vertical loads. If the horizontal forces are small they can often be resisted by ‘the passive pressure of the soll against vertical ples, otherwie if the forces are not smal then raking piles must be provided as shown in figure 10.19 To determine the load in eac pil either a static method or a available. The static method is simply a graphical analysis using Bow's notation a3 illusteated in figute 10.19. This method assumes that the piles are pinned at their ends so that the induced loads are axil. The elastic method takes into sccount the 10.19 Force aking ples FOUNDATIONS 295 displacements and rotations ofthe piles which may be considered pinned or fixed at thelr ends, The pile foundation is analysed in a similar manner to a plane frame or space frame and available computer programs ate cpamenly used The pile cap must be rigid and eapsble of tensferring the ealumm lowds to the piles. It should have sufficient thicknes for anchorage of the columa dowels and the pil reinforcement, and it must be checked for punching shear diagons shear, bending and local bond. Piles are rately positioned atthe exaet locations shown on the drawings, therefore this must be allowed for when desigaing and detailing the pile cap. 11 Water-retaining Structures and Retaining Walls ‘The design of both of these types of structure is based on fundamental principles and analysis techniques which have been discussed in previous chapters. Because of their specialised nature, however, design is often governed by factors which may be regarded as secondary in normal reinforced conerete work. Such structures are relatively common, in one form or another, and hence justify coverage in some detail 11.1 Waterzetaining Structures ‘This category includes those which are required to contain. or exclude, any liquid. Since water is that most commonly involved, however, the rather loose tite is Frequently used to deseribe such siructures. Common structures of this type in- clude water towers and reservoirs, storage tanks including sewage disposal and treatment systems, and floors end walls of basements and other underground construction where iti necessary to prevent Ingress of groundwater. ‘As Its smportant to restrain cracking so that leakages do not take place the design is generally governed by the requirements of the servicenbility limit state, but stability considerations are particularly important and design must take caré- ful account of the construction methods to be used. British Standard Code of Practice BS 5337 offers guidance on che design and consiruetion ofthis category of structure, snd incorporates a limit state approach as an alternative to the elastic design methods which have been traditionally used, This code recommends values for design stress limits for this type of structure. Ie is anticipated however. hat this code willbe modified and replaced by BS 8007 during 1987 to take account of general changes which ave been intro- ‘duced in BS 8110. It is expected that the principal changes to the established procedures which ere described in this chapter willbe: 6 WATER-RETAINING STRUCTURES AND RETAINING WALLS 297 (a) Use of y= 1.4 for liquid loxds. (b) Use of higher grade concrete For durability (grade 40 for Reinforced Concrete). (6) Use of 0.2 mm maximum crack width unless the aesthetic appecrance is critical. It i also possible that the elastic design method will no longer be included, 11.1.1 Design and Construction Problems Toensute a watertight structure the concrete must be adequately reinforced in sections where tension may occur. For this reason is important to be able to envisage the deflected shapo of che structure and its individual elements, Tensile suresses due to say dizect tensile forces as well s those due to bending must be included in the design calculations. Continuity reinforcement to prevent exacking must be provided at corners and a member junctions. Tis reinforcement must extend wel beyond where its required 0 resist the tensile sitesses, particularly when these stresses occur oa the face in contact withthe liquig. The design should consider the cases where the structure is full of liquid and 4x0 when itis empty. The sructure when empty muse have the strength to with stand the active presures of any retained earth, Since the passive cesstance of the ‘acth is never certain to be acting, it should generally be ignored when designing for the structure full. Cracking may occur not only from flexure and shrinkage, but also fom subs sidence and in some areas earthquakes. Careful attention must thus be given to geological aspects ofa proposed ste and in particular to the possibilities of Aiferentilsetilement. It may sometimes be necessary to provide movernent joints to cater for this. in addition to expansion and contraction joints required to allow for thermal and shrinkage movements. Flexural eracking cen be controlled by careful design and detaling and is discussed in chapter 6, while shrinkage and thermal effects can be reduced considerably by careful attention to the construc. tion factrs listed in section 1.3. With athick section, the heat generated by hydeation cannot readily be diss- pited, and the resulting temperature rise in the body of the concrete may be considerable, In addition to the normal precautions, it may be necessary to use low-heat cements and to restrict the size of pours, for example. Experimental work bas shows that in walls and slabs grester than $00 mm in thickness, the outer 250 mm on each face may be regatded asthe surface zone and the remainder as core, Minimum reinforcement quantities to control thermal and shrinkage cracking should thas be based on a member thickness of $00 mim. The bottom surface zone for ground sls should be 100 mm, Temperature rises due to hydration must he averaged to allow for the core temperature The importance of good curing cannot be overamphasised, but iis important to remember that good compsection onsite i just as vital if tot more ential, in producing an impermeable concrete. Iris essential therefor, that the concrete ‘mix used is suffleienily workable to enable easy handling curing coastruction, to tendency to segregetion. A slightly higher water content combined with a higher cement content will help 1o produce a mix that is plastic But not sloppy. A 28, REINFORCED CONCRETE DESIGN xing time, and the use of naturel aggregates in preference to crushed stone is helpful, ‘Formwork must also be carefully constructed to avoid grout leakage at joints and the consequent areas of concrete vulnerable to water penetration. Particular ‘eare must also be given to the uso of formwverk ties. Through ties should not be ‘used, as these offer a potential leakage path. Flotation. particularly Guring construction, is a major problem in many under. ‘ground tanks and basements. To ovescome this it may be necessary to dewater the site, inerease the dead weight ef the structure, or provide for temporary flooding of the structure. In any case, the construetien sequence must be care- fully studied, and specified at the design stage. ‘When filling a tank or reservoir for the fist time, this should be done slowly. ‘This permits stress redistrioutions to occur, and this, coupled with creep effects, will greatly reduce the extent of cracking. 11.2 Joints in Water-retaining Structures All concrote structures must inevitably contain construction joints, although the need for joints to accommodate movement in water-etaining structures is governed by the likelihood of. and need to restrict, unacceptable cracking principally due to shrinkage end thermal mavements. Frequently it may be possible to combine the two categories of joint The principal characteristics of joints are that they must be watertight, and in the case of movement joints must aso permit the repeated required movement to toke place as freely as possible. Waterbars will generally be incorporated, either the surface type in slabs, or possibly the centre bulb type in walls. These must be effectively held in positon during concreting while allowing good compaction of the concreis to be still possible. Such waterbers must furthermore be able to accommodate anticipated movement without teering, and withstand considerable water pressures, ‘All movement joints must be sealed with z flexible compound which effectively is watertight and also prevents dust and grit from entering and thus blocking the joint. Jointing materials must fusthermore be durable under the conditions of ‘exposure to which they may be subjected 1.2.1 Construction Joints Construction joints cannot be avoided, and the aim must be to ensure reinforce- ‘ment continuity with good bonding between the new concrete and old, Such requitements. of course, apply to any reinforced concrete construction but especial care must be taken in this instance if leakage is to be avoided. Laitance must alway’ be removed to expose coarse aggregate and a sound irregular con- crete surface. The new concrete is then poured either directly against this surface, co alternatively a thin layer of grout may be applied before casting. If well con- structed, such joints should be completely watertight, Waterstops are not usually necessary; htowever. itis sometimes preferred to seal the joint on the water- i Surface as an additional precaution, WATER-RETAINING STRUCTURES AND RETAINING WALLS 299 Prepared Figure 111 Constraetion jovat Wherever possible the construction should be arranged so that the joints are either all horizontal or all vertieal. In some instances long lengths of walls or slab are constructed in alternate lengths as shown in figure 1.2, 0 that when the Intermediate pours are made later the clder concrete in the earlier pours will have already taken up some of the shrinkage movement, But on the other end some ‘engineers prefer to construct successive lengths, arguing that this will mean there is only one restrained edge and the other edge of the slab is free to contract with- cout cracking, The method adopted, in fact, often depends on the preference of the contractor who has to be responsibie for producing a watertight structure at an economic cost. —-— = construction tet are | ana Jomts pour | teraate Bay Consivetien Continuous Gey construction Figure 11.2. Consimieiton procedure for walls 11.2.2 Movement Zoints Movement joints are provided to reduce the likelihood of unwanted thermal or shrinkage stress concentrations. They ensure there is only a partially restrained condition during contraction of the immature concrete. 3K) REINFORCED CONCRETE DESIGN eee face contre voto [concrete tona — bowen ot Jom’ surtoce (@) wot! () ste Figure 11.3. Pare contacto fine Joints to accommodte contraction may be of two types, ‘partial or “eom- plete’, depending upon the extent of contraction anticipated and the degree of restraint that can be tolerated, “Partial” contraction joints are the simplest to provide, and consist of s deliberate ciscontinuity of the concrete, but without an intial gap. es shown i figure 11.3. Care must be taken te prevent excessive adhesion of the concrete surfaces when the second slab is east against the first, and a waterbar may be desirable asa precaution in addition to the joint sealer. Reinforcement is continuous across the joint to provide some shear transfer. but at the same time this reduces the effective freedom of movement of the adjacent concrete sections. Such joints thus provide only limited relief from constraint ‘nid they must always be sepsraied by atleast one movement joint with cormplete Aiscontinuity of concrete an sil ‘Anexample of 2 ‘complete’ contraction joint which fulfils this requirement is shown in figure 11.43, In this ease both steal and concrete are discontinuous, but {any shear must be transferred then a shear key is required as shown, In this type ‘of joint a waterbar is considered to be essential, although there is n0 initial gap between the concrete surfaces Where expansion of the concrete is considered possible, joints must be provided which permit this to take place freely without the development of compressive stresses in the concrete, Expansion joints must, therefore, not only provide com- plete discontinuity of eonerete end stel reinforcement, but also must have an {nitiel gap to accommodate such movement. Contraction can aso of course be ‘catered for by this type of joint. Figure 1] 4b shows a commen expansion joint detail, where in addition to a sealer and special woterstop, the joint is filled with a non-tbsorbent compressible filler. Shear can obviously not be tcansmitted by this joint, but if tis essential that provision for shear transfer be made, a special joint involving sliding concrete surfeces must be designed, Water pressure on the joint ‘materials may also cause problems if the gap is wide, and this must be considered. Occasionally. structure may be designed on the basis that one partis to be free to move relative to another, for exaraple ina citcular tank on a flat base, the walls may be designed as independent of the base, In such cases special sliding joints WATER-RETAINING STRUCTURES AND RETAININ GWALLS 3) Secler contra bulb ‘worerber Concrete bene broken, Sealer + ‘sneer key Grrequres) ae (polythene shoot) Blinding concrate ~~ Surtace wotertor () Complete eantecetion (eh Expansion joint joint in wal in tloor slab Figure 114 Complete movement fonts are sometimes used. The essential requirement is that the (wo concrete surfaces are absolutely plane and smooth and that bond fs broken between the surfaces such as by painting or the use of building paper, o:thata suitable flexible rubber pads sed, Figure 11.5 shows atypical detail for such a joint. which must always be ffectiely sealed Flexible Preoares*sieina! yeint surtoce igure 11S Py plea! sliding fine Derween slab anc well 302 REINFORCED CONCRETE DESIGN Provisior of Moyement Joints The need For movement joints will depend 10 a considerable extent on the nature of the structure and the usage to which itis put. For instance an elevated structure ‘may be subjected co few restraints, while an underground structure may be ‘massive and restrained. On the other hand, cemperature and moisture variations tay be greater in exposed structures than those which are buried. If warm liquids are involved. then this must be reflected in the provision of adequate joints ‘The iype of member, and construction sequence, will also be an important ‘consideration. Floor slabs will generally be cast on a separating layer of polythene ‘or some similar material laid on the blinding concrete, and in this case joints should the complete contraction or expansion joints. Altemnatively, the slab may be exst Girectly on to the blinding and reinforeed to Innit cracking on the basis of full int as described in chapter 1. Any joints then provided should be the “partial! contraction type, although these may not even be necessary ‘Walls may similarly be designed as fully restrained. or alternatively contraction joints provided at centres not exceeding about 7.5 m in reinforced concrete [Expansion joints must be provided if necessary. In some instances roofs may be separated from the walls by sliding joints, Ifthe roof isto be designed as unre- stained then great care oust be taken 1o minimise the restraints to thermal ‘movement during construction, If significant estzaints cannot be avoided, rein: forcement must he designed to limit the likely cracking. Where roof and wall are ‘monolittic, joints in the roof should correspond to those in the wall, which in turn may be related to those in the Moor slab, If design of a member is based oa the fully restrained condition, it is assumed shat eracking will be controlled by the reinforcement: therefore the critical steel ratio Fogg which is discussed in section 6.5 must be exceeded. The reinforcement 4 then detailed to limit the maximum likely crack width to the velues given in table 11.1, using appropriate velues of likely temporature change and concrete properties recommended by the code of practic. In this instance greatest benefit Js obtained fram closely spaced small diameter bars ‘Alternativaly. if proper movement joints are provided so that cracks are con- centrated at the joints, reinforcement may be designed on the basis of only partist restraint as indicated in table 11.2, but bar spacing should not exceed 300 mm. 11.3 Design Methods “The design of water-etatning structures maybe carried out using ether (1}a limit state desiga, or 2) an elastic design ‘A limit state design would be based on both the ultimate and servieebilty limit sates. using the methods described in the previous chaptets, As the restraint of cracking sof prime importance with these structures, she simpliigd rales for ‘ninimum steel areas and maximum spacing would no longer be adequate. It would te necessary to check the conezete strains and erack widths, using the methods described in chapters 1 aad 6, The calculations tend to be lengthy and depend on factors such a the degree of restraint, shrinkage and creep which are dificult to assess accurately WATER-RETAINING STRUCTURES AND RETAINING WALLS 303, ‘The elastic desig isthe traditional method which will probably continue to be used for many structures, Its relatively simple and easy to apply. Possibly it will bbe used in conjunction with limit state methods when thete ate special elreum: stances, such as when stability calculations are npcessary, or when the structure Ings an irregular layout, so that the critical loading patterns for the ultimate limit state should be considered. Even though a structure has been designed by the elastic method it nay til be necessary to exeuate the pose movement and crack widths. 3.1 Limit State Design ‘The application of limit state techniques to water-etaining structures is relatively new and the recommendations of BS 8110 are used subject to modifications contained in BS 5337, For the ultimate limit state the procedures followed sre exactly the same as for any other reinforced concrete structure. The partial factor of safety on imposed loading due to contained Liquid could be taken as 1.4 for strength calculations to reflect the degree of accuracy with which hydrostatic loading may be predicted. Calculations for the analysis of the structure subject to the most severe losd combinations wall proceed in the usual way but the relnforce- iment characteristic strength should not be taken to exceed 425 N/mm? Serviceability design will involve the classification of each member according to its exposure caiegory as described in table 11.1. For example, coofs will generally be in class A, while columns or walls may be in either class & or B according to circumstances. A wall or slab exposed to class A or B conditions on one face only should be treated as subject to class A or B on both faces if less than or equal te 225 mmm thick, or elass C on the remote face if thicker thaa 225 mm, External members not in contact with the liquid will le in lass C and eaa be designed using the criteria discussed in other chapters for normal reinforced concrete work, The maximum allowable crack widths are listed in teble 11.1 and these may be calculated using the methods given in sections 1.3 and chapter 6 and then checked or compliance. Although the expressions are of the same general form itis recom: mended tha the coefficens are modified to ge a lower probaly OF the limits ing exceed Design flexural surface crack width = where wg, _ BTbIa’ x) cme, ~ A Ath Ds {In these expressions. all variables are as in section 64.2 while fi the sevice set inthe winforcement anda negate ve fo Indicates thatthe section Thermal cracking is taken to have a maximum spacing fa ® fo 2 Sinan as in section 6.5 304 REINFORCED CONCRETE DESIGN ‘Table 11.1 Limit state method — limitations for reinforced concrete “Deemed to Satisfy" Maximum service Chass Exposure Maximum allowable stresses in surface crack width reinforcement Direct Plain. Deformed Flexure tension bars bars (om) (mm) (N/mm?) (N/mm*) Moist or Corrosive A atmosphere, alternate 0.1 on 85 100 ‘wetting and drying Continious, or B almost continuous D2 oz ns 130 liquid contact Not exposed to © moist or corrosive 03 03 125 140 conditions, or liquid with values of f/f for immature concrete suggested as: 1.0 plain round bars, 08 deformed iype 1 bars, 0.67 deformed type 2 bars. ‘The width ofa fully developed crack may be taken generally as Wmax ™ max (én #7 % — (100 » 10-*)] imax while in practice the drying shrinkage strain may be of the order of 100 10 ps, hence a simplified expression wma * fan 7 is suggsted as adequate, ‘Temperature rises due to hydration of the concrete (7°C) in walls may be ‘expected 10 be of the order of 20°C in winter and 30 °C in summer but should bo inezeased for high cement contents, rapid hardening cement, thick members or simber shutters in summer. Values for ground floor slabs may be about 5 °C less, ‘Additional seasonal temperature falls may be directly substituted into the above expression since the effects of concrete maturity are offset by a smaller ratio of tensile to bond strength and other effects, ‘Asan alternative to such calculations of crack widths, table 11.1 offers maxi- mum service stresses for the reinforcement and if these values can be shown to bbe satisfied it may be assumed that maximum likely crack widths in the mature WATER-RETAINING STRUCTURES AND RETAINING WALLS 303 concrete will be bolow the limiting values. Tis requires an elastic analysis of the member under working conditions and is illustrated in example 11.1 ‘The final detail of reinforcement to be provided must be co-ordineted with ‘the joint spacing arrangernent, This is a complicated proceduce since @ wide range ‘of possibilities exist, but some altemnative combinations based on control of thermal and shrinkage effects are suggested in table 11.2 with their most appro- priate applications. The critical steel ratio ray is given by f/f, a8 in section 65. It willbe seen that small-sized, closely spaced bars are best wher joint spacing fs largo; however, since erack spacing is related to bar ameter, large bars should ‘be used when closer joints are combined with less stel. Although table 11.2 offers a genecal guide, flexural effects may dominate and its recommended that ‘the engineer consults specialist iterature when undertaking a major design on this bas If a water setcining structure isto be constructed in prestrossed concrete, the cexposute classes of member are paired with the categories of prestressed members, discussed in chopter 12. Under working conditions class A memipers must have no flexural tensile stresses, class B may have tension but no visible eracking and class CC raust have surface crack widths not exceeding 0.2 mm. Once the appropriate category has been established. each member willbe designed in the way described in chapter 12. Example 11.1 Limit State Design of a Wateretaining Section The section shown in figure 11.6 is subject toa moment of 11.0 kN m under work- Ing loads which may be considered as purely hydrostatic. The moment acts so that there i tension inthe face adjacent tothe lqud. A grade 30 concrete and plain bars are specified with a class B exposure, 3-day tensile strength 1.3 N/mm? ‘Minimum cover= 40 mm. therefore assume d= 130 mm. (a) Ulamate Limite State Ultimate moment M= 1] x 16=17.6KNm _M_. _176%10' _ . go35 bd?f, 1000 x 1307 x30 therfore leveram factor = 098 therefore 17.6 «10% Ag = SNS 655 mn? O87 x 250% 0.95 «1 SsSmumt/n ‘This requires 12 mm bars at 150 mm centtes farea = 754 mm? /). (b) Serviceability Limit State Flexural cracking: check service stress in reinforcement assuming cracked section. using an estimated &, = 26 kNémnmn? 0 WATER-RETAINING STRUCTURES AND RETAINING WALLS. : REINFORCED CONCRETE DESIGN 306, e z B 2 g : 2g = = 5 ga s g 2 : a2 g v 3 2 G2 2 8 3 2 22 4 68 Shy = 2 BB & = ea i* = ae $4 og 2 as _ s 2 gt « 4 ae |X ice 2 op? oS 5 2 alg 2 2,5: 4 gp 8 BE gas fo G2 ..8 3 3 ole So ee, ARE @ £8 Z z2 i = SSE ou 2 28 z sige S8 828 ba g : 3 ro 2 u ke 823 ¢ 5 * Tee 8 2 2 a fog 8 ae 3 a2 ra g Ba 53 Ey = 8 aed gé = £8 e:8 ge 2 ( 01 K001~ $e. )ae04 os eo me ua OO oy ov Sues owurdie 20 vonzenues yyun Aressv0e0—(z'0>) ‘aajdacs aA ‘unjoes9 ou pu porunumam — rele Muy eines oy aeneds wefan uosnde 10 woenu09 Syeeoo sea st oy teav Bu . we ayes wpm Asesszooy (220) m patjentos tpt pasumauen Mag ‘omscr-| efupeds oy Gnteds assy funds uot » . supoeis oUS8r YOR, my = PH Parorued TEA seyieus poy =e qtupeds ag Auoeds of seiopoH au gt ye worsen | wondo uni 9) sietdo3 10 gc vorinsos ‘ng sieg Tease uonsniuos ehiea snonuinuconisg —Z Aupeds 80)9 (uoyoysoeup ov) wsieqsug 2 suo T_vononnsuor smonsnuey und nT, . mee wemasojaay ———spnap of wousioy oem 80g poneny wordy ‘1398 aBeQUUS pu [PUNFEY 10} SUONEUTGuIOD yUaMIsD:OJtHEr PUR IU! POISS#BNG Z-TT AGEL 180 x 1000, xt KEINFORCED CONCRETE DESIGN hence for summer concreting and if a. = 10x 10-® us/°C nee #1589 12 30-4 302018 me Thies iio 02 and hm fects maybe onde deal Sorry he roped efosmen (6) Reinforcement and Joint Detailing Lee !3 000s R330 = 936 mm? Critical ratio rai Designed main reinforcement isles than rea; and Would require close joint spacing as options 4 or 5 (table 1.2). For class B construction it may be more appropriate to inerease steel marginally to 12 mm bers at 120 mm centres {Grea 943 mm®/m) and then adopt option 2 with partial contraction joints at 715 m centres. The reinforcement may be provided in one layer with, say, 12 mm. 1 128 mm centres distribution steal trenstersoly. 11.3.2 Blastie Design ‘This method is based on working loads, and permissible stresses in the concrete and steel which are considered 0 be acting within the elastic cange. Hence the design assumes a triangular stress block as analysed in section 4.10. Suitable permissible stresses are shown in tables 11.3, 11.4 and 11.5. The ratio (a4) of the ‘modulus of elasticity of steel to that of concrete is taken as 15. Calculations are performed on the basis of two criteria: strength, and resistance to cracking. Strength calculations assume a cracked section. Low permissible steel stresses are specified in order to limit the width of cracks and thus reduce the chance of leakage and corrosion of the reinforcement, ‘The analysis for resistance to cracking assumes a limiting tensile stress in the concrete and is based on an uncracked concrete section. The governing factor in such an analysis is inevitably the permissible tensile stress in the concrete, with the steel and concrete stresses being related by the compatibility of strains across, the section. ‘The calculations for cracking resistance must be carried out for (2) the face in contact with the liquid for sections thicker than 225 mm (2) both faces for sections of 225 mm or less in thickness Sections thicker than 225 mm are designed for strength only at faces remote from the liquid, Design bythe elastic method, therefore, consists of designing for strength, ‘checking eracking resistance, and increasing the member size or the area of rein forcement, if necessary. Using the oxpressions derived in section 4.10 for a triangulat stress block with cracked and uncracked section, the provecurs for calculating the area of reinforcement for e section sa follows. WATER-RETAINING STRUCTURES AND RETAINING WALLS 309 Table 11.3 Blast method ~ strength calculations, permissible concrete stresses Permissible concrete stresses (N/anm®) Concrete Compression ‘Shear Bond rade —— ss Direct Due to Average Local bending 30 337 11.0 087 10 1.49 25 695 81s 077 09 136 Average (anchorage) bond stress: (2) 14 for deformed bars; (0) x 0.7 for heslzontal bars ia Aiect tension. (@) Strength Calculations (1) Determine the depth of the neutral axis from a ay 1+ fe Oefee > The steses ate the petmissible values and ay = 15, (2) Chock the moment of resistance of the concrete from Med bxfor («-2) (2) (3) Calculate the area of tension steel from M Af > a3) ta (u-3 Table 114 Elastic method — strength calculations, permissible steel stresses Posmnissible stress (Nima?) Condition Chass of exposure iain bars Deformed bars Direct tension Flexural tension ft as 100 nd Shear B 1s 130 Compression AandB 125 140 a0 REINFORCED CONCRETE DESIGN Table 11.5 Elastic method — cracking calculations, permissible stresses Permissible concrete stresses (N/mm?) Shear Concrete Tension v rade — " Direct Due to bending 7 30 1a 2.02 25 131 134 (b) Cracking Resistance (1) Determine a depth of neutral axis fer the uncracked section from 2(We = I)rd yee a1.) where r= Agfbi. (2) Check that the tensile stress, fe nthe concrete does not exceed the permissible value from table 11.5. The stress is calculated from wna [( 2) (er s)) ‘The depth of the section or the steel area must be increased if the per- missible stress is exceeded, ‘Where the section has to resist direct tension, as for example ina circular tank, the resistence of the uneracked section is given by fale + Aste) where Ag is the area of concrete in tension. Therefore fora vectangule seetion and allowing for the area of concrete displaced by the reinforcement T= farbh + forsee ~1) When the direct tension is combined with bending, a rigorous enalysis of the section may be carried out but for many cases ths is not justified. Quite often with rectangular tanks, only small tensile forces ere associated with the bonding. Conversely, when a circular tank has to resist lage hoop tensions usully the bending moments have only secondary effects, If the shear stresses in the concrete should excead the permissible values given. in table 11.3 then shear einforcement must be provided. In an clastic analysis the shear strength of the concrete isnot included inthe formulae for celeulting the stirrup reinforcement, The area, Aye and spacing. of the stirrups given by fev ORE WATER-RETAINING STRUCTURES AND RETAINING WALLS 311 ‘where V = the shear force due to the working loads fa = the steel stress from table 11.4 2) = the lever arm of the resistance moment. Example 11.2 Blast Design of a Waterretaining Section ‘The seetion shown previously in figure 11.6 and designed by the ultimate method in example 11.1 fs subject toa moment of 11,0 kN ms under working loads, The moment acts so that there is tension in the face adjacent to the liquid. A grade 30 conerete and plain bars are specified with a cluss B exposure. (@ Strength Coleulations Depth of neutral axis x ie sx roo x766x11 (130 8) 10-6 Lx 10° ae 766 t(e-2) 113 (130- 8 Provide twelve RIO bars, area = 942 mm? =916 mm* (b) Check Cracking A 9 tn 000% 180 h42@,— Ind er Dri? 180+ 28 0.00523 «130 3280. 09973 2130 997 sam 28 x 0.00523 +2 wr 58) oor 9] 823 + 14x 0,00823 x 223 (130 — 227) 3 00523 therefore 3 M108 =f 1 180 [ 312 REINFORCED CONCRETE DESIGN from which fig = 1.90 N/mm®, which is less than the permissible tensile stress of 2.02 Nfmm? from table 11.5. therefore the section is adequate and extra steel is not required, 11.4 Reinforcement Details Reinforcement should normally be placed near the surface of the concrete but the ‘minimum cover should net be lest than 40 mm. For exposures A and B, a8 described in table 11.1, the mininwum steel areas in each of two directions at right angles are normally taken as re OF 2/3 reg 26cond {ng to joint arrangement, (2) In members up to 200 mm thick these areas of reinforcement should be in one layer. (2) In mernbers over 200 mma thick the reinforcement should be equally divided so that there is layer in each face (3) In merabors greater than SOO mm thick # should be taken as 500 mm, (4) Requirement (3) may be reduced for ground slabs (see BS 5337) 11 will be seen ftom the preveding sections, that under service conditions the reinforcement must be acting at stresses below those normally existing in rein- Forced concrote members. This reduves the advantages of increased strengths ‘usually associated with high-yield steels. It will be noted, however, that for “nominal ceinforcement the minimum quantity is considerably reduced if defor sed bars ere used, because of their improved bond characteristics. The cheive ‘between high-yield and mild steel reinforcement is, therefore, not well defined and often a matter of personal preference of the engineer. Exarale 11.3 Design of a Water-retaining Structure by Elastic Method A cross-section of the tank to be designed is shown in figure 11.7. The floor slab Of the tank spans on to supporting beams at Band C. A grade 30 concrete and plain mild steel bats are to be used. (1 m? of water weighs 9.81 kN/m?.) 3 day tensile strength fog = 1.3 Némman™ For the walls. For the slab i = 300 mm and d= 250mm 200 mm and 30mm WATER-RETAINING STRUCTURES AND RETAINING WALLS 313 200 nee 12-100 g a Rie-130 os §, a ye om 45m 10m igure 11.7. Woter tn showing location of main reinforcement (@) Walls ‘Since the water is on the tension side both strength and cracking must be designed for 1 base of wall = 981% 2.0= 19.62 N/m? For the effective span of the cantilever and considering a 1 m length of wall, ‘water pressur (1) Strength Neutral axis depth = he Ochee 150 15 isxt M 6x10 2 =88.4mm it Provide R12 bars at 100 mm centres, area = 1130 mm? Therefore aM REINFORCED CONCRETE DESIGN 1130 Bh 200x 1000 ‘which is greater than the minimum ofray/2 required in each face for continuous construction 2) Cracking = 0.00565 B42, = Nr 2a — Dt? 200 + 28 x 0.00565 x 150 sO OOO 103.7 min 28 x0,00565 +2 pen [O=2) @-0 (yx Me fab [ee +@,-vr 2 (0-¥)] ‘therefore 146 10 fe 1% 200 [983 +14 oss « #83 (150 2 a3 a froin which fg = 2.00 N/mar®, whichis less than the permissible tensile siress of 2.02 N/mm? (table 11,5) therefore extra ste! is not requized Minimum steel required in euch face = rel? 13 13 y 1000 x 200 = 520 mm? 3502 * am Provide R10 at 150 mm centres, sea = $23 mm®, This steel should be provided at right angles to the main steel and also cach way i the opposite Face (b) Floors ‘The bending-moment diagram is showa dian on the tension side of the structure in Tigure 11-8. As can be seen the floor must be Gesigned for strength plus cracking over the beams, but at mid-span between the beams the water is on the compression side of the slab, therefore strength only need be considered, since h > Weight oFslab + water = 0.3 % 24+ 9.81 x 2= 26.8 Nim? Woight of wall =23%02%2 = 1LOKNIm Considering 1 m breadth of slab: at the supporting beam M= 146+ 11.0(.0~ 0.1) +268 x 087/2.=33.1 KN m hogging pan between B and C = 26.8. 4.52/8 — 33.1 = 34.7 KN m sagging WATER-RETAINING STRUCTURES AND RETAINING WALLS 315 ‘Midsspan: design for strength ony = 1473 mm set 34.7 x 108 Hh raya 7 soem" uus( 230 3 Provide R16 barsat 130 mm centres bottom steel, area = 1546 mm". oa 3 126] has 77 Figure 118 Bentngsmoraent diagrams (8 m) ‘AL the support beam: design for strength and cracking M=33.1 kN ‘Therefore R16 bars at 130 mm centres top steel will also resist this moment, Check cracking Ay 1546 tn” 1000 «360 y= 300428 x 000515 28x 0.00515 +2 38108 =f 10? x 300 [83 +14 ogosis e885 3 13 100515 156.7 mm from which fey = 1.93 N/onm? which is less than 2.02 N/mm? Minimum steel required in each face = 0.0026 x 1000 x 300 = 780 mm? Provide R12 bars.at 140 mm centees at right angles to the main steel and at least this area each way in the compression faces of the sab, Direct tensle force on the slab due to pressure on the walls is given by 316 REINFORCED CONCRETE DESIGN $x 19.62 x 2.0= 19.62 kim Thus. with RUG bars at 130 mm centres (1546 mm*) and RIP at 140 centres (798 unm? in the compression Face direct tensile stress = 12.62 « 10° _ Bh (ee — Ay 19.62. 10° * je0% 300+ 141546 + 798) = 0.05 N/mm? For the tensile stresses actual bending stress, _actual direct stess__ 1.93, 0.06 permissible bending stress permissible direct stress 202” 1.44 = 0.955 +0.042 =0.997< 1.0 11.5 Retaining Walls Such walls are usually required to resist a combination of earth and hydrestatic Joadings. The fundamental requirement is that the wall i eapable of holding the retained material in place without undue movernent arising from deflection, over- turning or sliding 115.1 Types of Retaining Wall Concrete reteining walls may be considered in terms of three basic categories: (1) gravity, (2) counterfort, and (2) cantilever. Within these groups many common ‘variations exist, for example cantilever walls may have additional supporting ties into the retained material. The structural action of each type is fundamentally different, but the techniques used in analysis. design and detailing are those normally used for concrete structures. (8) Graviey Watts ‘These are usually constructed of mass concrete, with reinforcement included in ‘the faces to restrict thermal and shrinkage cracking. As illustrated in figure 11.9, reliant is placed on self-weight to satisfy stability requirements, both in respect (of overturning and sliding. Its generally taken asa requirement that under working conditions the result- ant of the self-weight and overturning forces must lie within the middle third at {te interface of the base and soil. This ensures that uplift is avoided at this inter- face, as described in section 10.1, Friction effects which resist sliding are thus ‘maintained across the entire base Bending, shear. and deflections of such walls are usually insignificant in view of the large effective depth of the section, Distribution steel to control thermal crack- WATER RETAINING STRUCTURES AND RETAINING WALLS ing is necessary, however, and great care must be taken to reduce hydration temperatures by mix design, construction procedure and curing techniques, Total soul tore TES Friction tores , esuitant soit ores wargnt Figure 11.9. Grovty wal (4) Counterfort Waits This type of construction will probably be used where the overall height of wall is 100 large to be consinueted economically either in mass concrete or as a cantilever. ‘The basis of design of counterfort walls is that the earth prossures act on a thin wall which spans horizontally between the massive counterforts (figure 11.10), ‘These must be sufficiently large to provide the necessary dead load for stability requirements, possibly with the aid of the weight of backfill on an enlerged base The counterforts must be designed with reinforcement to act as cantilevers 10 resist the eonsiderable bending moments that are concentrated st these points. ‘The spacing of counterforts will be governed by the above factors, coupled with the need to maintain a satisfactory span-depth ratio on the wall slab, which :must be designed for bending as a continuous slab. The advantage of this form of ‘construction is that the volume of concrete involved is considerably reduced, thereby removing many of the problems of large pours, and reducing the quanti= ties of excavation. Balanced against this must be considered the generally increased shuttering complication and the probable need for increased reinforcement, (iit) Cantilever Walls ‘These are designed as vertical cantilevers spenning from a large rigid base which often relies on the weight of backfill on the base to provide stability. Two forms of this construction are illustrated in figure 11.11. In both cases, stability calcula {dons follow similar procedures to those for gravity walls to ensure that the resultant force lies within the middle third of the base and that overturning and sliding requirements are met. a8, REINFORCED CONCRETE DESIGN Welt countertort |L_Sean Grass = Section Plan igure 11.10 Counterfore welt a Figure 1111 Canitever welts 11.5.2 Analysis and Design The design of retaining walls may be spit into three fundamental stages: (1) Stability analysis — ultimate limit stete, (2) Bearing pressure analysis — service: ability lit state, and (3) Member design and detailing — ultimate and serviceability limit states, (1) Stability Analysis Under the action of the loads corresponding to the ultimate limit state, a retaining wall must be stable in terms of resistance to overturning and sliding. This is demon: strated by the simple case of a gravity wall as shown in figure 11.12. ‘The eritical conditions for stability are when 2 maximum horizontal force acts with 2 minimum vertical load, To guard aguinst a stability failure, itis usual to WATER-RETAINING STRUCTURES AND RETAINING WALLS 319 Se Sy ees. Tea Tees pressure pressure Figure 11.12 Forces and pressures on a gpsity welt apply conservative factors of safety to the force and loads. The values given in table 2.2 are appropriate to strength calculations but ¢ value of 4 ~ 1.6 or higher should be used for stability calculations. If this force is predominantly hydrostatic and well defined, factor of 1.4 may. be used, A partial factor of safety of 7, = 1.0 is usually applied to the dead load G " For resistence to overturning, moments would normaly be taken about the toe of the base, point A on figure 11.12, thus the requirement is that LOG aly a8) Resistance to sliding is provided by friction between the underside of the base and the ground, and thus is also related to total self weight Gy. Resistance provided by the passive earth pressure on the front face of the base may make some contribu: tion, but since this material is often dacktilled against the face, this resistance cannot be guaranteed and is usually ignored. Thus, if the coefficient of friction betsveen base and soll isu. the total friction force will be given by MG, for the length of wall of weight Gy: 2nd the requirement is that LOW, 2 bi ay where Hi, isthe horizontal force on this length of wall If this riterion is not met, 2 heel beam may be used, and the force due to the sive earth peessure over the face area of the heel may be included in resisting the sliding force, The partial load factor yy on the heel bearn force should be taker. as 1.0 to give the worst condition, To ensure the proper action of a heel beam, the front face anust be cast cirectly against sound, undisturbed material, and itis important that this is nat overlooked during construction, In considering cantilever walls, « considerable amount of backfil is often placed on top of the base, and this is taken into account in the stability analysis. ‘The forees acting in this case are shown in figure 11.13. In addition to Gy, and ff, there is an additional vertical load H;, due to the material above the base acting a 320 REINFORCED CONCRETE DESIGN distance q from the toe, The worst condition for stability will be when this is at a 1minimara; therefore a partial load factor, = 1.0 is appropriate. The stability requirements then become LOG x + 1.00.93 y4Hiey for overturning 018) HUOG,+ 10%) 2 yl, — for sliding as) ‘When 2 heel beam is provided the additional pesive cesistance of the earth must be included in equation 11.9 ‘Stability analysis, as described here, will normally suffice. However, if there is doubt about the foundation material in the region of the wall or the reliability of loading values, it may be necessary to perform a full slip-ciele analysis, using techitiques common to soil mechanics, or to use increased factors of safety. oer Resultant Soil force eearing % pressures Figure 11.13 Forces on a canttever watt (ii) Bearing Pressure Analysis ‘As with foundations, the bearing pressures underneath retaining walls are assessed ‘onthe basis of the serviceability lic state when determining the size of base that is required. The analysis willbe similar to that discussed in section 10.1 with We foundation being subject to the combined effects of an eccentric vertical lozd coupled with an overturning moment Considering a unit length of the cantilever wall (figure 11.13) the resultant moment about the centeoidal axis of the base i WATER-RETAINING STRUCTURES AND RETAINING WALLS 321 M= A Hy +9 Ge (D2 3) + 79 MD!2- 41.19) and the vertical lond is N21 G+ tea My a5 where in this case for the serviceability limit state the partial factors of safety are Tn 32 He ‘The distribution of bearing pressures will be as shown in the figure, provided the effective eccentricity lies within the ‘middle third’ of the base, that is Me2 x6 “The maximum bearing pressure is then given by N,M oD MP PDT 2 where f= D/12. Therefore yy 1 tS 1.12) # DD and x ot 2X a cnasy POD {it) Member Design and Devaling ‘As with foundations, the design of bonding and shear reinforcement is based on an. analysis of the loads for the ultimate mit state, with the corresponding bearing pressures. Gravity walls will seldom require bending or shear steel, while the walls in counterfort and cantilever construction will be designed as slabs. The design of counterforts will generally be similor to that of a canttlover beam unless they are massive, With a cantilover-lype retaining wall the stem is designed to resist the moment caused by the force 7H, with 7¢ = 1.4 or larger, depending on how aveurately the load may be predicted. For preliminery sizing, the thickness of the wall may be taken as 80 mm per metre depth of backiil. The thickness of the base is usually of the same order as that of the stem. The hee] and toe must be designed to resist the moments due to the upward earth bearing pressures and the downward weight of the soil and base. The soil bearing pressures are calculated from equations 1.10 to 11.13, provided the resultant of the horizontal and vertical forces lies within the ‘middle third’. Should the result- ant lie outside the ‘middle third’, then the bearing pressures should be calculated ‘using equation 10.4. The partial factors of safety 7, %¢q and %45 should be taken to provide a combination which gives the critical design condition. Reinforcement detailing must follow the general rules for slabs and beams as appropriate. Particular care must be given to the detailing of reinforeement to 322 REINFORCED CONCRETE DESIGN limit shrinkage end thermal cracking. Gravity walls ae particularly vulnerable because of the large concrete pours that are generally involved, and these should be treated in the manner described in section 11.4 for thick sections Restraints to thermal and shrinkage movement should be reduced to a mini- mum; however, this is counteracted in the construction of bases by the need for {good friction between the base and soil; thus a sliding layer is not possible, Rein- forcement in the bases must thus be adequate to control the cracking caused by a hhigh degree of restraint. Long walls restrained by the rigid bases are particularly susceptible to cracking during thermal movement due to loss of hydration heat, and detailing must attempt to distribute these cracks to ensure acceptable widths, Complete vertical movement joints must be provided, and the methods used for the design of joints for water-etaining structures can be used. These joints will often incorporate a shear key 10 prevent differential movement of adjacent sections of wall. and watezbars and sealers should be used at shown in igure 11 .4e. ‘The back faces of retaining walls will usually be subject te hydrostatic forces from groundwater. This may be reduced by the provision of a drainage path at the face of the wall. [tis usual practice to provide such a drain by a layer of rubble or porous blocks as shown in figure 11.14, with pipes to remove the water, often through to the front of the Wal, In addition to reduetag the hydrostatic pressure fon the wall, the likelihood of leakage through the wall is zeduced, and wate also less likely to reach and damage the soil beneath the foundations of the wall, Rubble imo wot aro Porous pips Orange tie to oe Figure 11.14. Droinage layer Example 11.4 Design of a Retaining Walt The cantlever etaning wal shown in figure 1115 supports a granular material of satucated density 2000 kg/m? and the allowable bearing pressure ls 110 kN/mn™ eis required to (1) cheek the stability ofthe wall (2) determine the aciual beating pressures, and (G) design the bending einforcement using high yield steel, fy = 460 Nfmm? and grade 35 conciet. WATER-RETAINING STRUCTURES AND RETAINING WALLS (a) Stability Horizontal foree: itis essumed that the coefficient of active pressure K, = 0.33, Which isa typical value for a granular material. So that the earth pressure is piven P= Kypgh where p is the density of the backfill and fis the depth considered. Thus, atthe base 33 x 2000 x 10-* x 9.81 4.9 = 31.7 Nim? ‘Therefore horizontal force on 1 m length of wall is Pe Hy=0Sph=0.5 x 31.7 x4.9= 77.7 KN Vertical loads wall = $(0.4 + 0.3) x45 x 2: 37.8 KN base = 04x 34x24 326 earth = 2.2.x 4.5 x 2000 x 10 x 9.81 = 194.2 Total = 264.6 KN For stability caleulations a partic! factor of safety of 1.6 is used for the lateral loadings, while 1.4 will be used for strength calculations. {9 Sliding: from equation 11.9 itis necessary that (1.06, * 1.04) > apf, for no heel beam Assuming a value of coefficient of friction «= 0.45 ftiotional resisting force = 0.45 x 1.0 x 264.6 = 119.1 KN, sliding force = 1.6 x 77.7 = 124.3 kN Since the sliding force exceeds the frictional force, resistance must also be pro vided by the passive earth pressure acting against the heel beam and this force is given by Hy= 4X 05Ky 98a? where Ky is the coefficient of passive pressure, assumed to be 3.0 for this granular ‘material and. is the depth of the heel. Therefore Hy = 1.00.5 x 3.0 x 2000 10? «9.81 x0.6 =1063N ‘Therefore total resisting force is 119.1 + 10.6 = 129.7 RN 324 REINFORCED CONCRETE DESIGN 300 ape 2500] __--120.200 120-200 oof (Et rece ALS’ | | posse te | PERS el no ere T veering a iim _——24m__| Fue 1s Which exceeds the sliding force. Gi} Overturning: taking moments about point A at the edge of the toe, at the Ultimate limit stato overturning moment = yi t/3 = 1.6 x 77.7 x 4.913 = 203 kN im restraining moment = 1.0(37.8 x 1.0+32.6 x L.7 4194223) = 340 KN m ‘Thus the criterion for overturning is satisfied, (b) Bearing Pressures From equations 11.12 and 11.13 the bearing pressutes aze given by WATER-RETAINING STRUCTURES AND RETAINING WALLS Ny om DDR ‘where M is the moment about the base contro-line. Therefore f M=71.1 x 4.9/3 +37.8(1.7 — 1.0) + 1942 (1.7 23) 26.9 + 26.5 ~ 116.5 = 36.9 Nm Therefore maximum bearing pressure py = 2048 » $4262 ee esa" gat = 77.8 +19.2= 97 kN/m? which is less than the allowable, () Bending Reinforcement @ Wall, Horizontal force = 740.5%, ogi? A x 0.5 x 0.33 x 2000 x 107? x 9.81)x 4.57 =918kN considering the effective span, the maximum moment is M=91,8(0.2 + 4.5/3) = 156 kN m M__ 156x108 ba?fe T0033" 35 for which fy = 0.95 (figure 7.5). Therefore Ase 1369.0 = 1243 mm? /m 330 x0.87 x 460 Provide T20 bars at 200 mm centres, (i) Base: the bearing pressures are obtained from equations 1-10 to 11.13. The critical partial factors of safety ere en = Mt and pp = Ips = 1.0 Using the figures from part (b) of this example, the moment about the base centretine is M= yp, X 126.9 + 442 X 26.5 — pq 116.5 = 87.7 KN m and N= Hq (378 432.6) + 949 ¥ 194.2 = 264.6 KN Therefore 264.8 , 6x87.7 ressure p; = 048 = 18 +45 = 123 kNim? Pe 34 ee ips 326, REINFORCED CONCRETE DESIGN Be = 78 — 45 =33 kNim? ‘nd in figure 11.15 ps =33-4(123 ~ 33) 3.4 = 91 kfm? Heel: taking moments about the stem centesine for the ve and the besring pressures Mg 2328413 T5449 1A x13 = 39 22% 13 ~(91 ~33)x 0,93 = 126 KN m therefore 126» 108 O87 «460 0.95 «350 Provide 120 bars at 200 mm centres, top steel Toe: taking moments about the stem cente-Line Is = 1004 mm?/m Mem, #326 06% OS — 95 123 x08 x06 5 kN m (in fact for this wall the design moment for the toe would be marginally higher with ygq = 1.4 and yyy = 1.4 throughout.) 55 x 108 0.87 x 460 x 0.95 x 830 ls = 438 mm?) ‘The minimum azea for this, and for longitudinal distribution steel which is also required in the wall and the base is A, =0.13 x 1000 x 400= 520 mm? ‘Thus prouide T12 bars t 200m centes, boom and ditbuton sted Ako sel should be provided inthe compression fac of the wal n ores to proton crcking sy, 10 bars ¢ 200 mn cents exch way. Wein inf gute in te elu oe he ome de the pase earth pear This enforcement woud probably bein of closed links, ® % eter 12 Prestressed Concrete “The analysis ane design of prestressed concrete isa specilised field which cannot possibly be covered comprehensively in one chapter. This chapter concentrates therefore on the basic principles of prestrossing, and the analysis and design of statically determinate members in bending for the serviceability and ultimate limit states, 'A fundamental aim of prestressed concrete is to limit tensile stresses, and hence flexural cracking in the concrete under working conditions. Design is therefore based initially ov the requirements of the serviceability limit state. Subsequently considered are ultimate limit state criteria for bending and shear. Ia addition to the concrete stresses under working loads. deflections must be checked, and attention must sso be paid to the construction stage when the prestcess force is first applied to the immature concrete. This stage 1s known as the ransfer condirion, Design of prestressed concrete may therefore be summarised as (1) design for serviceability — cracking (2) check stresses at transfer (3) check deflections (4) check ultimate limit state — bending (3) design shear reinforcement for ultimate limit state “The stages are illustrated by the flow chart in figure 12.1 ‘When considering the basic design of a conevete section subject to prestress, the stress distribution doe to the prestress must be combined with the stresses from the Joading conditions to ensure that permissible stress limits are satisfied. Many analytical appreaches have been developed to deal with this problem: however, it Ss considered that che method presented offers many advantages of simplicity and cease of manipulation in design 12.1 Principles of Prestressing In the design of a reinforced conerete beam subjected to bending itis accepted that the concrete in he tensile zone is cracked. and that ull the tensile resistance is 307 - 228 aseno CLAUSE 4 eae aa REINFORCED CONCRETE DESIGN CALCULATE MamenT VARIATION uve COsos " FINSHES) my + { saape. ogP ts, wea TRIAL SECTION =——~ t SELF wT 4 Tora MaMENT LOSS aLLowaNce DEAD Loan MOMENT ‘ {cage zone worn wow aacsraeseronce «—} Liv ; {crm DESIGN PRESTRESS FOR 4 NOON PROFWE 4 TRANSFER STRESSES <— LOSSES NAL STRESSES 1 CHECK DEFLECTIONS 4 DESIGN END eLOCe ‘ ULTIMATE MoM, OF RESISTANCE t UNTENSIONED REINET <— ULT. MOMENT 4 SHEAR DESIGN + CHECK END BLOCK IUNBONDED) ‘ FiNse a prestaess system — wt, sean FoncE Figure 12.1, Prestressed concrete design flow chart FLANGE L'MIT5,cOVER, res PRESTRESSED CONCRETE 329 provided by the reinforcement. The stress that may be permitted inthe reinforce Trent is limited by the need to keep the cracks in the concrete to acceptable widths Under werkng conditions, thus there is no advantage to be gained from the we of ‘ery high strength swecls which are avalable. The design i therefore uneconomic Jn two respects (1) dead weight includes ‘useless’ concrete in tensile zone, and (2) economic use of stel resources fs not possible. “Drestressing’ means the atiflelal ezeation of scsses ina structure before load sng. s0 that the stesses which then exist under Toad are more favourable than {would otherwise be the ease, Since concrete i strong in compresion. he mater Iva Bear willbe used most efficiently if it can be maintained ina state of com pression throughout, Provision of e longitudinal eompresive force acting on a Concrete beam may therefore overcome both of the disadvantages of reinforced concrete cited above, Not only isthe concrete fully ulised, but also the need for ‘omveational tension reinforcement is removed. The compressive force is usually provided by tensioned steel wires or strands whieh are anchored agaist the con. rete and, since the stress in this steel is not an important factor inthe bebaviour of the bean but merely a means ef applying the appropriate force, full advantage may be taken of very high strength steels “The way in which the stresses due to bending und an applied compressive force may be combined are demonstzated in igure 12-2 fer the case an axially applied force acting over the length of a beam. The stress distribution at eny section will equal the sim ofthe compression and bending stresses if is assumed thatthe concrete beheves elastically, Thus itis possible to determine the applied Force 0 that the combined stesses are always compressive, [ve + = Benang stron Prastress — arding Total Gieteibution Section BE Stress Distribution Section 8-8 Figure 12.2. Effects of axel prestress 330 REINFORCED CONCRETE DESIGN [By applying the compressive force eccentrcally on the concrete crosssect further stress distribution. due to the bending effects of the couple thus ded to tote shown in fre 12.2. This ffs i usted figure 12.3 an offers further advantages when atiempting to produce working stesss wi requited limits meee a arly attempts to achieve this effect were hampered both by the limited strengths available and by shrinkage and creep of the concrete under susta compression, coupled with claxation of the steel. These meant that the steel 2 large part ofits intial pretension and asa result residual stresses were so sma to be useless, Its now possible, however to produce stronger concretes whieh have good ereep properties, and very high strength steels which can be stressed 10 a high percentage of thet: 0.2 per eent proof stress are also available. For example. hard-drawn wires may carry stresses up to about six times those possib In mild stee. This not only rests in savings of steel quantity, but also the cf shrinkage and ereep become relatively smaller and may typically emount loss of only about 25 per cent of the initial applied force. Thus, modern m mean that the prestessig of conocete isa practical proposition, with the for being provided by see! passing through the beam and anchored a each end under high tensile load, c ¢ L ‘ + + = nial Presirass Bending Eccentricity Tetol Stress Distrioution- Section 8-2 Figure 12.3. Eyfects of eccentic presress 12.2 Methods of Prestressing ‘Two baste techniques are commonly employed in the construction of prest concrete, their chief difference being whether the steel tensioning process is pet formed before or after the hardening of the concrete. The choice of method will be governed largely by the type end size of member coupled with the need for precast or in sing construction, PRESTRESSED CONCRETE 331 12.2.1 Pretensioning {n this method the steel wives or strands are stretched to the required tension and achored to the ends of the moulds for the concrete. The concrete is cast around the tensioned steel. and when it has reached sufficient strength, the anchors are released and the force in the stec is transferred to the conerete by bord. In sda tion to longsterm losses due to ereep, shrinkage and relaxation, an immediate drop in presttess force occurs due to elastic shortening of the concrete, These features are ilustrated in figure 12.4. ‘Because of the dependence on bond, the tendons fer this form of construction generally consist of small diameter wires or small strands which have good bond characteristics. Anchorage near the ends of these wires is often enhanced by the provision of small indentations in the surface of the wire. “The method is ideally suited for factory production where large aumbers of identical units ean be economically made under controlled conditions, a develop- ment of this being the ‘Long ine’ system where several units can be cast at once rend to end — ahd the tendons merely cut between each unit after release of the anchorages. An advantage of factory production of prestressed units is that special ised curing techniques such as steam curing ean be employed to increase the rate of hardeming of the concrete and to enable earlier ‘transfer’ of the stvess to the concrete. This is particularly important where re-use of moulds is required, but it fs essential that under no cifcumstances must ealeium chloride be used as an Gear with pretansines wil Me __ (as Gran ~ Fase) and Ms se 29)" Gran — Fi) ‘These minimum values of section moduli must be satisfied by the chosen section in order that prestress foree and eccentricity exist which will permit the stress limits to be met. Stresses at transfer are discussed in 12.4.3, and to avoid overstress- ing in that condition the chosen section must have a margin above the minimum values of section moduli calculated above. Detailed calculations may be based on loss estimates, but it will usually be adequate at this stage to provide a section with moduli exceeding the ealculated minima by 20 per cent for post-tensioned and 35 percent for pre-ensioned construction. The masximum moment on the section has not directly beer included in these figures, thus itis possible thatthe resulting pre stress force may not be economic or practicable. However, itis found in the majority of eases that ifa section is chosen which satisfies these minimum requlte ments, coupled vith any other specified requirements regarding the shape of the section, then a satisfactory design is usually possible. The ratio of acceptable span-depth for a prestressed beam cannot be categorised on the basis of defle tions as easily as for reinforced concrete. In the absence of any other criteria, the following formulae may be used as a guide end will generally produce reasonably conservative designs for post tensioned members. span 36 m 2 01m 25 span span > 36m hee SPE gy . 20 In the case of short-span members it may be possible to use very much greater span-depth ratios quite satisfactorily, although the resulting prestress forces may ‘become very high Other factors which must be considered at this stage include the slenderness satio of beams, where the same criteris apply as for reinforced conezete, and the possibility of web and flange splitting in langed members. Example 12.2 Seleetion of Crosssection Select rectangular section for a post-tensioned beam to carry, in addition to its self.-weight, a unformly distributed load of 3 kN/m over a simply supported span M2 REINFORCED CONCRETE DESIGN cof 10m, The member is to be designed as class 1 with grade 40 concrete, without lateral support. Class 1 member. thus 40 2 fnas = 32 = 13.3 Nimo fo 7 7 Soin = ON/tnm? “Live toad? moment at = 2% 2375 kN m mid-span My 5 tis Ps 375 x 108 = SSA = 2.82 x 10° mm? (minimum) Gam —fmind 133 1mm a ‘This should be increased by 20 per cent to allow for transfer case. Therefore 2x 282 x 106 mm? =3,38 x 10° ma? ‘To prevent lateral buckling. BS 8110 specifies maximum permissible span) breadth = 60, that is hence ‘minimum f= Jess x 1) 170 = 345 mm This epresemts a span=depth eatio = 10 x 102/345 = 29.0 which may prove to be excessive when deflections are checked (see example 12.7) but asa first trial a section 350% 170 is adopted (2, = 2 = 2= 3.47 10% mm*) and this fs used in subsequent examples. 12.4.2 Design of Prestress Force ‘The inequalities of equations 12.9 and 12.12, and 12.10 and 12.11 may also be combined to yield expressions involving the moment variation Mfy, thus PRESTRESSED CONCRETE 343 129 and 12.12 y= Wax Mose) ( +2) (fas PD 27) 12.1Dand 12.11 My = Woes ~ Mra) © (a2) P= Afri) 2a thus iff and P are treated as variables, these are both of the general form ‘My 2, the value of ¥ is given by the intersection of equations 12.14 and 12.18, that is (2.19) a4 REINFORCED CONCRETE DESIGN and ¥ is similarly given by pZa + Sinz ute Izy > 2, then 12.13 lies below 12.14 and the values of ¥ and Y are interchanged. ‘The minimum prestress force for a given moment variation is therefore given by equation 12.18 whieh Is based on satisfying the minimum stress requirements, ‘This may be rewritten in the form Mt P> + Abin satis p> thes fawkes +23) (zane (2 Although a range of values of permissible prestressforve ean be found, this makes xo allowance for the fact that the corresponding eccentricity must lie within the bbeam. It is necessary therefore to consider the effect of limiting the eccentricity to a maximum practical valve for the section under consideration, The effect of this limitation wil be most severe when considering the maximum moment acting fon the section, that is, the inequalities of equations 12.9 and 12.11. If the limiting value for maximum eccentricity, émox depends on cover requite- ments: equation 12.9 becomes ine Sines am .) 2 Mrsx [fms 350 REINFORCED CONCRETE DESIGN sand equation 12.2% es[ 2 fags | Mos 4? ? quation 12.27 becomes ea[o- BAT x ie] 350% 170, >= 58.3 mm Simic equation 12.28 becomes [stx10§ | 0 380% 170 54583 mm ‘Thus at the ends of the member, the tendons must lie at a practical eccentricity ln the range * 58 mm. (b) Midspan Equation 12.27 gives elo 17.9 « 108 >- 583-556 >— 113.9 mm and equation 12.28 gives é [sstzte o| 55.4 x 10° 350%170 32210" 6583-172 <-113.7 mm Hence at mid-span the tendon must theoretically lie at an eccentricity of 113.8 mmm and the practice! width of cable zone is zer0 for this prestress force, 124.5 Width of Cable Zone ‘The widths (6,) of the permissible cable zone at any section x may be obtained by subtracting equations 12.28 and 12.27 for a simply supported beam, thus fitntt — Mmas _ Saints £2 4 Mein 4° e See A PRESTRESSED CONCRETE 351 therefore — fay Et) — Uh an OE ‘where the values of moment, prestress and section properties are those relating to stction x, The design expression for minimum prestress foree based on moment variation (equation 12.21) may be ceweitten so that (ap) mz) ‘where My = design moment variation at critical section. Hence ot (Mom =Mnie) * P ‘When section x coincides with the critical section, (Mag — Mpg) corresponds to Mz and hence 5, =O, unless the value of Mf, used in the design eateulaions for pre stress force is increased to provide fer a postive zone width, in which case the value of M, used in the design must be ebtained from the above expression, so that My Serax ~ Minin) +P ‘where is the minimum required zone width at section x, Hence the expression for minimum peestzess force corresponding to equation 12.21 becomes 02.30)* Example 12.6 Calculation of Prestress Force for Minimum Cable Zone Width Find the minimum prestres force necessary for the beam in examples 12.2 and. 12.3 ifthe minimuin cable zone width is to be * 10mm and minimum cover remains at 50 rm (a) Based on Moment Variation from Equation 12.30 Atimid:span sestion 382 REINFORCED CONCRETE DESIGN where 5, = 20mm, Hence > 37510 +0 2347 x10 350x170, 2388 kN x10 -20 (b) Based on Limits of Eeventrieity from Equation 12.25 ‘Now for maximum tolerance. émay may be reduced to ~ [(i/2 ~ Cover] + 20, thus, emax =~ (175 ~ 50) #20=— 105 mm hence p> Mass + fa I 55.4 x 108 +0 3.47 x 10% x10 +105 ‘Thus. moment variation still governs, Ie will be noted immediately that the minimum prestress force is ineveased con. siderably by this requirement, and approaches the maximum economic value of 395 KN for the section chosen. as determined in example 12.3, This demonstrates clearly the need for prevision of construction in prestressed concrete merabers, particularly with reference to tendon fixing. The importance in selecting section properties which exceed the minimum values by a reasonable margin i also ‘emphasised and confirmed by example 124 in order that transfer conditi De met 124.6 Preseress Losses From the moment that the prestressing force is first applied to the concrete member, losses of this force will take place because of the following causes (2) Elastic shortening of the concrete, (2) Creep of the concrete under sustained compression (3) Relexation of the prestressing steel under sustained tension. (4) Shrinkage of the concrete PRESTRESSED CONCRETE ‘Those losses will oecur whichever form of construction is used, although the effects of elastic shortening will generally be much reduced when post-tensioning is used. This is beceuse stressing isa sequential procedure, and not instantaneous as with pre-tensioning. Creep and shrinkoge losses depend to a large extent on the properties of the concrete with particular reference to the maturity at the time of stressing. In pretensioning, where the conccete is usvally relatively immature at transfer these losses may therefore be expected to be higher than it pos tensioning. In addition to losses fom these causes, which will generally total berween 20 10 30 per cent of the inital presteess force at transfer, further loses ozcur in post- tensioned concrete during the stressing procedure, These are due to friction between the stands and the duct, especially where curved profiles are used, and to mechanical anchorage slip during the stessing operation, Bota these factors depend on the actusl system of ducts, anchorages and stressing equipment that are used, ‘Thus although the basic losses are generally highest in pre-tensioned members, in some instances overall losses in posttensioned members may be of similar rragnitude, Elastic Shortening ‘The concrete will immediately shorten elastically when subjected to compression, and the stee! will generally shorten by a simitar ammount (as in pre-tensioning) with ‘a corresponding loss of prestress force. To calculate this it is necessary to obtain the compressive strain at the level of the steel. If the transfer Force is Py, and the force after elastic losses is P’ then =P, — loss in force ‘and the corresgonding stress in the conerete at the level of the tendon F Weare ot 4 COKE 5 gy, A Tt fbea) where f(g) isthe stiess due to self weight which wil be eelatvely small when averaged over the length ofthe member and may thus be neglected. Hence ak (+ #4) and concrete stain = fe/B,, thus reduction in steel strain = fe/E, and reduetion in steel stra thus los in prestiess force = acfeAe whore 4 = area of tendons 3st REINFORCED CONCRETE DESIGN f Ag +H) rie ( TH hence rena dsy (1424) 4 7 so that remaining prestress force P ’ 1+a, 48 (1424 4 7 In prectensioned construction this full loss will be present; however when post- tensioning the effect will only apply to previously tensioned cables and although a detailed calculation could be undertaken itis normally adequate to assume 50 per cent of the above losses. In this case the remaining prestres force is fo 205 and it is this value which applies to subsequent loss calculations, Pe Greep of Concrete ‘The sustained compressive stress on the concrete will also cause a long-term short= ening due to creep, which will similarly reduce the prestress force. As above, itis the stress in the concrete at the level of the steel which is important, that is P eA aoF (184) loss of steel stress and sf Kasi creep satin then Tou of presse force =, 48 ( +2 4) x specific ereep stain ‘The value of specifi cee use in this calculation wil be influenced by the factors cessed i ection 63.2, and may be obtained ftom the vals of cre content 9 fiver in figure 6 5 using the ratonship @ cfc exeep stzain = © / N/mm? spe P =/™ For most outdoor exposure purposes in the UK it will be adequate to use values ‘of creep coofficiont between 1.8 for transfer within 3 days and 1.4 for transfer after 28 days, PRESTRESSED CONCRETE Relaxation of Steet Despite developnents in prestressing steel manufacture which have taken place in recent yeas relaxation of the wite or strand under sustained tension may still be expected to be a significant factor. The precise value will depend upon whether pre-tensioning or post-tensioning is used and the characteristics of the steol (ype defined in BS $896, Factors allowing for method of construction are given in 'BS 8110 which should be applied to 1000 hour relaxation values provided by the manufacturer. The amount of relaxation will also depend upon the initial tendon load relative to its breaking load, In most practical situations the transfer steel stress is about 79 per cent of the characteristic strength and relaxation losses are likely to be approximately 810 per cent of the tendon load remsining after transfer. This los decreases linestly to zero for a transfer stress of about 40 per cent characterise Shrinkage of Concrete ‘This is based on empirical figures for shrinkags/anit length of concrete (éq)) for particular curing conditions and transfer maturity as discussed in chapter 6. ‘Typical values for prestensioned concrete (stressed at 3 to 5 days) range from 100 x 10-* for UK outdoor exposure to 300 x 10~* for indoor exposure. Cor: responding values for post-lensioning (stressed at 7 to 14 days) are reduced t0 70 x 10~* and 200 x 10°®. More detailed guidance in unusual circumstances ‘may be obtained from section 6.3.2. “The loss in steel stress i thus given by eq, hence Joss in prestres force = ey) EsAg ing only) When a pest-tersioned cable is stressed, it will move relative to the duct and other ccebles within the duct and friction will tend to resist this movement hence reducing the effective prestress force at positions remote from the jacking point. This effect may be divided into unintentional profile variations, and those due to designed curvature of ducts. (@) ‘Wobbve' effects in straight ducts will usually be present. If P, = jack force, and P, = cable force at distance x from jack them itis generally estimated that Friction in Ducis Post-tensi Py = Poe ‘whet e = base of napierian logs (2.718) and k = constant, depending on uct characteristics and construction procedures. generally taken as <£33% 10-* but reducing to 17 x 10 in special cases. (©) Duct curvature will generally couse greater prestrss force losses, and is given by Py = Poe Bos Where 4 = coefficient of friction (typically 0.55 steel on concrete, 0.3 steel on steel, 0.12 greased strand on plastic) and 7), = radius of cusva 356 REINFORCED CONCRETE DESIGN tute of duct. If7;q isnot constant, the profile must be subdivided into sections, each assumed to have constant rpg, in Which ease Py is taken 2s the force at the jacking end of the section and x the length oF the seg- iment, Py. the force at the end remote from the jack then becomes P, for the next section and so on. The above effects may be combined to produce an effective prestress force diagram for a member. If frition losses are high, it may be worth while to jack simultaneously from both ends. in which case the two diagrams may be super. imposed, maintaining symmetry of prestess force relative to the length of the member. Example 12.7 Estimation of Prestress Losses ‘A rectangular 250 x 150 mim pre-tensioned beam is stressed by wires of total area 200 mum? with a total characteristic strength of 370 kN al an eccentricity of ~0 mm, If the transfer prestress force is 250 KN, estimate the final value after losses, Assume: Eq (transfer) = 28 kN/oum? ; £, = 205 N/mm? : shvinkage/unit length (6) * 300 x 10": specific creep = 48 x 1078 N/mm? (1) Hlastie Shortening a r 1+a, 48 184) gee where A = 37500 mm? BE = 105. 108 momt ’ +0 14 25, 200 (Resta 28 37500 195 x 10% @) Oe Lonintoceeaitesaentix® (1224) ae 4830 205 110 BE 50* x 37500 i 097500) (1+ Rigr ) PRESTRESSED CONCRETE 387 wnat (saisi= 82 #732) (8) Relaxation 250 ransfer force = 2 104 Transfe 5 67.5 per cent characteristic ‘therefore assuming that a joss of about § per cent corresponds to a stress of 70 per cent characteristic, relaxation loss 8 x ©2549) 07.3 percent therefore losin force = 2 4 236.3 17.3 kN (4) Shrinkege Loss in Force = equB,Ay = 300 x 10° x 205 x 200 =1232N Thus final prestrss force = 2363 — 18.3 —17.3 - 123 = 188 kN and 62 total estimated loss of force is 12.4.7 Calculation of Deflections ‘The anticipated deflections of a prestressed member must always be checked since specific span-elfective depth ratios are not met in the design procedure. The deflection due to the eccentric presiress foree must be evaluated and added to that from the normal dead and applied loading on the member. In the vase of clas 1 and 2 structures. the member is designed to be uncracked, and a similar procedure Js followed to that described in chapter 6, Although class 3 members are designed as cracked under full load, when evaluating deflections due to non-prestress load- ings ithas been found that litle error is introduced if the uneracked case is again considered, thus simplifying cslculations considerably. BS 8119 recommends that for class 3 members such an assumption may be made ifthe permanent load is no ‘more than 25 per cent of the total design load, If this isnot satisfied then the ‘member deflections must be evaluated as cracked unless the basic span-effective dept ratios {section 6,2) are satisfied. in which case the deflections of the member may be assumed to be not excessive. 358 REINFORCED CONCRETE DESIGN ‘The basie requirements which should generally be satisfied in respect of deflee- tions are similar to those for a reinforced beam (section 6,3), which are (1) Final deflection > span/250 measured below the level of supports, @) 20mm or span?500 maximum movernent after Einishes applied. Additionally in prestressed concrete (3) Total upward detlection > span/3S0 or 20 mm where finishes are applied, unless uniformity of camber Between adjacent units can be ensured. ‘The evaluation of deflections due to prestress loading can be obtained by ‘double integration of the expression Eley oo M, = Pe. = ‘over the length of the member, although this calculation can prove tedious for ‘complex tendon profiles, ‘The simple case of straight tendons in a uniform member however, yields Af= Pe =a constant, which isthe situation evaluated in section 6.3.3 to yield a raximum mid-span deflection of ~ML?/8E# =~ PeL?/SEF.If the cables le blow the centroid axis, is negative, and the deflection due to prestress is then positive, that is upwards. ‘Another common ease of a symmetzical parabolic tendon profile in a beam of constant section can also be evaluated quite simply dy considering the bending. ‘moment distribution in terms of an equivalent uniformly distributed load. For the beam in figure 12.13 the moment due to prestcess loading at any section Is My = Fe, but since e, is parabolic, the prestress loading may be likened to a uniformly distributed load w, on 2 simply supported beam: then mid-span ‘moment Ma Mel? ope, thas w= See. oT But since the mids span B isgiven by. on due to a uniformly distributed load w over a eet wet 3a Er the deflection de tow is _ 5 fey El & PRESTRESSED CONCRETE 359 Figure 12.13 Porebole tendon profile W the prestras force does not lie at the centrotd of the section at the ends of the beam, but at an eoventrieity & as shown in figure 12.14, the expression for flection must be modified, [tan be shown that the deflection is the same 25 thet eqused by a force P acting at a constant eccentricity eg throughout the length ‘of the member, plus « force P following a parabolic profile with midspan eccenti city ef as shown in figure 12.14 Figure 12.14. Peeabelie tendon profile eccentric at ends of bet ‘The mid-span deflection thus becomes a Peojt? _ 5 (Pespbt Sal aB ET Deflection: due to more complex tendon profiles are most conveniently est ‘mated on the basis of coefficients which can be evaluated for commonly occurring arrangements. These are on the basis »’ = (KL? )/EY where K inconporates the varis- tons of curvature due to prestress along the member length. ‘There are three principal stages in the life of a prestressed member at which deflections may be critical and may requite to be assessed 360) REINFORCED CONCRETE DESIGN (1) At transfer — a check of actual deflection at transfer for comparison ‘with estimated values is a useful guide that a prestressed bear has been correctly constructed. (2) Under dead load, before application of finishes — deflections must be ‘evaluated to permit subsequent movement and possible damage to be estimated (3) Long term under full load ~ deflections are required, both to determine the subsequent movement and also to assess the appearance of the final structure. Short-term deflections will be based on materials properties associated with ehatacteristie strengths (7g, = 1) end with actual loading (yq= 1). Long-term assessment however must not only take into account loss in prestress force, but aso the effects of ereep both on the applied loading and the prestess loading ‘components of the deflection. Creep is allowed for by using an effective modulus of elasticity forthe concrete, as discussed in section 6.3.2 ‘Thus if Beis the instantaneous value. the effective value after creep i given by Eure ‘where the value of @, the creep coefficient can be obtained from figure 6.5, It ean be shown in some instances that when net upward deflections occur, these often increase because of creep. thus the most critical downward deflection may well be before oreep losses occur, while the most critical upward deflection ‘may be long term, This further compliea‘es & procedure which already has many ‘uncertainties as discussed in chapter 6; thus deflections must always be regarded as estimates only. Example 12.8 Catculation of Deflections Estimate transfer and long-term deflections for the 350 x 170 mm beam of 10m span in examples 12.2 to 12.4 if itis assumed that the mid-span eccentricity = ~ 100 mm and the end eccentricity = 0. Assume that prestress losses amount to 25 per cent and that the creep coefficient = 2.0. The prestress force may be assumed constant throughout the memaber. (From the previous examples, final presties force P= 322 KN, minimum loading Wayq = 1.43 N/m and maximum Joading Wmax = 4.43 KN/m.) (@) At Transfer P= 322 =430KN o7s take £, = 31 kN/mm? as typical for @ grade 40 concrete. Thus defecton ue toseltneight=— 5 niad* ia eg defection te topesuen =~ Beol* a Es PRESTRESSED CONCRETE si But ro HE 5 10350 «6074108 mat 2 2 ‘Thus deflection. 5 x 1.43 x 10* x 10! 384 x31 x 10 x 607 x 10% 5. , 430 10? x (~ 100) x 10? x 10° 8 31x 10° x 607 x 10 =99+24 14 mm (upward) (b) At Application of Finishes If the dead load due to finishes = 2,0 kN/m, the instantaneous deflection due to finishes =5x20x 10 x10" 384 x 31 x 10° x 607 x 10 = 14 mm (downward) Assuming that only a small proportion of prestress losses have occurred at this stage, the total deflection will be given by pets 14=0 (©) Inthe Long Term Fe, 2 = fe 22103 eN/mm? fare ied mm ‘Thus defection under sustained minimum loading of dead load plus finishes becomes = SX(0.43-+2.0) x108 x 10"? $4322 x 10 (— 100) x 10? x 10 BB x 103 10" «607 x 10% 48x 10.3 x 10? x607 x 1 == 71+53=— 18 mm (downward) ‘The deflection under sust 1d maximum loading is given by =x $3 455 3.43 = 92453 — 39 mm (downward) 362 REINFORCED CONCRETE DESIGN ‘The criteria that should be satisfied are (Q) Maximum downward deflection ‘This is just satisfied. (2) Maximum upward deflection = span/3S0= 29 mm or 20 mm. This is satisfied. (3) Maximam movement after finishes = span/S00 = 20 mm, span/250 = 10000/250 = 40 mn, ‘The actual value is given by ‘Maximum long-term deflection ~ instantaneous deflection after application of finishes ~39-0= 39mm Hence this requirement is not satisfied, and special consideration must be given to the importance attached to this criterion In this particular instance 12.4.8 End Blocks In pre-tensioned members, the prestress force is transferred to the conerete by bbond over a definite length st each end of the member. The transfer of stress to the concrete is thus gradual. In post-tensioned members however, the force is ‘concentrated over a small area at te end faces of the member, and this leads to high tensile forces at right angles to the direction of the compression foree. This effect will extend some distance from the end of the member until the compression hhas distributed itself across the full concrete cross-section, This region is known 28 the ‘end block’ and must be heavily reinforced by stool to resist the bursting tension forces. End block reinforcement will generally consist of closed links which surround the anchorages, and the quantities provided are usually obtained from empirical methods - |r| + Flot Pinte anchorage. Conicel anchorage Figure 12.15 ‘Typical flow lines’ of compressive stress are shown in igure 12.15, from which it can be seen that whatever type of anchorage is used, the required distribution ccan be expected to have been attained at a distance fom the loaded face equal to the lateral éimension of the member. This is relatively independent of the anchor age type, and the distribution of bursting tensile stress is generally as shown in figure 12.16. ; | RrSTRESSED CONCRETE 6s = a + “Send WYK. Distonee irom end face of member igure 12.16 ‘The magnitude of theso strosses depends on the ratio of the dimensions of the loaded area to the dimensions of the end block. It will normally be nevessary to ‘establish the end-block dimensions both horizontally and vertically besed on the ‘ize of the end face of the bearn and the layout of the enchorages. The end block for each individual anchorage will be syrminetrical about the centre line of the anchorage and its total width (2),) will be limited by the distance (yg) to an edge of the concrete member or half the distance to an adjacent anchorage. Values of bursting tensile force (Fig) ate given in table 12.1 related to the jacking force (Po) for a square end block of side 2), loaded theough a square anchorage of side 2)po- Ifa cirevlar anchorage is used, ther. 2)pq is taken as the side of a square of equiva: Jent area, and if the end block is not Square, then separate values of Fg; must be evaluated for both vertical and horizontal planes based on the largest symmetrical end block, Once Fp. has been obtained, reinforcement is provided to act ata stess of ‘200 Nimm@ and is usually distributed evenly over the length of the end block. The calculation is thus based on serviceability conditions and will be adequate for Table 12.1 Bursting forces in end blocks oe 03 ot os 06 o7 20 be 023 0.20 017 ond on 364 REINFORCED CONCRETE DESIGN bonded tendons, If tendons are unbonded, an ultimate limit state check with Fie ‘based on the tendon characteristic load and with reinforcement acting at its design, strength of 0.87f, will be necessary. High local streses should also be controlled by Limiting the maximum corapres- sive bearing stress to 0.6 x transfer eube strength, and extra helical reinforcement is often incorporated into "wedge’ type anchorages. In situations where there is more than one anchorage, each should be treated {indjviduelly and then combined as indicated in example 12,9. Example 12.9 Design of End Block Reinforcement (Bonded) “The beam end in figure 12.17 is stressed by four identical 100 mm conical anchot- ages located as shown, with a jacking force of 400 KN applied to each. ‘The area may be subdivided into four equal end zones of side 200 x 150 mm (Giguse 12.17), that is 2y%9 = 200 mn vertically = 150 mm horizontally 200. saa! 150 [am 5 Figure 12:17 [Equivalent square anchorage has side 2y, vertically fon x 100? /4) = 88 mm. Thos po $8 94g Bye 200 hence from able 12.1 Fg = 400 «0.188 = 75.2N to be resisted by horizontal steel within 200 mm of end face; and horizontally PRESTRESSED CONCRETE ws Ye = B aos9 Be 8D hence from table 12. Frog = 400 x 0.143 = 57.2 KN to be resisted by vertical steel within 150 mm of end face. ‘Then using High-yield steel with f, = 460 N/mm? for bonded tendons 200 N/mm? allowable stress hence forve of 75.2 KN requires that is, three 10 mm closed links (471 mm*) adequate, at say $0, 100 and 150 mm from end face Gheck horizontal plane: two links lie within 150 mm of end face, thus srequitement satisfied ‘Consider combined effects of anchorages, Py, = 4 x 400 = 1600 kN side of end block = 400 mm each way (Figure 12.17) side of equivalent anchorage = (88? x 4) = 176 mm hence and. Fp = 1600 x 0,188 = 301 KN {o be resisted by horizontal and vertical steel over 400 mm from end face needing 301 0? © 1505 mm? 200 provide es soven 12 min links (1384 mm) at $0 mm centres commencing 50 mm from end face of the beam 12.5 Analysis and Design at the Ultimate Limit State After a prostrossed member has been designed to satisfy serviceability require- ‘ments, a check must be carried out to ensure that the ultimate moment of resist ‘ance and shear resistance are adequate to satisfy the requirements of the ultimate limit state. The partial factors of safety on loads and materials for this analysis are the normal values for the ultimate limit state which are given in chapter 2. 366 REINFORCED CONCRETE DESIGN 12.5.1 Analysis of the Section As the loads on a prestressed member increase above the working values, eracking ‘occurs and the prestiessing steel begins to behave as conventional reinforcement, The behaviour of the member at ultimate is exactly the same as that of an ordinary reinforced conerete member except that the initial stain in the stee) must be taken into account in the calculations. The section may easily be analysed by the use of tie equivalent rectangular stress block dseribed in chapter 4, BS 8110 contains tables to permit the stress in the prestressing steel at ultimate, and the correspond: ing neutral axis position to be obtained for rectangular sections. These are based ‘on empirical results but alternatively the simplified method illustrated in example 12.10 may be adopted for bonded members. ‘Although illustrated by a simple example this method may be applied to a cross-section of any shape which may have any arrangement of prestressing wires cr tendons, Use is made of the stress-strain curve for the prestressing steel as shown in figure 12.18, to calculate tension forces in each layer of steel. The total steel strain is that Gue to bending added to the initial strain in the steel resulting from prestress. For a series of assumed neutral axis positions, the total tension capacity is compared with the compressive force developed by a uniform stress of 0.45 f.,, and when reasonable agreement is obtained, the moment of resistance can be evaluated, Stross varies 165kNtmm? $0. 2OBKNF mom? ‘occoraing to steel type (884486 or B55296) 0-008 stroin Figure 12.18. Siestsirin curve for presresing steel Example 12.10 Colewlation of Utimate Moment of Resistance ‘The section ofa presensioned beam shown in figure 12.19 is stresed by ten 5 mam wires of characteristic strength fpy = 1470 N/rom?, If these wires ae PRESTRESSED CONCRETE 367 initially stressed to 1000 N/mm? and 30 per cent losses are anticipated, estimate ‘Area of 5 mm wire axSt/4 = 19.6 mm? ‘Stress in steel after losses = 1000 x 0.7 = 700 N/mm? Ee +0035 048 fey r & 09x | OT 6 Section Bending Stress strains Block Figore 12.19 ° e & (0-098) (oan) Figure 12.20. Siresssecin curve for wire 368, REINFORCED CONCRETE DESIGN therefore strain in stel after losses = 4 = —700__ B 205x10 which sles than the lower yield strain, ‘A dopth.x of neutral axis must be found for which the compressive force Fe in the concrete i balanced by the tensile force F in the stel. Then the ultimate moment of resistance is given by My = Fe = Fee 231) where 2 is the lover arm between Fe and Fy ‘As a fist attempt try x = 130 mm, approximately equal to 0.5¢. = 0,0034 (a) Steel Strains Final stet strain ¢ = prestress strain + bending strain, ¢} (in calculating ey, the initial concrete strain due to prestress can be ignored with- out undue error.) Top layer a= 0.0034 + eb, therefore eq = 0.00344 C502 0232) = 0.0034 + 252-139) p9935 130 = 0.0066 Bottom layer ay" 0.0034 + ey =0,0034+ a foe 0233) = 0.0034 + 275 139 9.9935 130 = 0.0073 ) Steel Stesses From the sese-strain curve the corresponding stel stestes are Top layer Fa ee Many (on 0.008) (1234) PRESTRESSED CONCRETE 369 = 1022 + 41 29010,0086 ~ 0.005) = 1088 N/mm? and Jap 102 + 41 290 fey) — 0.005) 0235) = 1022 +41 290700073 — 0.005) = 1117 Némm? (©) Forces in Steel and Concrete Steel tensile foroe F, = Ess = Usa + fn)S X 19.6 (12.36) = (1088 + 1117)98 = 216% 10° N With a rectangular stress block concrete compressive force Fe~ 0.45 fob x 0.9% 0239 = 0.45 x 40x 120 130 x0.9 253x109 N ‘The force Fe in the concrete is larger than the force F, in the steel, therefore @ smaller depth of neutral axis must 3e tried. Table 12.2 shows the results of calculations for further trial depths of neutral axis, Forx = 110, F became smaller than F,, therefore x = 120 and 116 were tried and it was then found that F= Table 12.2 Stains Stresses Fowes x fe Sh fw ov on Fe (um) (10°) (Nim?) en) 130 66 73 1088 nia 216 253 no 78 861138 7 226 na 120 72 790 1113 142 221 233 116 14 82121 usa 23 225, In terms of the tensile force in the steel, the ultimate moment of resistance of the section is given by My = Fz DUGAgid —0.45%)] (12.38) = 5.4 196 [1124 (250 — 0.45 x 116) + 1154(275 ~ 0.45 x 116)] = 46.9 x 10% Nimm 370 REINFORCED CONCRETE DESIGN Ix had been incorrectly chosen as 130 mm then using equation 12.38 My would equal 44.1 KN m, or in terms of the concrete My = 0.45 foyb x O92 0.45 x 40% 120 x 0.9 x 130(262.5 ~ 0.45 x 130) x 10% =S1kNm Comparing the avorage of these two values of Mfy (= 47.5 kN m) with the corcect answer, it can be seen that a slight error in the position of the neutral axis does rot have any significant effect on the calculated moment of resistance. 12.5.2 Design of Additional Reinforcement If itis found, es may be the case with class 2 or 3 members, that the ultimate limit state requirements are aot met, additional untersioned o partially tensioned steet -may be added to increase the ultimate mament of resistance, Example 12.11 Design of Untensioned Reinforcement Design unteasioned high yield reinforcement (fy = 460 N/mm?) for the rectangular beam section shown in figure 12.21 whichis stressed by five 5 mm wires ifthe ultimate moment of resistance is to exceed 40 KN m for grade 50 concrete. The charaeterstie strength of tensioned stel,f, = 1470 Num? (a) Check Ultimate Moment of Resistance Maximum tensile force if prestrossing steel yielded 519.6 x49 107 1S = 125kN 125 x 107 Das x 50 Concrete compressive area to balance ‘thus, neutral axis depth x = 51mm. ‘Assuring prestrain as calculated in example 12.10 = 0.9% 120" total stel strain = prestrain + bending strain = 0.003 + 4%) 0.0035 24 = 0.0038 + ee Sr 0.0035 = 0.0187 (> yield) Lever arm = 278 04S x $1= 252mm Hence 252. 125 x 10 =31.5 Nm Untensioned steel is therefore required to permit the beam to support an ultimate ‘moment of 40 KN ma, ultimate moment of resistance r § PRESTRESSED CONCRETE 71 Additional moment of capacity to be provided = 40 ~ 31.5=85kNm Effective depth of additional steel then lever arm to adaitional steel® 210 mm then ‘additional tension force required = ®59 = 40.5 kN 210 e120, £10 00: 5 ‘4 Poy Pea [. & |. os] | 4 ol | pee} 4 = . e| 2710 et reps oy ee sendin ress Bloc sect oe Stress lock gue 1221 tus elumated aa of untonsoned seal. 49500 _ 199 apt fequted ot sya tes 360x087 ‘Try two 10 mm diameter bats (157 mm?) (b) Cheek Steel Strain If additional steel has yielded, force in two TIO bars = 157 x 460 x 10-*/1.15 = 62.38 KN, therefore total tensile force if all the steel has yielded = 125 +62.8 = 187.8 kN Thus 1878 x10" th of neutral axis at ultimate = 1878 x10" __ Senna 0.45 x 50x 1200.9 77 mam 37 REINFORCED CONCRETE DESIGN oe pein sing « 9B ors + 0084 = 0.0124 © yield) wt inion ain = 227 coos =0,0076 “This value is greater than the yield stain of 0.002 from section 4.1.2, (©) Check Ultimate Moment of Resistance Taking moments about the centre of compression M,, = 125(275 — 0.45x) + 62.8 (245 —0.45x) = [125(275 - 0.45 x 77) + 62.8(245 — 0.45 x 77)] 10-7 243.2kNm Fit had been found in (b) that either the prestressing steel or untensioned steel hhad not yielded, then a trial and error approach similar to example 12.10 would ‘have been necossury. 12.3.3 Shear Shear in prestressed concrete is considered atthe ultimate Limit state, Design for shear therefore fnvalves the most severe loading conditions, with the usual partil factors of safety on loading forthe ultimate limit state being incorporate. ‘The action of a member in resisting shear is similar to that for reinforced concrete, but with the additional effects of the compression due tothe prestress force. This will ierease the sheer resistance considerably, since design is based on limiting the diagonal principal tensle stresses in the concrete ‘Although most prestressed concrete members willbe uncracked under working toa, when carrying the Loads fr the ultimate limit state they may well be cracked over part oftheir span. Tis will reduce the shear capacity, bur fortunately the regions of cracking in simply supported members will generally be the centre part of the span whete shear forces are relatively smal. Uneracked Section ‘At an uncracked section, a Mohr’s circle analysis of a beam element shown in Figure 12.22 which is subjected to a longitudinal compressive stress fe and a shear stress veo, gives the principal tensile stress as N-2] (9 PRESTRESSED CONCRETE 33 ‘This can be rearranged to give the shear stress Yeo = VUR + Soh) Figure 12.22 ‘The actual shear stress at any level of a beam subjected to a shear foree, ¥, can be shown to be act) oF ‘where (A) isthe first moment of area of the part of the section above the level considered about the centinidal axis of the beam, as shown in figure 12.23, bis the breadth of the section at the level considered and Zs the second moment of area of the whole section about its centroidal axis Hence iff; is the limiting value of principal tensile force, the ultimate shear resistance Veg of an uneracked section becomes » Veg PE Mut + ff ay) ~ Fora rectangular section the maximum shear stress occurs at the centroid, thus A= bid, 1= bh? [12, ¥ =h/4, then ib Jb s067th aw) and 3 Veo ro" > ae siving Vigg = 0.676 VRE + Fee) “This equation forms the basis of the design expression given in BS 8110. A partial factor of safety of 0.8 is applied to the ceatroldal compressive stress due to resttess fey. hence fe = O.8fep. Je is taken as positive and is given a limiting value 374 REINFORCED CONCRETE DESIGN centroves Ie t SY cross -Seetion, Shear Strass ste Figure 12.23 0f 0.24 fin, which may be roparded as being eqvalent £0 0.3 Veal) With Yn “The resulting expression Von = 0.67 bh YUP + O.8fen.h) ‘may also be applied to I-and T-sections with sufficient accuracy for practical purposes, sithough the maximum principel tens stress may not coincide with the centroid. If the centroid of the section lies within the flange however, the ‘expresion should be evaluated forthe fange/web junction with & taken as the web width and fay being the compression due to presress at that level. Ifa duct les inthe web, then the value of 6 used in caleuations should be reduced by the duct diameter ifthe tendons are unbonded or two-thirds of the diameter if bonded. Additional shear resistance will be provided by the vertical omponent of the prestress force whece curved cables ate used, proved the section is uneracked, "Near the ends of beams where shear forces ere highest, and cable slopes generaliy steatest, a considerable increase in resistance can be obtained from this, and shear Strength contribution should be a consideration when detailing tendon profiles. The total shear resistance of an uncracked section may ther be taken ts Vo= Veg * B sn where is the cable slope. Oacked Section BS 8120 gives an empirical expression for the calculation of shear resistance of 2 section which is cracked in flexure Lou Mo Veg= (1-055 28) y.ba+ MO v0.1 bd Vin (oss) where fpe = prestressng stel stress after losses a” = effective depth to controid of tendons b= width of web for flanged beam ¥¢_ allowable ultimate shear stress (as for reinforced concrete) Y= ultimate shear force acting on section PRESTRESSED CONCRETE 375 ultimate moment acting on section 8 fy fy ls the moment necessary 10 produce zero stress at the eatrere tensile fibre which is a distance y from the centroid of ihe section, where fa the concrete compressive stress at this level due to preszess and'a factor of safety of 08 is applied to this value ‘Thus i AT< My it follows that Vo, will always be greater than the applied ‘ultimate shear force, and a check on cracking Is thus incorporsted. The vertical ‘component of prestressing force should not be added to the value of Vc, obtsined from this expression Upper Limit to Shear Force ‘A farther upper limit to shear force must be imposed to avoid web crushing and this is achieved by limiting the value of shear stress so that Fbd <7qgy in the same way as for reinforced concrete. Yaya Is the maximum allowable ultimete shear stress with a value which may be calculated as the lesser of 0.8, oF 5 Nom? Design Frocedire ‘The usual design procedure consists of eating the shea estan of the cracked and untacked sections at intervals along the length ofthe ember for comparison with the applled ultimate shear force. The lower of te two values obtaned from he analyses must be taken shear resistance tthe point concer ed. This ultimate shear resistance Vc = lesser of Vor 0F Ven +Psin B IF Vises than 0.5 no shear reinforcement required, but fr vas betwen O15 Vand V+ 0-4 nominal Tinks shoul be provided such that Ae» 040 Aa > OS in and where V> Ve +0 dd designed tel is needed sl that fo inks Aa Yah fy OTF In this expreson yi the greater of the depth tothe centroid ofthe tendons or the corner logtudinal bars anchoring inks. ‘As for rein‘oroed concrete the usual design proceduce wil be to evaluate the shear resstantes ofthe sections phis nonin! tel fo dentify areas which requive more dtaled attentions iusrated in example 12.12, Example 12.12 Design of Shear Retnforcement ‘The beam cross-section in figure 12.24 is constant over a 30 m span witha pata bolic tendon profile and an eccentricity varying between ~300 mim atthe ends to =750 mm at mid-span, The beam supports an ultimate uniformly distributed load of 43 kNfm and is of grade 40 concrete. REINFORCED CONCRETE DESIGN P=2590kN 1 = 145 106 x 10° mm? 4=500% 107 mm? (2). Upper limit to shear force ¥ _ 43x18 x1000 bd 150x950 5 N/mm? S08 Vfey (= 5.1 N/mm?) and <5 N/mm? at end of bear (b)_ Uncracked resistance: sinco controid lies within web Vig = 0.67 Bh VF? + 08 feph) where fy = 0.24 Vifey = 0.24/40 = 1.51 Nfmm? and P 1590 x 10? op = B= O01 5 8 Nea? fee S00 x10" a 1080 | | be a eertronte! 012.24 Veg = 0.67 x 150 x 1500JG.51? +08 « 5.18 1.51) x 10 = 440.4 kN ~ PRESTRESSED CONCRETE a7 ‘The vertical component of prestress force is P sin 8, where f = tendon slope. ‘Tendon profile is » = Kx? + C, and if origin is at mid-span x= 0, y =O and C= Ohence atx = 15 000, y = 750 ~ 300 = 450and 450 = K x 15000" K=20x 10 therefore tendon profile is "= 2.0% 10°F, therefore tendon sope = B= 2k stead 2 =2% 20% 10° x 18000 =0.060 = tang hence 6 = 3.43° and sin 8 ~ tan 8 = 0.06. Therefore ‘vertieal component of prestress force at end of beam = 2590 x 0.06 = 155 KN Hence ‘maximum uncracked resistance = 440+ 155 = 595 kN ‘This value will decrease away from the end of the beam at 2m fiom support = 440 + 134 = 574 kN 5 m from support = 440 + 103 = 543 kN 10 m from support = 4404 51=491 kN (©) Cracked resistance Vee= (1-055 22) gba +My & ( a) M ‘This will vary long bear. At mid-span Y= 0, d = 1400 mam, If tendons stressed to 70 per cent characteristic strength at transfer and then subject to 30 per cent losses foe 2075072049 fo If total area of tendons = 3450 mm®, then 1004, _ 100% 3450 bd 150% 1400 therefore from table §.1, ve = 0.86 N/mm? for grade 40 concrete Voy #(1 = 0.85 x 0.49) 086 x 150 x 1400 x 10 = 132kN 378 REINFORCED CONCRETE DESIGN Also check minimum Veg £0.1 bd fou 4.0.1 x 180 x 1400 x V40x 107° = 133 KN At section 10 m from supports, therefore 400 ~ 2.0 x 10~¢ x $000* = 1350 mm, 1004, 187 1004s. = 1.70 and hence %e ba 13 x 5= 21S KN M=15 «43 x 10 — 10x43 x5 ©4300 KN m 1 My = 08 foe 4 my where y= y= 850 mm, and 2? Pos fea wg yg 25902-7009 850 10" 145 106 x 10 = 15.8 N/mm? hence tg = 98X15 » 145 106 350 and 2158 x 215 x10? Ve x 87 yasp+ 28x 21Sx 10 fer = (I~ 0.55 x 0.49)0.87 x 150 x 1350 oa = {128.7 + 107.9) x 10° N = 236.6 KN This calulation may be repeated for other settons co give the resistance diagram shown in figure 12.35, rom this diagram it can be sen tha at al points except for about 3 m at migspan ¥>2 1, and hence nominal reinforcement is required sich that Aw. PRESTRESSED CONCRETE 379, uncracked resistance BZ psete asso cracker 400 ce Me ultimate ssnaoe Force ¥ 200 ° . ° 5m tom 1m Support Micsoon igure 12.25 150 Mmm? hence therefore with mild-steel links fy Ay _ 04x 150 Sy 087% 250 ‘which could be provided by 8 mm stirrups at 350-mm contres (Agy/Sy = 0.287). = 0.276 se mitre py e042) «0.287 x 10-7 sans hence at the ends of the beam ‘Since this is greater then the ultimate shear foree of 645 KN, mo additional reinforcement is required. Thus provide 8 mm mild-steel links at 350 mm centres throughout. Appendix ‘Typical Weights and Live Loads 1 b= 0.454 kg A48 N force LIbjfi? = 4.88 key/m? = 47.9 N/m? LIbjft? = 16.02 kefm® = 157 Nim? Weights Aluminium, cast Asphialt paving Bricks, common Bricks, pressed Clay. dry Clay, wet Concrete, reinforced Glass, plate Lead Oak Pine, white Sand, dry Sand, wet Steel Water Brick wall, 115 mm thick Gypsum plaster, 25 mm thicle Glazing, single 380 kNin® APPENDIX 381 Floor and Roof Loads Nm? Classrooms 30 Dance halls 50 Flats and houses 1s passenger cars 23 s 5.0 Hospital wards 20 Hotel bedrooms 20 Offices for general use 28 Flat roofs, with access 13 Flat roofs, no access 075 Bar Areas and Perimeters Sectional reas of groups of bars (mm?) Bar Number of bars Be (mm) 1 304 5 6 7 8 9 6 283 56.6 84.9 113 142 170 198 226 255283 8 503 101 151 201 352 302 352 402 453.503 10 785 157-236 314-393 471-550 628707785 12 113 226-339-452. 566 679 792 905 1020 1130 16 201 402-603-804 10101210 1410 1610 1810 2010 20 314 62B_—943.:1260 1870 1890 2200 2510 2830 3140 25 491-397” 1470 1960 2450 2950 3440 3930 4420 4910 32 804 1610 2410 3220 4020 4830 5630 6430 7240 8040 401260 2510 3770 5030 62807540 880010100 11300 12600 Perimeters a weights of bars Rarsie(mm) 6 8 10 12 16 20 2 32 40 Perimeter(mm) 18.85 25.1 314 37.7 $0.2 628 78S 1005 1256 Weight (kg/im) 0.222 0.395 0.616 0.888 1.579 2.466 3.854 6.313 9.864 ‘Bar woights based ona density 3° 7850 kg 382 APPENDIX, Sectional areas per metre width For vatfous bar spacings (mm?) Spacing of bars 50751001280 «175200280300. 565 377-283-226 «189162 142113 94.3 8 1010 S71 503.402 «335-287-252 2M 168 101570 1080785628 «533 449 303-314 262 12-2260 1510-1130 905-754 646 56645237 16 4020 2680 20/0 1610 1340 1150 1010 804 670 20 628041903120 25)0 2090 1800 1570 1260 1050 25 9820 6550 4910-3930 3270 2810 2450 1960 1640 32 16100 10700 8040 6430 5360 4600 4020 3220 2620 40 25100 16800 12600 10100 8380 7180 6280 5030 4190 Shear Reinforcement Aulsy for varying stirrup diameter and spacing Stirrup Stirrup spacing (mm) diameter (mm) 85 90 100 125 150 175 200 225 250 275 300 8 1183 1.118 1.006 0.805 0.671 0.575 0,503 0.447 0.402 0.366 0.335 10 1.847 1.744 1.57 1.236 1.047 0.897 0.785 0.698 0.628 0.571 0.523 12 2.6592.511 2.26 11808 1.507 1.291 1.13 1.004 0.904 0.822 0.753 16 47294.4674.02 3.216 2.68 2.2972.01 1.787 1.608 1.462 1.34 APPENDIX 383 ‘Anchorage and Lap Requirements ‘Anchorage lengths (anchorage length L = Kg x bar size) Kx fos = 25 3035 Mormon. Plain (250) Tension 39 3633 31 Compression 32 2» OD 28 Deformed Type 1 (460) “ Tension 31 6 483 40 Compression 41 3734 2 Deformed Type 2 (460) Tension 4 3734 32 Compression 32 29 7 26 Basic lap lengths in tension and compression (lap length = K;, x bar size) Ky feu 85 3038 40 or more Plain (250) 39 26 33 31 Deformed Type 1 (460) 31 46 3 40 Deformed Type 2 (460) 41 a 34 32 Minimum lap lengths : 15 bar se of 300 Refer to figute 5.8 for ineeased lp lengths t certain Tocatlons ins member section ‘Type Land 2 bare ase desctsed in sto 1.6.2, Further Reading (2) British Standards BS 1881 Methods of testing concrete BS 4449 Specification for hot rolled steet bars forthe reinforcement of concrete ‘BS4461. Specification for cold worked steel bars forthe reinforcement of concrete ‘BS 4466. Specification for bending dintensions and scheduling of reinforcement Jor concrete BS 4482 Hard drawm mild steel wire for the reinforcement of concrete BS 4483. Stee! fabric for the reinforcement of concrete BS 5075 Concrete admixtures BS'5896. Specification for high tensile steel wire strand for the prestessing of ‘concrete BS 6399. Design loading for buildings BS 8097 Code of practice forthe design of concrete structures for retaining aqueous liquids BS 8110. Structural use of concrete, Perts 1, 2and 3 CPF Code of basic data for the design of buildings Chapter V Loading Part 2 Wind loads P8004 Foundarions (b) Textbooks and Other Publications R. D. Anchor, Design of Liguid Retaining Structures (Blackie) J.-H. Bungey, The Testing of Concrete in Structures (Susrey University Press) BLP. Hughes, Limi Stare Theory for Reinforced Conerere (Pitman) R. Hulse and W. H. Mosley, Reinforced Concrete Design by Computer (Macmillan) R Hulse and W. H. Mosley, Prestressed Concrete Design by’ Computer (Macmillan) M.K. Hurst, Prestressed Concrete Design (Chapman and Hall) LL Jones and R. H, Wood, Yield Line Anaiysis of Slabs (Chatto and Windus) F.K. Kong and R. I. Evans, Remforced and Prestressed Gonerete (Nelson) K. Leet, Reinforced Concrete Design (MeGrasv-Hill) 384 FURTHER READING 385 TY. Lin and N.H. Bums, Design of Prosiresed Concrete Structures (Wiley) ‘TJ. MacGinley, Retnforced Concrete (Spon) S$’ Mindess and J. F. Young, Concrete (Prentice-Hall) A.M. Neville. Properties of Concrete (Pitman) R Park and W. L. Gamblo, Reinforced Concrete Slabs (Wiley) R_ Park and R. Paulay, Reinforced Concrete Structures (Wiley) CE, Reynolds and J. C. Stedman, Reinforced Concrete Designers Handbook (Viewpoint Publications) F, Sawko (ed.), Developments in Prestressed Concrete, Vols 1 and 2 (Applied Science) Handbook on BS 8110: 1986 (Viewpoint Publications) Index Age factors 3 ‘Analysis of structures ‘beams 26-33 column moment 39, 42-3, 739, 249-52 ddamnaged structure 153 frames 35-47, 249 lateral loads 44-7 retaining walls 316-26 Analysis of the section bending 57,.61.67 elastic 91-7 Flanged 68-73 uuncracked 95-7 with axial load 79-88 ‘Anchorage bond 103-6 ‘Anchorage bond lengths 105, 383 ‘Areas of bars. 381-2 Bolanced fallure 82-3 Bars. see Reinforcement Bases see Footings Beams ‘analysis of moments and shears 26-33 analysis of sections 57, 61, 67 cantilever 187 coatiayons 26-33, 180-7 deflections 119-32, 357-62 design 154-91 design charts $9, 65-6, 161 Soubly reinforced 43-8, 162-7 effective spans 156 fone span 29, 187-8, 173, 177 prestressed 327-1 reinforcement details 113-17, 164, 170-2, 175 singly reinforced 58-62, 160-2 sizing 156-7 Bearing pressures 271-5, 320 Bearing stresces at a bend 179 Bending moments ‘coefficients 34, 183, 206 ‘envelopes 34, 42, 170, 181 redistribution 48-52, 75-9, 180 Bending with axial load 79-88, 239 Bends and hooks 108, 179 Bent-up bars 101-2, 178 Biaxial bending 238-61 Bond, anchorage 103-6 Bond lengths 383, Braceé columns 35, 239, 246-7 Bundled bars 115 Cantilever beams 187-8 Cantilever retaining walls 317-26 Characteristic loads 17, 24, 380-1 (Characteristic material strengths 13, 0 Circumference of bars 381 Coefficients of bending moments and shears 34, 183, 206 Columns anslysis of section 79-88, axislly loaded 246 biaxial bending 258-61 braced 35, 239, 246-7 devign 239-69 design charts 80-8, 258 elfective height 241-2 386 INDEX 387 loading arrangements. 36, 42, 240,25) moments 36, 42, 240, 251, 266, 263 ontectanaulat section 86-8, 28-4 reinforcement details 244-6 short 241-4, 246-7 simplified design 246-7, 262-4 slender 241-3, 264.9 Substitute frame 36, 42, 251 ‘unsymmetrically reinforced 253-8 Combined footings 280-4 Compression reinforcement 63-8, 162-7 Conerste ‘age factor 3 characteristic strength 13, 17 cover 12, 113-14, 146,272, 312 cracking "7, 134-44, 297, 303-5 creep 11, 124, 354, 360, durability’ 12, 144-8 clastic modulus 4-5, 123, 360 shrinkage 7-11, 124, 141-4, 297, 304, 355 stress-strain curve 3, 5,54 thermal expansion 2, 7.10, 141-4, 298-303 Continuous beams analysis 29-34 curtailment of bars 170-2 design 182-7 envelopes 34, 181 loading arrangements 26-7, 31 moment and shear coefficients 34, 183 Counterfort retaining walls 317 Cayer to reinforcement 12, 113-L4, 146, 272, 312 Cracking control 7, 140-1, 297 Flexural 134-41 thermal and shrinkage 7-11, 141-4 Creep 11, 124, 354, 360 Creop coofficients 124, 354 Critical section 194, 276 Critical span 345 Critical steel ratio 141, 302, 308 Curtailment of bars 170-2 Cumatures 125-6 Dead loads 19,25, 380 Defleetions 119-32, 198, 357-62 Design charts eamns 59, 89-6, 163 columns 80-8, 348 Diagonal tension 98, 197, 372 Diseribution stool 117, 200 Doubly reinforced beams 63-8, 162-7 Dowels 245,275 Durability 12, 144-6 Bad bearing pressures 271-, 320-1 Eifective depth $6, 156 Effective flange width 167 Effective height of a column 241-2 Effective span 156, 201, 206 Elastic analysis of a Section 91-7, 308-12 Elastic modulus concrete 4-5, 123, 360 steel S445 End blocks 362 Envelopes, bending moment and shear force 34, 42, 170, 181 Equivalent rectangular stress block 55-7 Factors of safety global 19-20 partial 18-19, 270 Fire resistance 16, 113-15, 147 Flanged section see T-beams Flat slab 214-21 Floors see Slabs Footings allowable soil pressures 271 combined 280-4 horizontal loads 271, 294 pad 273-9 piled 291-5 raft 289-91 stp 284-6 strip 286-9 Foundations see Footings Frames analysis 35-47 braced 35-43 laterally losded 44-7 loading arrangements 26-8, 36, 239-40 unbraced $5, 44-7, 239 388 INDEX Gravity retaining walls 316-17, 319 Hooks and bends 105,175, 179 Joints construction 298-9 contraction and expansion 299— 302, 306, 322 Tap lengths 106-7, 383 Laps. 106-7, 383 Libeams see T-beams Lever arm 38-9, 77, 161 Lever arm curve '59, 161 imi state design 15-23, 303-8 Limit states serviceability 16, 112-44, 270 ultimate 16 Links "99-101, 108-9, 164, 174-8, 188, 2451 375 Load combinations 26-8 Loading arangements 26-7, 36, 240, 271, 319-20 Loads characteristic 17 dead 25 imposed or tive 25,303, 319 typical values 380-1 Longtterm deflection 119-20, 123-9, 387, 360 Loss of prestress 352-7 Material properties 1-14, 54 Maximum bar spacing 113-15, 117, 164, 175, 180, 200, 245 Maximum steol areas 116, 159, 284-5 Minimum bar spacing 115, 174 Minimum steel areas 115-17, 159, 176, 194, 200, 213, 244-5, 305) Modular ratio 92, 308 Modulus of elasticity. see Elastic ‘modulus Moment cocfficients 34, 208, 210, 216 Moment envelopes ‘34, 41, 170, 181 Moment redistribution 48-52, $7, 78-9, 162, 193 Moments in columns 36, 39-47, 239-44, 287-64 Neutrabaris depth 56-7, 59, 69 Nominal reinforcement 115-17, 159, 176, 194, 200, 213, 244s, 308 Non-rectangular section 86-8, 261-4 Overturning 22, 27, 273, 318-20 Pad footings 273-9 Parabola, properties of 89-90 Partial safety factors 18-19, 303, 319 Permissibie bearing pressures 271 Permissible stresses 21, 304, 309-10, 339 Piled foundations 291-5 Prestressed concrete analysis and design 327-79 able zone 349-52 Geflections 357-62 end block 362-5 losses 352-7 post-tensioning 332-3, 341 retensioning 331-2, 341 shear 372-9 iransfer stress 347-8 ultimate strength 365-79 Punching shear 194-8, 217, 220-1, 276 Raft foundations 289-91 Rectangular stress block 55-9 ‘Rectangular-parabolic stress block 54, 89-91 Redistribution of moments 48-52, 57, 75-9, 162, 193 Reinforcement areas 381-2 bond lenaths 383 characteristic strengths 14, 17 circumference 381 lap lengths 106, 383 ‘maximum and minimum areas, 115-17, 159, 176, 194, 200, 213, 244-5, 305 properties 6, 13-14, $45 Spacing 113215, 119, 164, 174-5, 180, 200, 245 torsion 107-11, 188-91 uuntensioned 370-2 Retaining walls anelysis and design 316-26 cantilever 317-18, 320, 322-6 INDEX 389 counterfort 217 gravity 316-17, 319 Sesviceability limit state cracking "134-84, 303-5, 308-10 deflections 119-38, 357-62 durability 12,112, 144-6. factors of safety 18-19 fire resistance 114-15, 147 Shea beams 30, 98-103, 174-9 concrete stresses 101, 174 Footings 276, 278-9 prestressed beams 372-9 Punching 194-8, 217, 220-1, reinforcement 99-103, 174-9, 375~ slabs “193-8, 220-1 torsion 107-10, 188-91 Short columns 241-4, 246-7 Shrinkage 7-11, 124-5, 131-2, 141-4, 304, 355 Slabs ‘continuous, spanning one direc- tion 206-8 fat 214-21 hollow block 222-3 ‘one span, spanning one direction 201-6 ribbed 222-5 spanning two directions 209-14 stair 226-30 sitip method 236-8 Slender column 241-4, 264-9 Spacing of reinforcement. 113-15, 117, 164, 174-8, 180, 200, 245) Spen-effective depth ratios 117-19, 132-4, 198-200, 217 Stability 143-53, 271, 318-20 Stairs 226-30 Steel characteristic stresses 14, 17 stress-strain curve $6, $45, 366-7 yield strains 55 Stirrups see Links Strap footings 264-6 Stress blocks 54-5, 89-92, 95 Stresses anchorage 103-6, 304 bond 103-5, 304 Sonctete, characteristic 13, 17 permissible 21, 308, 309-1 shear 102-3, 310, 37245 steel, characteristic 14,17 Stresssiain curves 3-6, $4-7, 366 Strip footings 256-9 Strip method 236-8 Substitute treme braced 35~43 column 36, 42-3 continuous beam 29-37 Tanks 296-316 Teoeams analysis 68-75 Gesign 167-9, 183 flange reinforcement 115, 168 flange width 167 second moments af area 43 span-effective depth ratio 117 Tendons 329-33 ‘Thermal cracking 10, 141-4, 304-5 Thermal movement 7, 19-11, 299 Tie forces 148-52 Torsion analysis 107-11 design 188-91 ‘Teansier stresses 330, 339-41, 347-8 Transmission length 331 ‘Triangular stress block 91-7, ‘Ultimate limit stare factors of safety 18, 19, 303, 319 loading arrangements 26-7, 193, 240, 318-21 prestressed concrete 365-79 stability 27, 271, 318-20 Uncracked section $5-7, 121-2, 310-11 Untensioned steel in prestressed concrete 370-2 ‘Wateretaining structures clastic analysis 91-7 elastic design 38-12 joints 298-302, 306 limit state design 303-8 reinforcement details 312 Weights of materials 350 Wind loading 19,25, 44-7, 271 Yield tines 230-6 Yield strains 55 ‘Young's modulus see Elastic modulus

Вам также может понравиться